[NURS11A] Exam 1

Pataasin ang iyong marka sa homework at exams ngayon gamit ang Quizwiz!

At birth a newborn measured 20 inches. What length should the nurse instruct the mother to expect the baby to be at 4 months? (Calculate the anticipated length in c m and round to the nearest whole number.)

61 cm

The homecare nurse is visiting a newborn-and-mother couplet. Which nursing action has the highest priority? 1. Establish rapport with the family members. 2. Review the hospital medical records. 3. Determine the newborn's sleeping arrangements. 4. Examine the umbilical cord stump.

A

A full-term infant has just been born. Which interventions should the nurse perform first? Select all that apply. 1. Placing the infant in a radiant-heated unit 2. Suctioning the infant with a bulb syringe 3. Wrapping the infant in a blanket 4. Evaluating the newborn using the Apgar system 5. Offering a feeding of 5% glucose water

A, B, D

The 22-year-old client is scheduled for her first gynecologic examination. What can the nurse do to make the client more comfortable during this exam? Note: Credit will be given only if all correct choices and no incorrect choices are selected. Select all that apply. Create a trusting atmosphere. Show the client what the speculum looks like. Avoid telling the client what the exam involves. Ask the client why she has delayed her first Pap test this long. Provide a mirror for the client.

A, B, E

5) What is the primary carbohydrate in mammalian milk that plays a crucial role in the nourishment of the newborn? 1. Colostrum 2. Lactose 3. Lactoferrin 4. Secretory IgA

B

21) The nurse is caring for a client who delivered by cesarean birth. The client received a general anesthetic. To prevent or minimize abdominal distention, which of the following would the nurse encourage? Select all that apply. 1. Increased intake of cold beverages 2. Leg exercises every 2 hours 3. Abdominal tightening 4. Ambulation 5. Using a straw when drinking fluids

B, C, D

18) Which of the following symptoms would be an indication of postpartum blues? Select all that apply. 1. Overeating 2. Anger 3. Mood swings 4. Constant sleepiness 5. Crying

B, C, E

31) The postpartum client is about to go home. The nurse includes which subject in the teaching plan? 1. Replacement of fluids 2. Striae 3. Diastasis of the recti muscles 4. REEDA scale

C

A patient is scheduled to have the following type of incision for a cesarean birth. What advantage should the nurse explain to the patient that this type of incision has? Easier to repair Less blood loss Expedites delivery of multiple fetuses Less likely to rupture with future pregnancies

C

A mother is concerned because the anterior fontanelle swells when the newborn cries. What would the nurse include in her teaching to a new mother about the normal findings concerning the fontanelles? (Select all that apply.) A. The fontanelles might be depressed B. The fontanelles might bulge C. The fontanelles can swell with crying D. The fontanelles can pulsate with the heartbeat E. The fontanelles can swell when stool is passed

C, D, E

14) Which of the following is a risk factor for urinary retention after childbirth? 1. Multiparity 2. Precipitous labor 3. Unassisted childbirth 4. Not sufficiently recovering from the effects of anesthesia

D

7) A client had a cesarean birth 3 days ago. She has tenderness, localized heat, and redness of the left leg. She is afebrile. As a result of these symptoms, what would the nurse anticipate would be the next course of action? 1. That the client would be encouraged to ambulate freely 2. That the client would be given aspirin 650 mg by mouth 3. That the client would be given Methergine IM 4. That the client would be placed on bed rest

D

The nurse is assessing the emotional state of a client following the delivery of her newborn. Which response by the client requires further follow up by the nurse? A. Excitability B. Crying C. Quiet D. Withdrawn

D

The nurse is preparing to assess a laboring client who has just arrived in the labor and birth unit. Which statement by the client indicates that additional education is needed? 1. "You are going to do a vaginal exam to see how dilated my cervix is." 2. "The reason for a pelvic exam is to determine how low in the pelvis my baby is." 3. "When you check my cervix, you will find out how thinned out it is." 4. "After you assess my pelvis, you will be able to tell when I will deliver."

D

The student nurse notices that the newborn seems to focus on the mothers eyes. The nursing instructor explains that this newborn behavior is which of the following? 1. Habituation 2. Orientation 3. Self-quieting 4. Reactivity

B

1) A female patient with amenorrhea is suspected to have pituitary dysfunction. For which health problems should the nurse explain that the patient will most likely be evaluated? Select all that apply. 1. Cancer 2. Adenoma 3. Head trauma 4. Turner syndrome 5. Polycystic ovarian syndrome

1, 2, 3

1) The nurse is reviewing a list of families scheduled for community health visits. To visit these families according to the family life cycle each is in, in which order from first to last should the nurse visit these families? 1. Family with a 12-month-old child 2. Family whose oldest child is in the 5th grade 3. Family whose oldest child is attending college 4. Family whose youngest child just got a driver's license 5. Family whose youngest child got married last weekend 6. Family whose male partner retired from full-time employment

1, 2, 4, 3, 5, 6

1) The nurse is concerned that a patient is at risk for developing vulvovaginal candidiasis (V V C). What assessment information caused the nurse to have this concern? Select all that apply. 1. 16 weeks pregnant 2. +3 glucose in the urine 3. Elevated blood pressure 4. Type 2 diabetes mellitus 5. Edematous lower extremities

1, 2, 4

1) While reviewing data, the nurse determines that a patient is at risk for pelvic inflammatory disease. Which information did the nurse use to make this clinical determination? Select all that apply. 1. Age 23 2. Douches weekly 3. Smokes cigarettes 1 ppd 4. I U D inserted 2 months ago 5. Received H P V vaccination

1, 2, 4

A newborn who has not voided by 48 hours after birth should be assessed for which of the following? Select all that apply. 1. Restlessness 2. Pain 3. Kidney distention 4. Adequacy of fluid intake 5. Lethargy

1, 2, 4

1) A pregnant patient is concerned about the development of several urinary tract infections (U T Is) over the last few months of her pregnancy. What should the nurse explain as reasons for the development of these infections in this patient? Select all that apply. 1. Decrease in bladder tone 2. Hyperemic bladder mucosa 3. Urethral stricture and loss of micturition reflex 4. Ureters elongate and are displaced by the uterus 5. Distal ureters hypertrophy leading to ureteral stenosis

1, 2, 4, 5

When providing anticipatory guidance to a new mother, what information does the nurse convey about the newborns neurologic and sensory/perceptual functioning? Select all that apply. 1. Newborns respond to and interact with the environment in a predictable pattern of behavior, reacting differently to a variety of stresses. 2. The usual position of the newborn is with extremities partially flexed, legs near the abdomen. 3. Newborns do not react to bright light, and their eye movements do not permit them to fixate on faces or objects until they are 3 months of age. 4. Newborns have the capacity to utilize self-quieting behaviors to quiet and comfort themselves. 5. The newborn is very sensitive to being touched, cuddled, and held.

1, 2, 4, 5

A postpartum mother questions whether the environmental temperature should be warmer in the babys room at home. The nurse responds that the environmental temperature should be warmer for the newborn. This response is based on which newborn characteristics that affect the establishment of thermal stability? Select all that apply. 1. Newborns have less subcutaneous fat than do adults. 2. Infants have a thick epidermis layer. 3. Newborns have a large body surface to weight ratio. 4. Infants have increased total body water. 5. Newborns have more subcutaneous fat than do adults.

1, 3, 4

1) A college student is distraught after being diagnosed with pediculosis pubis. What should the nurse instruct this student to do to help prevent future infections? Select all that apply. 1. Both partners need to be treated 2. Take the prescribed medication for 10 days 3. Avoid all sexual contact until treated and cured 4. Wash bed linens in hot water and dry in a dryer for 20 minutes 5. Testing for other sexually transmitted infections is recommended

1, 3, 4, 5

1) The nurse is preparing instructions for a patient newly diagnosed with genital herpes. What should the nurse encourage to promote healing of the lesions? Select all that apply. 1. Take sitz baths 2. Use vaginal sprays 3. Wear cotton underwear 4. Douche after intercourse 5. Wear loose fitting clothing

1, 3, 5

A postpartum patient weighing 165 l b is prescribed a subcutaneous injection of Enoxaparin 1 m g/k g twice daily. The medication available is 50 m g/m How many m L of medication should the nurse provide for each injection? (Calculate to the nearest tenth decimal point.)

1.5 mL

During a home visit the mother of a 2 week old newborn is concerned that the baby always seems to be "wet" and wonders if this is normal. The newborn weighs 4 k How many m L of fluid should the nurse explain that the infant makes each day? (Round to the nearest whole number.)

100 mL

At birth, a newborn weighs 8 pounds 4 ounces. When discussing the infant's weight over the next week, what is the maximum amount of weight the mother should expect that the infant will lose? (Calculate to the first decimal point.)

13.2 oz

29) The nurse is ensuring that a patient has provided informed consent before agreeing to an amniocentesis. In which order should the nurse validate that informed consent was provided by the patient? 1. Information provides risk and benefits 2. Information provided clearly and concisely 3. Information included treatment alternatives 4. Information explaining the right to refuse treatment 5. Information reviews consequences if no treatment provided

2, 1, 3, 5, 4

1) A patient experiencing menopause asks what complementary and alternative therapy can be taken to reduce the symptoms. After reviewing the patient's health history, for which problems should the nurse encourage the patient to avoid taking phytoestrogens? Select all that apply. 1. Allergy to soy 2. Currently taking tamoxifen 3. Treated for breast cancer 5 years ago 4. Surgery for uterine fibroids in her 20s 5. Experiences insomnia several times a week

2, 3, 4

1) A patient seeks medical attention after being exposed to blood during a gang fight several weeks ago. For which types of hepatitis should the nurse anticipate that this patient will be tested? Note: Credit will be given only if all correct choices and no incorrect choices are selected. Select all that apply. 1. A 2. B 3. C 4. D 5. E

2, 3, 4

The nurse is teaching new parents how to dress their newborn. Which statements indicate that teaching has been effective? Select all that apply. 1. We should keep our home air-conditioned so the baby doesnt overheat. 2. It is important that we dry the baby off as soon as we give him a bath or shampoo his hair. 3. When we change the babys diaper, we should change any wet clothing or blankets, too. 4. If the babys body temperature gets too low, he will warm himself up without any shivering. 5. Our baby will have a much faster rate of breathing if he is not dressed warmly enough.

2, 3, 4, 5

1) The nurse in the community clinic is preparing educational materials to be used for teaching patients with sexually transmitted infections. What information should the nurse include regarding the medications metronidazole or tinidazole? Select all that apply. 1. Take this medication until symptoms disappear 2. Abstain from all alcohol while taking these medications 3. Stop taking oral contraceptives while taking these medications 4. Abstain from all alcohol for 72 hours after completing tinidazole 5. Abstain from all alcohol for 24 hours after completing metronidazole

2, 4, 5

The nurse is caring for a newborn 30 minutes after birth. After assessing respiratory function, the nurse would report which findings as abnormal? select all that apply 1. Respiratory rate of 66 breaths per minute 2. Periodic breathing with pauses of 25 seconds 3. Synchronous chest and abdomen movements 4. Grunting on expiration 5. Nasal flaring

2, 4, 5

1) A female patient comes into the clinic because of concerns about a sore that was present on her labia but spontaneously healed. During the interview the patient asks what could occur if the infection is syphilis. In what order should the nurse explain the course of this sexually transmitted infection? 1. Latent period with no lesions 2. Development of a chancre sore 3. Skin eruptions and sore throat occur 4. Tertiary stage with various symptoms 5. Development of a fever, weight loss, and malaise

2, 5, 3, 1, 4

1) The nurse is preparing an educational seminar for a group of middle-aged healthy women on health screening recommendations. What information should the nurse include during this educational session? Select all that apply. 1. Get a Pap test every 3 years 2. Schedule mammograms every 5 years 3. Get testing for H I V before the age of 60 4. Have a screening for colorectal cancer 5. Have blood pressure measured every year if 140/90

3, 4

Clinical risk factors for severe hyperbilirubinemia include which of the following? Select all that apply. 1. African American ethnicity 2. Female gender 3. Cephalohematoma 4. Bruising 5. Assisted delivery with vacuum or forceps

3, 4, 5

28) The nurse is reviewing the Quality and Safety Education for Nurses (Q S E N) competencies while preparing an in-service program to address safety in the neonatal intensive care unit. In which order should the nurse present these competencies? 1. Safety 2. Informatics 3. Patient-centered care 4. Quality improvement 5. Evidence-based practice 6. Teamwork and collaboration

3, 6, 5, 4, 1, 2

A pregnant patient's first day of her last menstrual period was 6/14. What would be this patient's estimated date of birth?

3/21

1) The nurse is preparing to meet with a female patient to review the most appropriate contraceptive method. In which order should the nurse complete the steps of this process? 1. Emphasize actions if pregnancy occurs 2. Instruct on the use of the selected method 3. Review side effects and warning symptoms 4. Assess for medical contraindications to specific methods 5. Learn about lifestyle, attitudes, religious beliefs and plans for children

4, 5, 2, 3, 1

A newborn weighs 7 l b 10 ounces at birth. What is the maximum number of calories that the nurse should instruct the mother that the baby needs to consume each day? (Calculate to the nearest whole number.)

400 calories

The manager of a maternal-child care area is preparing information to share with nursing staff regarding the leading causes of infant death in the United States. In which order, from most to least frequent, should the manager provide this information? 1. S I D S 2. Low birth weight 3. Unintentional injuries 4. Maternal complications 5. Congenital malformation

5, 2, 1, 4, 3

1) A patient is concerned about contracting herpes genitalis from a sexual partner and asks the nurse what to expect if the infection is present. In which order should the nurse explain the infection to the patient? 1. Emotional trigger occurs 2. Lesions spontaneously appear 3. Take oral acyclovir as prescribed 4. Virus enters a dormant phase with no lesions 5. Development of single or multiple blister-like vesicles

5, 3, 4, 1, 2

The nurse is instructing a new mother on the amount and frequency of bottle-feeding for her newborn. The mother plans to use formula that is available in 6-ounce cans. If the infant ingests 25 m L for each of 8 feedings per day, how many cans of formula should the mother have available for a week? (Round to the nearest whole number.)

8 cans

At 3 weeks a newborn weighs 8 l b 1 ounce. What percent of this body weight should the nurse explain to the mother as being water? (Calculate the average weight in ounces to the first decimal point.)

93.5 oz

1) A client comes to the clinic complaining of difficulty urinating, flu-like symptoms, genital tingling, and blister-like vesicles on the upper thigh and vagina. She denies having ever had these symptoms before. The medication the physician is most likely to order would be: A) Oral acyclovir B) Ceftriaxone I M C) Azithromycin P O D) Penicillin G I M

A

1) A client in the women's clinic asks the nurse, "How is the cervical mucus method of contraception different from the rhythm method?" The appropriate response by the nurse is that the cervical mucus method is which of the following? A) More effective for women with irregular cycles B) Not acceptable to women of many different religions C) Harder to work with than the rhythm method D) Requires an artificial substance or device

A

1) A female client with an intrauterine device calls the clinic because she is unable to locate the strings after her last menstrual period. What should the nurse counsel this client? A) Schedule an appointment immediately B) Wait a few days and recheck for the strings C) Take a tub bath and then recheck for the strings D) Perform a douche and then recheck for the strings

A

1) A female college student comes into the student health clinic, concerned about being pregnant from unprotected intercourse the evening before. What should the school nurse counsel this student? A) "Take Plan B One Step now." B) "Take 1 pill of Plan B now and the second pill in 5 days." C) "Take one half of Plan B One Step now and the second half in 3 days." D) "Wait 5 days and take 1 pill of Plan B followed by the second pill in 2 days."

A

1) A female patient schedules an appointment for a gynecologic examination. Which finding should indicate to the nurse that the patient is experiencing a vaginal infection? A) Foul odor from used tampons B) Scant menstrual flow at the end of the cycle C) Abdominal bloating a few days prior to menstruation D) Saturating a tampon every 2 hours during menstruation

A

1) A patient and her partner are being treated for trichomoniasis. What should the nurse emphasize when teaching the couple about this infection? A) Avoid intercourse until symptom free B) Ensure a repeat test is completed in 3 months C) Limit alcohol intake while taking metronidazole D) Have annual screening for recurrence of the infection

A

1) A patient schedules an appointment to be seen in the community clinic for dysuria, urgency, frequency, blood in the urine, and low back pain. For which health problem should the nurse provide care for this patient? A) Cystitis B) Pyelonephritis C) Glomerulonephritis D) Asymptomatic bacteriuria

A

1) A premenopausal female received a recommendation by her healthcare provider to have a bone mineral density (B M D) test done. What should the nurse identify as being the reason for the test at this time in the patient's life? A) History of an eating disorder B) Takes N S A I Ds for osteoarthritis C) Lives with a spouse who smokes cigarettes D) Surgery for carpal tunnel syndrome last year

A

1) After a pelvic examination, a patient is scheduled for tests to diagnose pelvic inflammatory disease. Which finding from the physical examination suggested to the nurse practitioner that further testing is required? A) Cervical tenderness B) Greenish vaginal discharge C) Open sores along the vagina D) Condylomata acuminata on the vulva

A

1) Care delivered by nurse-midwives can be safe and effective and can represent a positive response to the healthcare provider shortage. Nurse-midwives tend to use less technology, which often results in which of the following? A) There is less trauma to the mother. B) More childbirth education classes are available. C) They are instrumental in providing change in the birth environment at work. D) They advocate for more home healthcare agencies.

A

1) The charge nurse is looking at the charts of laboring clients. Which client is in greatest need of further intervention? A) Woman at 7 c m, fetal heart tones auscultated every 90 minutes B) Woman at 10 c m and pushing, external fetal monitor applied C) Woman with meconium-stained fluid, internal fetal scalp electrode in use D) Woman in preterm labor, external monitor in place

A

1) The client reports relief from headaches when she rubs the temples on each side of her head. The nurse understands that this is a form of which of the following? A) Acupressure B) Acupuncture C) Reflexology D) Hydrotherapy

A

1) The fetal heart rate baseline is 140 beats/min. When contractions begin, the fetal heart rate drops suddenly to 120, and rapidly returns to 140 before the end of the contraction. Which nursing intervention is best? A) Assist the client to change position. B) Apply oxygen to the client at 2 liters per nasal cannula. C) Notify the operating room of the need for a cesarean birth. D) Determine the color of the leaking amniotic fluid.

A

1) The laboring client's fetal heart rate baseline is 120 beats per minute. Accelerations are present to 135 beats/min. During contractions, the fetal heart rate gradually slows to 110, and is at 120 by the end of the contraction. What nursing action is best? A) Document the fetal heart rate. B) Apply oxygen via mask at 10 liters. C) Prepare for imminent delivery. D) Assist the client into Fowler's position.

A

1) The nurse assesses the postpartum client to have moderate lochia rubra with clots. Which nursing intervention would be appropriate? 1. Assess fundus and bladder status. 2. Catheterize the client. 3. Administer Methergine IM per order. 4. Contact the physician immediately.

A

1) The nurse auscultates the F H R and determines a rate of 112 beats/min. Which action is appropriate? A) Inform the maternal client that the rate is normal. B) Reassess the F H R in 5 minutes because the rate is low. C) Report the F H R to the doctor immediately. D) Turn the maternal client on her side and administer oxygen.

A

1) The nurse is assessing the baseline fetal heart rate for a client in labor. What action should the nurse take first? A) Measure the fetal heart rate for 10 minutes B) Round the heart rate to increments of 5 beats/minute C) Exclude periods of marked variation D) Calculate the mean (average) heart rate

A

1) The nurse is caring for a client with fetal heart rate monitoring, and the fetus is discovered to have tachycardia. Which complication should the nurse anticipate in the fetus? A) Infection B) Umbilical cord compression C) Vagus nerve stimulation D) Hypoxemia

A

1) The nurse is caring for a postpartal client of Hmong descent who immigrated to the United States 5 years ago. The client asks for the regular hospital menu because American food tastes best. The nurse assesses this response to be related to which of the following cultural concepts? A) Acculturation B) Ethnocentrism C) Enculturation D) Stereotyping

A

1) The nurse is completing the health history for a client desiring the Essure method of permanent sterilization. What should the nurse specifically ask when assessing this client? A) "Are you allergic to any metals?" B) "How many children do you have?" C) "When was your last menstrual period?" D) "Is your spouse aware of the procedure?"

A

1) The nurse is interviewing an adolescent client. The client reports a weight loss of 50 pounds over the last 4 months, and reports running at least 5 miles per day. The client asserts that her menarche was 5 years ago. Her menses are usually every 28 days, but her last menstrual period was 4 months ago. The client denies any sexual activity. Which is the best statement for the nurse to make? A) "Your lack of menses might be related to your rapid weight loss." B) "It is common and normal for runners to stop having any menses." C) "Increase your intake of iron-rich foods to reestablish menses." D) "Adolescents rarely have regular menses, even if they used to be regular."

A

1) The nurse is preparing a program about osteoporosis for a group of community members. What should the nurse emphasize as being the greatest risk factor for the development of this disorder? A) Family history B) Caucasian race C) Sedentary lifestyle D) Low lifetime intake of calcium

A

1) The nurse is preparing to assess the fetus of a laboring client. Which assessment should the nurse perform first? A) Perform Leopold maneuvers to determine fetal position. B) Count the fetal heart rate between, during, and for 30 seconds following a uterine contraction (U C). C) Dry the maternal abdomen before using the Doppler. D) The diaphragm should be cooled before using the Doppler.

A

1) The nurse is reviewing the F H R monitor for a client in labor. The rhythm strip yields the following result: How should the nurse interpret this pattern? A) Moderate variability B) Minimal variability C) Absent variability D) Marked variability

A

1) The nurse is teaching a class to the community on mind-based therapies. A class participant gives an example of a friend with leukemia who was taught by her complementary therapist to concentrate on making antibodies that will fight and kill the cancer cells in the bloodstream. How would the nurse identify this technique? A) Guided imagery B) Qigong C) Biofeedback D) Homeopathy

A

1) The nurse is telling a new client how advanced technology has permitted the physician to do which of the following? A) Treat the fetus and monitor fetal development. B) Deliver at home with a nurse-midwife and doula. C) Have the father act as the coach and cut the umbilical cord. D) Breastfeed a new baby on the delivery table.

A

1) The nurse obtains a health history from four clients. To which client should she give priority for teaching about cervical cancer prevention? A) Age 30, treated for P I D B) Age 25, monogamous C) Age 20, pregnant D) Age 27, uses a diaphragm

A

1) The nurse seeing a client just diagnosed with Chlamydia trachomatis knows that which client is at greatest risk for the infection? A) 16-year-old sexually active girl, using no contraceptive B) 22-year-old mother of two, developed dyspareunia C) 35-year-old woman on oral contraceptives D) 48-year-old woman with hot flashes and night sweats

A

1) When analyzing data collected during a sexual history, the nurse notes that a patient has limited information about contraception. What should the nurse do to address this patient's need? A) Provide the patient with the information B) Suggest that the patient talk with the nurse practitioner C) Schedule an appointment for the patient to see the midwife D) Discuss the implications if contraception is not used correctly

A

21) The nurse understands that the classic symptom of endometritis in a postpartum client is which of the following? 1. Purulent, foul-smelling lochia 2. Decreased blood pressure 3. Flank pain 4. Breast is hot and swollen

A

A 26-year-old client is having her initial prenatal appointment. The client reports to the nurse that she suffered a pelvic fracture in a car accident 3 years ago. The client asks whether her pelvic fracture might affect her ability to have a vaginal delivery. What response by the nurse is best? 1. "It depends on how your pelvis healed." 2. "You will need to have a cesarean birth." 3. "Please talk to your doctor about that." 4. "You will be able to delivery vaginally."

A

A breastfeeding postpartum client reports sore nipples to the nurse during a home visit. What intervention would be the highest priority? 1. Infant positioning 2. Use of the breast shield 3. Use of breast pads 4. Type of soap used

A

A client delivered 30 minutes ago. Which postpartal assessment finding would require close nursing attention? 1. A soaked perineal pad since the last 15-minute check 2. An edematous perineum 3. The client experiencing tremors 4. A fundus located at the umbilicus

A

A client describes breast swelling and tenderness. What piece of data would be most important for the nurse to gather initially? Timing of the symptoms Birth control method Method of breast self-examination Diet history

A

A laboring client's obstetrician has suggested amniotomy as a method for creating stronger contractions and facilitating birth. The client asks, "What are the advantages of doing this?" What should the nurse cite in response? 1. Contractions elicited are similar to those of spontaneous labor. 2. Amniotomy decreases the chances of a prolapsed cord. 3. Amniotomy reduces the pain of labor and makes it easier to manage. 4. The client will not need an episiotomy.

A

A new mother at 36 hours post-delivery has asked to be discharged to home. The nurse explains that criteria for discharge before the newborn is 48 hours old include which of the following? 1. The newborn's respiratory rate is less than 60/min. 2. Singleton birth at a minimum 35 weeks' gestation. 3. The newborn has passed at least three spontaneous stools. 4. The newborn has normal and stable vital signs for 24 hours before discharge.

A

A postpartum client calls the nursery to report that her newborn's umbilical cord stump is draining, and has a foul odor. What is the nurse's best response? 1. "Take your newborn to the pediatrician." 2. "Cover the cord stump with gauze." 3. "Apply Betadine around the cord stump." 4. "This is normal during healing."

A

A woman gave birth last week to a fetus at 18 weeks' gestation after her first pregnancy. She is in the clinic for follow-up, and notices that her chart states she has had one abortion. The client is upset over the use of this word. How can the nurse best explain this terminology to the client? 1. "Abortion is the obstetric term for all pregnancies that end before 20 weeks." 2. "Abortion is the word we use when someone has miscarried." 3. "Abortion is how we label babies born in the second trimester." 4. "Abortion is what we call all babies who are born dead."

A

A woman has been having contractions since 4 a.m. At 8 a.m., her cervix is dilated to 5 cm. Contractions are frequent, and mild to moderate in intensity. Cephalopelvic disproportion (CPD) has been ruled out. After giving the mother some sedation so she can rest, what would the nurse anticipate preparing for? 1. Oxytocin induction of labor 2. Amnioinfusion 3. Increased intravenous infusion 4. Cesarean section

A

A woman is admitted to the birth setting in early labor. She is 3 cm dilated, -2 station, with intact membranes and FHR of 150 beats/min. Her membranes rupture spontaneously, and the FHR drops to 90 beats/min with variable decelerations. What would the initial response from the nurse be? 1. Perform a vaginal exam. 2. Notify the physician. 3. Place the client in a left lateral position. 4. Administer oxygen at 2 L per nasal cannula.

A

Abdominal hysterectomy is generally recommended for which condition? Severe endometriosis Removal of the ovaries Suspected or confirmed cancer removal Abnormal uterine bleeding

A

After being in labor for several hours with no progress, a client is diagnosed with CPD (cephalopelvic disproportion), and must have a cesarean section. The client is worried that she will not be able to have any future children vaginally. After sharing this information with her care provider, the nurse would anticipate that the client would receive what type of incision? 1. Transverse 2. Infraumbilical midline 3. Classic 4. Vertical

A

Appropriate nursing interventions for the application of erythromycin ophthalmic ointment (Ilotycin) include which of the following? 1. Massaging eyelids gently following application 2. Irrigating eyes after instillation 3. Using a syringe to apply ointment 4. Instillation is in the upper conjunctival surface of each eye

A

At birth, an infant weighed 6 pounds 12 ounces. Three days later, he weighs 5 pounds 2 ounces. What conclusion should the nurse draw regarding this newborns weight? 1. This weight loss is excessive. 2. This weight loss is within normal limits. 3. This weight gain is excessive. 4. This weight gain is within normal limits.

A

Babies should sleep in what position every time they are put down for sleep? 1. On their backs 2. On their stomachs 3. On their left sides 4. On their right sides

A

By inquiring about the expectations and plans that a laboring woman and her partner have for the labor and birth, the nurse is primarily doing which of the following? 1. Recognizing the client as an active participant in her own care. 2. Attempting to correct any misinformation the client might have received. 3. Acting as an advocate for the client. 4. Establishing rapport with the client.

A

During a postpartum home visit, which step should the nurse take to establish a caring relationship? 1. Ask family members how they want to be addressed. 2. Do a portion of what the nurse agrees to do for the family, to avoid overwhelming them. 3. Speak directly to the father when asking questions. 4. Present information to the family instead of asking questions.

A

During newborn resuscitation, how does the nurse evaluate the effectiveness of bag-and-mask ventilations? 1. The rise and fall of the chest 2. Sudden wakefulness 3. Urinary output 4. Adequate thermoregulation

A

How would the nurse best analyze the results from a client's sonogram that shows the fetal shoulder as the presenting part? 1. Breech, transverse 2. Breech, longitudinal 3. Breech, frank 4. Vertex, transverse

A

In planning care for a new family immediately after birth, which procedure would the nurse most likely withhold for 1 hour to allow time for the family to bond with the newborn? 1. Eye prophylaxis medication 2. Drying the newborn 3. Vital signs 4. Vitamin K injection

A

On assessment, a labor client is noted to have cardiovascular and respiratory collapse and is unresponsive. What should the nurse suspect? 1. An amniotic fluid embolus 2. Placental abruption 3. Placenta accreta 4. Retained placenta

A

The 12-year-old client reports that menarche occurred 5 months ago. She has had bleeding every day this month, and is very worried. The nurse should explain that the most common cause of this bleeding is which of the following? Dysfunctional uterine bleeding (D U B) Diabetes mellitus (D M) Pregnancy Von Willebrand's disease

A

The charge nurse is looking at the charts of laboring clients. Which client is in greatest need of further intervention? 1. Woman at 7 cm, fetal heart tones auscultated every 90 minutes 2. Woman at 10 cm and pushing, external fetal monitor applied 3. Woman with meconium-stained fluid, internal fetal scalp electrode in use 4. Woman in preterm labor, external monitor in place

A

The client gave birth to a 7 pound, 14 ounce female 30 minutes ago. The placenta has not yet delivered. Manual removal of the placenta is planned. What should the nurse prepare to do? 1. Start an IV of lactated Ringer's. 2. Apply anti-embolism stockings. 3. Bottle-feed the infant. 4. Send the placenta to pathology.

A

The client has asked the nurse why her cervix has only changed from 1 to 2 cm in 3 hours of contractions occurring every 5 minutes. What is the nurse's best response to the client? 1. "Your cervix has also effaced, or thinned out, and that change in the cervix is also labor progress." 2. "When your perineal body thins out, your cervix will begin to dilate much faster than it is now." 3. "What did you expect? You've only had contractions for a few hours. Labor takes time." 4. "The hormones that cause labor to begin are just getting to be at levels that will change your cervix."

A

The client has been pushing for 3 hours, and the fetus is making a slow descent. The partner asks the nurse whether pushing for this long is normal. How should the nurse respond? 1. "Your baby is taking a little longer than average, but is making progress." 2. "First babies take a long time to be born. The next baby will be easier." 3. "The birth would go faster if you had taken prenatal classes and practiced." 4. "Every baby is different; there really are no norms for labor and birth."

A

The client tells the nurse that she has come to the hospital so that her baby's position can be changed. The nurse would begin to organize the supplies needed to perform which procedure? 1. A version 2. An amniotomy 3. Leopold maneuvers 4. A ballottement

A

The client's Pap smear result is A S C-U Which statement is the best way for the nurse to explain this A S C-U S result? "Abnormal cells of an unknown cause." "Cancer has invaded the upper cervix." "High-grade squamous intraepithelial lesion (H S I L), which includes C I " "The focus of the Pap smear is the detection of high-risk pregnancy." "The cervical cells are abnormal and the reason why is severe dysplasia and carcinoma in situ."

A

The fetal heart rate baseline is 140 beats/min. When contractions begin, the fetal heart rate drops suddenly to 120, and rapidly returns to 140 before the end of the contraction. Which nursing intervention is best? 1. Assist the client to change position. 2. Apply oxygen to the client at 2 liters per nasal cannula. 3. Notify the operating room of the need for a cesarean birth. 4. Determine the color of the leaking amniotic fluid.

A

The home care nurse is examining a 3-day-old infant. The childs skin on the sternum is yellow when blanched with a finger. The parents ask the nurse why jaundice occurs. What is the best response from the nurse? 1. The liver of an infant is not fully mature, and doesnt conjugate the bilirubin for excretion. 2. The infant received too many red blood cells after delivery because the cord was not clamped immediately. 3. The yellow color of your babys skin indicates that you are breastfeeding too often. 4. This is an abnormal finding related to your babys bowels not excreting bilirubin as they should.

A

The laboring client is complaining of tingling and numbness in her fingers and toes, dizziness, and spots before her eyes. The nurse recognizes that these are clinical manifestations of which of the following? 1. Hyperventilation 2. Seizure auras 3. Imminent birth 4. Anxiety

A

The laboring client presses the call light and reports that her water has just broken. What would the nurse's first action be? 1. Check fetal heart tones. 2. Encourage the mother to go for a walk. 3. Change bed linens. 4. Call the physician.

A

The laboring client's fetal heart rate baseline is 120 beats per minute. Accelerations are present to 135 beats/min. During contractions, the fetal heart rate gradually slows to 110, and is at 120 by the end of the contraction. What nursing action is best? 1. Document the fetal heart rate. 2. Apply oxygen via mask at 10 liters. 3. Prepare for imminent delivery. 4. Assist the client into Fowler's position.

A

The mother of a 2-day-old male has been informed that her child has sepsis. The mother is distraught and says, I should have known that something was wrong. Why didnt I see that he was so sick? What is the nurses best reply? 1. Newborns have immature immune function at birth, and illness is very hard to detect. 2. Your mothering skills will improve with time. You should take the newborn class. 3. Your baby didnt get enough active acquired immunity from you during the pregnancy. 4. The immunity your baby gets in utero doesnt start to function until he is 4 to 8 weeks old.

A

The mother of a 3-day-old infant calls the clinic and reports that her babys skin is turning slightly yellow. What should the nurse explain to the mother? 1. Physiologic jaundice is normal, and peaks at this age. 2. The newborns liver is not working as well as it should. 3. The baby is yellow because the bowels are not excreting bilirubin. 4. The yellow color indicates that brain damage might be occurring.

A

The nurse administered oxytocin 20 units at the time of placental delivery. Why was this primarily done? 1. To contract the uterus and minimize bleeding 2. To decrease breast milk production 3. To decrease maternal blood pressure 4. To increase maternal blood pressure

A

The nurse assesses the newborn's ears to be parallel to the outer and inner canthus of the eye. The nurse documents this finding to be: A. A normal position B. Facial paralysis C. A possible chromosomal abnormality D. Prematurity

A

The nurse at the prenatal clinic has four calls to return. Which phone call should the nurse return first? 1. Client at 32 weeks, reports headache and blurred vision. 2. Client at 18 weeks, reports no fetal movement in this pregnancy. 3. Client at 16 weeks, reports increased urinary frequency. 4. Client at 40 weeks, reports sudden gush of fluid and contractions.

A

The nurse auscultates the FHR and determines a rate of 112 beats/min. Which action is appropriate? 1. Inform the maternal client that the rate is normal. 2. Reassess the FHR in 5 minutes because the rate is low. 3. Report the FHR to the doctor immediately. 4. Turn the maternal client on her side and administer oxygen.

A

The nurse has completed a community education session on growth patterns of infants. Which statement by a participant indicates that additional teaching is needed? 1. "Newborns should regain their birth weight by 1 week of age." 2. "Breastfed and formula-fed babies have different growth rates." 3. "Formula-fed infants regain their birth weight earlier than breastfed infant." 4. "Healthcare providers consider breastfeeding to be the 'gold standard' for neonatal nutrition."

A

The nurse has completed an initial physical assessment for a client admitted to the birthing unit. Which action should the nurse take next? A. Obtain the client's social history B. Document the physical assessment findings C. Report findings to the physician D. Perform interventions for pain management

A

The nurse is answering phone calls at the pediatric clinic. Which call should the nurse return first? Mother of a 2-week-old infant who doesn't make eye contact when talked to Father of a 1-week-old infant who sleeps through the noise of an older sibling Father of a 6-day-old infant who responds more to mother's voice than to father's voice Mother of a 3-week-old infant who has begun to suck on the fingers of the right hand

A

The nurse is assessing a new mother 2 days after a normal vaginal delivery. The mother has chosen not to breastfeed. What would an abnormal finding be? 1. Weight loss of 3 pounds 2. Small amount of breast milk expressed 3. Pink striae on the abdomen 4. Lochia serosa

A

The nurse is caring for a female client with a history of pelvic inflammatory disease (P I D) who reports having difficulty conceiving after unprotected sex for over 2 years. Which deviation from the norm does the nurse recognize is most likely the cause of the client's infertility? Non-patent fallopian tube Unfavorable cervical mucus Absence of ovulation Abnormal endometrial preparation

A

The nurse is caring for four newborns who have recently been admitted to the newborn nursery. Which labor event puts the newborn at risk for an alteration of health? 1. The infant's mother has group B streptococcal (GBS) disease. 2. The infant's mother had an IV of lactated Ringer's solution. 3. The infant's mother had a labor that lasted 12 hours. 4. The infant's mother had a cesarean birth with her last child.

A

The nurse is collecting information during the health history assessment for the client profile during the initial prenatal visit. Which question is appropriate when assessing the current pregnancy? "What was the date of your last menstrual period?" "How many times have you been pregnant?" "What were your children's birth weights?" "How many living children do you have?"

A

The nurse is completing the discharge teaching of a young first-time mother. Which statement by the mother requires immediate intervention? 1. "I will put my baby to bed with his bottle so he doesn't get hungry during the night." 2. "My baby will probably have a bowel movement each breastfeeding, and will wet often." 3. "Nursing every 2 to 3 hours is normal, for a total of 8 to 12 feedings every day." 4. "I will drink fenugreek tea from my grandmother to prevent my milk from coming in."

A

The nurse is evaluating the effectiveness of phototherapy on a newborn. Which evaluation indicates a therapeutic response to phototherapy? 1. The newborn maintains a normal temperature 2. An increase of serum bilirubin levels 3. Weight loss 4. Skin blanching yellow

A

The nurse is instructing parents of a newborn about voiding and stool characteristics. Which of the following would be considered an abnormal pattern? 1. Large amounts of uric acid crystals in the first days of life 2. At least 6 to 10 wet diapers a day after the first few days of life 3. 1 to 2 stools a day for formula-fed baby 4. Urine that is straw to amber color without foul smell

A

The nurse is making an initial visit to a postpartum family's home. The mother states that she is having difficulty with breastfeeding. Which resource should the nurse tell the family about? 1. The lactation consultant at the hospital 2. Free immunizations through the county public health department clinics 3. Sources of free formula at a local food pantry 4. A support group for mothers who are experiencing postpartum depression

A

The nurse is making client assignments for the next shift. Which client is most likely to experience a complicated labor pattern? 1. 34-year-old woman at 39 weeks' gestation with a large-for-gestational-age (LGA) fetus 2. 22-year-old woman at 23 weeks' gestation with ruptured membranes 3. 30-year-old woman at 41 weeks' gestation and estimated fetal weight 7 pounds 8 ounces 4. 43-year-old woman at 37 weeks' gestation with hypertension

A

The nurse is observing a student nurse care for a neonate undergoing intensive phototherapy. Which action by the student nurse indicates an understanding of how to provide this care? 1. Urine specific gravity is assessed each voiding. 2. Eye coverings are left off to help keep the baby calm. 3. Temperature is checked every 6 hours. 4. The infant is taken out of the isolette for diaper changes.

A

The nurse is performing a vaginal exam on a client who was admitted to the birthing unit after her membranes ruptured, and discovers a cord prolapse. Which intervention is priority at this time? A) Pushing the presenting fetal part upward B) Administering oxygen C) Initiating intravenous fluid D) Inserting an indwelling bladder catheter

A

The nurse is performing an assessment on an infant whose mother states that she feeds the infant in a supine position by propping the bottle. Based on this information, what would the nurse include in the assessment? 1. Otoscopic exam of the eardrum 2. Bowel sounds 3. Vital signs 4. Skin assessment

A

The nurse is preparing a class on breastfeeding for pregnant women in their first trimester. The women are from a variety of cultural backgrounds, and all speak English well. Which statement should the nurse include in this presentation? 1. "Although some cultures believe colostrum is not good for the baby, it provides protection from infections and helps the digestive system to function." 2. "Some women are uncomfortable with exposing their breasts to nurse their infant, but it really isn't a big deal. You will get used to it." 3. "No religion prescribes a feeding method, so you all can choose whatever method makes the most sense to you." 4. "In most cultures, it is culturally acceptable to speak about intimate matters in front of their families."

A

Two hours after delivery, a client's fundus is boggy and has risen to above the umbilicus. What is the first action the nurse would take? 1. Massage the fundus until firm 2. Express retained clots 3. Increase the intravenous solution 4. Call the physician

A

The nurse is preparing new parents for discharge with their newborn. The father asks the nurse why the baby's head is so pointed and puffy-looking. What is the best response by the nurse? 1. "His head is molded from fitting through the birth canal. It will become more round." 2. "We refer to that as 'cone head,' which is a temporary condition that goes away." 3. "It might mean that your baby sustained brain damage during birth, and could have delays." 4. "I think he looks just like you. Your head is much the same shape as your baby's."

A

The nurse is scheduling a client for an external cephalic version (ECV). Which finding in the client's chart requires immediate intervention? 1. Previous birth by cesarean 2. Frank breech ballotable 3. 37 weeks, complete breech 4. Failed ECV last week

A

The nurse is teaching a class on vaginal birth after cesarean (VBAC). Which statement by a participant indicates that additional information is needed? 1. "Because the scar on my belly goes down from my navel, I am not a candidate for a VBAC." 2. "My first baby was in a breech position, so for this pregnancy, I can try a VBAC if the baby is head-down." 3. "Because my hospital is so small and in a rural area, they won't let me attempt a VBAC." 4. "The rate of complications from VBAC is lower than the rate of complications from a cesarean."

A

The nurse is teaching a group of new parents about their infants. The infants are all 4 weeks of age or younger. Which statement should the nurse include? "Your baby will respond to you the most if you look directly into his eyes and talk to him." "Each baby is different. Don't try to compare your infant's behavior with any other child's behavior." "If the sound level around your baby is high, the baby will wake up and be fussy or cry." "If your baby is a cuddler, it is because you rocked and talked to her during your pregnancy."

A

The nurse is teaching a prenatal class about feeding methods. A father-to-be asks the nurse which method, breast or formula, leads to the fastest infant growth and weight gain. Which response by the nurse is best? 1. "In the first 3 to 4 months breastfed babies tend to gain weight faster." 2. "In the first 3 to 4 months there is no difference in weight gain." 3. "In the first 3 to 4 months bottle-fed babies grow faster." 4. "In the first 3 to 4 months growth isn't as important as your comfort with the method."

A

The nurse is working with a woman who is undergoing chemotherapy for breast cancer. The client states, "First, the cancer seemed unreal. Now I feel like I can cope." What is the nurse's best response? "Women with breast cancer often go through several stages of adjustment." "Women with breast cancer cope better than their partners cope." "Women with breast cancer seek multiple opinions before starting treatment." "Women with breast cancer become angry after treatment begins."

A

The nurse knows that a contraindication to the induction of labor is which of the following? 1. Placenta previa 2. Isoimmunization 3. Diabetes mellitus 4. Premature rupture of membranes

A

The nurse manager is examining the descriptive statistics of increasing teen pregnancy rates in the community. Which inferential statistical research question would the nurse manager find most useful in investigating the reasons for increased frequency of teen pregnancy? A) What providers do pregnant teens see for prenatal care? B) What are the ages of the parents of pregnant teens in the community? C) Do pregnant teens drink caffeinated beverages? D) What do pregnant teens do for recreation?

A

The nurse obtains a health history from four clients. To which client should she give priority for teaching about cervical cancer prevention? Age 37, multiple partners Age 22, abstains from sexual intercourse Age 32, pregnant with twins Age 27, uses female condom

A

The parents of a newborn are receiving discharge teaching. The nurse explains that the infant should have several wet diapers per day. Which statement by the parents indicates that further education is necessary? 1. Our baby was born with kidneys that are too small. 2. A babys kidneys dont concentrate urine well for several months. 3. Feeding our baby frequently will help the kidneys function. 4. Kidney function in an infant is very different from that in an adult.

A

The pediatric clinic nurse is reviewing lab results with a 2-month-old infants mother. The infants hemoglobin has decreased since birth. Which statement by the mother indicates the need for additional teaching? 1. My baby isnt getting enough iron from my breast milk. 2. Babies undergo physiologic anemia of infancy. 3. This results from dilution because of the increased plasma volume. 4. Delaying the cord clamping did not cause this to happen.

A

Two hours ago, a client at 39 weeks' gestation was 3 cm dilated, 40% effaced, and +1 station. Frequency of contractions was every 5 minutes with duration 40 seconds and intensity 50 mmHg. The current assessment is 4 cm dilated, 40% effaced, and +1 station. Frequency of contractions is now every 3 minutes with 40-50 seconds' duration and intensity of 40 mmHg. What would the priority intervention be? 1. Begin oxytocin after assessing for CPD. 2. Give terbutaline to stop the preterm labor. 3. Start oxygen at 8 L/min. 4. Have the anesthesiologist give the client an epidural.

A

What condition is due to poor peripheral circulation? Acrocyanosis Mottling Harlequin sign Jaundice

A

What is one of the most common initial signs of nonreassuring fetal status? 1. Meconium-stained amniotic fluid 2. Cyanosis 3. Dehydration 4. Arrest of descent

A

What would be a normal cervical dilatation rate in a first-time mother ("primip")? 1. 1.5 cm per hour 2. Less than 1 cm cervical dilatation per hour 3. 1 cm per hour 4. Less than 0.5 cm per hour

A

Which assessment findings by the nurse would require obtaining a blood glucose level on the newborn? 1. Jitteriness 2. Sucking on fingers 3. Lusty cry 4. Axillary temperature of 98°F

A

Which client in the gynecology clinic should the nurse see first? 32-year-old taking gonadotropins, reporting extremity edema 15-year-old, no menses for past 4 months 18-year-old seeking information on contraception methods 31-year-old, taking progestins, reports increasing dyspareunia

A

Which nonspecific immune mechanism has the ability of antibodies and phagocytic cells to clear pathogens from an organism? 1. Complement 2. Coagulation 3. Inflammatory response 4. Phagocytosis

A

Which of the following is a benefit of delayed umbilical cord clamping for the preterm infant? 1. Fewer infants require blood transfusion for anemia 2. Fewer infants require blood transfusion for high blood pressure 3. Increase in the incidence of intraventricular hemorrhage 4. Increase in incidence of infant breastfeeding

A

Which of the following is a localized, easily identifiable soft area of the infant's scalp, generally resulting from a long and difficult labor or vacuum extraction? Caput succedaneum Cephalohematoma Molding Depressed fontanelles

A

While caring for a client admitted to the birthing unit, the nurse suspects that the client may be experiencing a uterine rupture. Which assessment finding should the nurse expect to appear first? A) Nonreassuring fetal heart rate B) Constant abdominal pain C) Loss of fetal station D) Cessation of contractions

A

While performing a uterine assessment on a client in the birthing unit, the nurse notes a loss of fetal station and a change in uterine shape. The client reports constant abdominal pain, uterine tenderness, and is exhibiting signs of shock. Which condition should the nurse suspect? A) Uterine rupture B) Anaphylactoid syndrome of pregnancy C) Circumvallate placenta D) Breech presentation

A

1) The nurse is caring for a client who has experienced premature rupture of membranes. For which maternal implication(s) should the nurse monitor? Note: Credit will be given only if all correct choices and no incorrect choices are selected. Select all that apply. A) Infection B) Preterm labor C) Dyspnea D) Discomfort E) E) Uterine distention

A, B

Premonitory signs of labor include which of the following? Note: Credit will be given only for all correct choices and no incorrect choices. Select all that apply. 1. Braxton Hicks contractions 2. Cervical softening and effacement 3. Weight gain 4. Rupture of membranes 5. Sudden loss of energy

A, B, D

1) Why is it important for the nurse to understand the type of family that a client comes from? Select all that apply. A) Family structure can influence finances. B) Some families choose to conceive or adopt without a life partner. C) The nurse can anticipate which problems a client will experience based on the type of family the client has. D) Understanding if the client's family is nuclear or blended will help the nurse teach the client the appropriate information. E) The values of the family will be predictable if the nurse knows what type of family the client is a part of.

A, B

A new mother is concerned about a mass on the newborn's head. The nurse assesses this to be a cephalohematoma. Which of the following characteristics would indicate a cephalohematoma? (Select all that apply.) A. The mass appears only on one side of the head. B. The mass appeared on the second day after birth. C. The head appears asymmetrical. D. The mass appears larger when the newborn cries. E. The mass overrides the suture line.

A, B

The nurse has taken a detailed social history from a client admitted to the birthing unit. Which insights may the nurse gain as a result of this assessment? Select all that apply. A. Social habits B. Psychologic factors C. Presence of H I V D. Readiness for discharge E. Need for bed rest

A, B

The nurse is assessing the comfort of the parents during the third stage of labor. Which finding(s) indicate that the parents feel comfortable during this stage? Select all that apply. A. Talking to the newborn B. Verbally expressing feelings of pride C. Requesting to dim the lights D. Preferring limited contact with the newborn initially E. Immediately placing phone calls

A, B

The nurse is cross-training maternal-child health unit nurses to provide home-based care for parents after discharge. Which statements indicate that additional teaching is required? Note: Credit will be given only if all correct choices and no incorrect choices are selected. Select all that apply. "The behavioral assessment should be done as soon after birth as possible." "The behavioral assessment can be performed without input from parents." "The behavioral assessment might be incomplete in a 1-hour home visit." "The behavioral assessment includes orientation and motor activity." "The behavioral assessment can detect neurological impairments."

A, B

1) During a wellness visit, a 50-year-old female experiencing menopause says that she jogs three times a week and feels like her symptoms are becoming worse. What should the nurse recommend to help with the discomfort of menopause? Select all that apply. A) Yoga B) Tai chi C) Meditation D) Weight lifting E) Kegel exercises

A, B, C

1) During the assessment phase of a family, the community nurse recognizes that culture influences childrearing and childbearing in which of the following ways? Select all that apply. A) Beliefs about the importance of children B) Beliefs and attitudes about pregnancy C) Norms regarding infant feeding D) Acculturation is important in rearing children E) Time orientation to the future is very important

A, B, C

1) The nurse is discharging a client after hospitalization for pelvic inflammatory disease (P I D). Which statements indicate that teaching was effective? Select all that apply. A) "I might have infertility because of this infection." B) "It is important for me to finish my antibiotics." C) "Tubal pregnancy could occur after P I D." D) "My P I D was caused by a yeast infection." E) "I am going to have an I U D placed for contraception."

A, B, C

1) What issues should the nurse consider when counseling a client on contraceptive methods? Select all that apply. A) Cultural perspectives on menstruation and pregnancy B) Effectiveness of the method C) Future childbearing plans D) Whether the client is a vegetarian E) Age at menarche

A, B, C

18) A postpartal client recovering from deep vein thrombosis is being discharged. What areas of teaching on self-care and anticipatory guidance should the nurse discuss with the client? Select all that apply. 1. Avoid crossing the legs. 2. Avoid prolonged standing or sitting. 3. Take frequent walks. 4. Take a daily aspirin dose of 650 mg. 5. Avoid long car trips.

A, B, C

24) A nurse suspects that a postpartum client has mastitis. Which data support this assessment? Select all that apply. 1. Shooting pain between breastfeedings 2. Late onset of nipple pain 3. Pink, flaking, pruritic skin of the affected nipple 4. Nipple soreness when the infant latches on 5. Pain radiating to the underarm area from the breast

A, B, C

A client attending a prenatal class asks why episiotomies are performed. The nurse explains that risk factors that predispose women to episiotomies include which of the following? Note: Credit will be given only if all correct choices and no incorrect choices are selected. Select all that apply. 1. Large or macrosomic fetus 2. Use of forceps 3. Shoulder dystocia 4. Maternal health 5. Shorter second stage

A, B, C

During the initial prenatal visit, the nurse obtains a weight of 42 kg (92.4 lb). The nurse must further assess the client for information about which of the following? Note: Credit will be given only if all correct and no incorrect choices are selected. Select all that apply. 1. Eating habits 2. Foods regularly eaten 3. Income limitations 4. Blood pressure and pulse rate 5. Weight loss during pregnancy

A, B, C

Nursing research is vital to do which of the following? Select all that apply. A) Expand the science of nursing. B) Foster evidence-based practice. C) Improve client care. D) Visually depict nursing management. E) Plan and organize care.

A, B, C

The Quality and Safety Education for Nurses (Q S E N) project focused on competencies in which areas? Select all that apply. A) Client-centered care B) Teamwork and collaboration C) Evidence-based practice D) Family planning E) Injury and violence prevention

A, B, C

The labor nurse would not encourage a mother to bear down until the cervix is completely dilated, to prevent which of the following? Note: Credit will be given only for all correct choices and no incorrect choices. Select all that apply. 1. Maternal exhaustion 2. Cervical edema 3. Tearing and bruising of the cervix 4. Enhanced perineal thinning 5. Having to perform an episiotomy

A, B, C

The nurse is caring for a client who could be at risk for uterine rupture. The nurse is monitoring the fetus closely for which of the following? Note: Credit will be given only if all correct choices and no incorrect choices are selected. Select all that apply. 1. Late decelerations 2. Bradycardia 3. Loss of ability to determine fetal station 4. Tachycardia 5. Early decelerations

A, B, C

What are some of the advantages and disadvantages of formula-feeding that a nurse should discuss with new parents? Note: Credit will be given only if all correct choices and no incorrect choices are selected. Select all that apply. 1. The nutritional value of formula depends on the proper preparation/dilution. 2. There is a potential for bacterial contamination during preparation and storage. 3. Both parents can participate in positive parent-infant interaction during feeding. 4. Refrigeration is not necessary if preparing more than one bottle at a time. 5. Formula has higher levels of essential fatty acids, lactose, cystine, and cholesterol than does breast milk.

A, B, C

Which findings would the nurse expect when assessing a newborn infected with syphilis? Note: Credit will be given only if all correct choices and no incorrect choices are selected. Select all that apply. 1. Rhinitis 2. Fissures on mouth corners 3. Red rash around anus 4. Lethargy 5. Large for gestational age

A, B, C

1) A 17-year-old high school student comes into the nurse's office to find out what to do about severe menstrual cramps. What should the nurse recommend to this student? Select all that apply. A) Rest B) Good nutrition C) Regular exercise D) Application of heat E) D & C of the uterus

A, B, C, D

1) A 40-year-old patient is being seen in the clinic for gynecological changes. Which approaches should the nurse use when completing this patient's health interview? Select all that apply. A) Avoid writing B) Clarify terms used C) Maintain eye contact D) Analyze body language E) Use simple yes-no questions

A, B, C, D

1) The nurse is caring for a client undergoing fetal heart rate monitoring, and the F H R is greater than 162 beats/min for 12 minutes. For what cause(s) should the nurse anticipate treatment? Note: Credit will be given only if all correct choices and no incorrect choices are selected. Select all that apply. A) Maternal anxiety B) Fetal asphyxia C) Prematurity D) Fetal anemia E) Maternal hypotension

A, B, C, D

3) The postpartum nurse provides anticipatory guidance for the new mother as well as teaching on self-care and infant care before discharge. Which topics should be included? Select all that apply. 1. Role changes brought on by the addition to the family unit 2. The realities of having a new baby, and how it affects previous lifestyle 3. Potential complications such as infant colic and postpartum issues 4. Sexuality and contraception 5. Toilet-training and preschool options

A, B, C, D

Characteristics of a caring relationship that the nurse cultivates when interacting with the client in the home include which of the following? Note: Credit will be given only if all correct choices and no incorrect choices are selected. Select all that apply. 1. Regard for the client 2. Genuineness 3. Empathy 4. Rapport 5. Attachment

A, B, C, D

What are the nurse's responsibilities when teaching the new mother about infant feeding? Note: Credit will be given only if all correct choices and no incorrect choices are selected. Select all that apply. 1. The nurse should be well informed about infant nutrition and feeding methods. 2. The nurse should provide accurate and consistent information. 3. The nurse should use each interaction to support the parents and promote the family's sense of confidence. 4. The nurse should familiarize the mother with information about community resources that might be helpful after discharge. 5. The nurse should aggressively promote breastfeeding, even if the parents have decided to bottle-feed their infant.

A, B, C, D

What assessments of the newborn should be completed during the initial home visit? Note: Credit will be given only if all correct choices and no incorrect choices are selected. Select all that apply. 1. Sleep-wake cycles 2. Parent-infant interaction 3. Fontanelles 4. Umbilical cord status 5. Breast engorgement

A, B, C, D

Childbirth preparation offers several advantages including which of the following? Note: Credit will be given only for all correct choices and no incorrect choices. Select all that apply. 1. It helps a pregnant woman and her support person understand the choices in the birth setting. 2. It promotes awareness of available options. 3. It provides tools for a pregnant woman and her support person to use during labor and birth. 4. Women who receive continuous support during labor require more analgesia, and have more cesarean and instrument births. 5. Each method has been shown to shorten labor.

A, B, C, E

During a visit to the obstetrician, a pregnant client questions the nurse about the potential need for an amniotomy. The nurse explains that an amniotomy is performed to do which of the following? Note: Credit will be given only if all correct choices and no incorrect choices are selected. Select all that apply. 1. Stimulate the beginning of labor 2. Augment labor progression 3. Allow application of an internal fetal electrode 4. Allow application of an external fetal monitor 5. Allow insertion of an intrauterine pressure catheter

A, B, C, E

The nurse has completed the physical assessment of a client in early labor, and proceeds with the social assessment. A social history of the client would include which of the following? Select all that apply. 1. Use of drugs and alcohol 2. Family violence or sexual assault 3. Current living situation 4. Type of insurance 5. Availability of resources

A, B, C, E

The nurse is preparing teaching material for a new mother. What should the nurse include when instructing on areas to include when observing the infant? Note: Credit will be given only if all correct choices and no incorrect choices are selected. Select all that apply. Touch Vision Hearing Diaper care General appearance

A, B, C, E

Risk factors for tachysystole include which of the following? Note: Credit will be given only if all correct choices and no incorrect choices are selected. Select all that apply. 1. Cocaine use 2. Placental abruption 3. Low-dose oxytocin titration regimens 4. Uterine rupture 5. Smoking

A, B, D

The client is undergoing an emergency cesarean birth for fetal bradycardia. The client's partner has not been allowed into the operating room. What can the nurse do to alleviate the partner's emotional distress? Note: Credit will be given only if all correct choices and no incorrect choices are selected. Select all that apply. 1. Allow the partner to wheel the baby's crib to the newborn nursery. 2. Allow the partner to be near the operating room where the newborn's first cry can be heard. 3. Have the partner wait in the client's postpartum room. 4. Encourage the partner to be in the nursery for the initial assessment. 5. Teach the partner how to take the client's blood pressure.

A, B, D

29) When preparing for and performing an assessment of the postpartum client, which of the following would the nurse do? Select all that apply. 1. Ask the client to void before assessing the uterus. 2. Inform the client of the need for regular assessments. 3. Defer client teaching to another time. 4. Perform the procedures as gently as possible. 5. Take precautions to prevent exposure to body fluids.

A, B, D, E

35) The nurse is providing postpartum care to an obese client. As part of care for this client, the nurse should do which of the following? Select all that apply. 1. Apply sequential compression devices 2. Have the mother ambulate as early as possible 3. Encourage bottle-feeding over breastfeeding 4. Supervise breastfeeding 5. Instruct the client on signs of infection

A, B, D, E

The nurse is caring for a client who is showing a sinusoidal fetal heart rate pattern on the monitor. The nurse knows that possible causes for this pattern include which of the following? Note: Credit will be given only if all correct choices and no incorrect choices are selected. Select all that apply. 1. Fetal anemia 2. Chronic fetal bleeding 3. Maternal hypotension 4. Twin-to-twin transfusion 5. Umbilical cord occlusion

A, B, D, E

1) Fetal factors that possibly indicate electronic fetal monitoring include which of the following? Note: Credit will be given only if all correct choices and no incorrect choices are selected. Select all that apply. A) Meconium passage B) Multiple gestation C) Preeclampsia D) Grand multiparity E) Decreased fetal movement

A, B, E

18) What information should the nurse include when teaching the postpartal client and partner about resumption of sexual activity? Select all that apply. 1. Couples should be encouraged to abstain from intercourse until the episiotomy is healed and the lochial flow has stopped. 2. Postpartum women often experience vaginal dryness, and should be encouraged to use some kind of lubrication initially during intercourse. 3. Breastfeeding the newborn after intercourse can reduce the chance of milk spouting from the nipples. 4. Maternal changes in libido are usually indicative of psychological depression. 5. Maternal fatigue is often a significant factor limiting the resumption of sexual intercourse.

A, B, E

29) Risk factors associated with increased risk of thromboembolic disease include which of the following? Select all that apply. 1. Diabetes mellitus 2. Varicose veins 3. Hypertension 4. Adolescent pregnancy 5. Malignancy

A, B, E

A new mother is planning to bottle feed her infant and wants helps with suppressing lactation. What should the nurse suggest to help this new mother? Note: Credit will be given only if all correct choices and no incorrect choices are selected. Select all that apply. Wear a 24-hour support bra Apply cabbage leaves to the breast tissue Apply warm compresses every 4 to 6 hours Massage lotion on the breasts 3 times a day Avoid all nipple stimulation for 7 to 10 days

A, B, E

During labor, the client at 4 cm suddenly becomes short of breath, cyanotic, and hypoxic. The nurse must prepare or arrange immediately for which of the following? Note: Credit will be given only if all correct choices and no incorrect choices are selected. Select all that apply. 1. Intravenous access 2. Cesarean delivery 3. Immediate vaginal delivery 4. McRoberts maneuver 5. A crash cart

A, B, E

Fetal factors that possibly indicate electronic fetal monitoring include which of the following? 1. Meconium passage 2. Multiple gestation 3. Preeclampsia 4. Grand multiparity 5. Decreased fetal movement

A, B, E

To maintain a healthy temperature in the newborn, which of the following actions should be taken? Note: Credit will be given only if all correct choices and no incorrect choices are selected. Select all that apply. 1. Keep the newborn's clothing and bedding dry. 2. Reduce the newborn's exposure to drafts. 3. Do not use the radiant warmer during procedures. 4. Do not wrap the newborn. 5. Encourage the mother to snuggle with the newborn under blankets.

A, B, E

When caring for a 13-year-old client in labor, how would the nurse provide sensitive care? Note: Credit will be given only if all correct choices and no incorrect choices are selected. Select all that apply. 1. Using simple and concrete instructions 2. Providing soothing encouragement and comfort measures 3. Making all decisions for the client when she expresses a feeling of helplessness 4. Deciding whom the client should allow in the room 5. Providing encouragement and support of the client's decisions

A, B, E

Which of the following activities allows the nurse to provide individualized parent teaching on the maternal-infant unit? Note: Credit will be given only if all correct choices and no incorrect choices are selected. Select all that apply. 1. Teach by example and role modeling when caring for the newborn in the client's room. 2. Teach at every opportunity, even during the night shift, if the occasion arises. 3. Teach using newborn care videos and group classes. 4. Teach using the 24-hour educational television channels in the client's room. 5. Teach using one-to-one instruction while in the client's room.

A, B, E

What are the primary complications of placenta accreta?Select all that apply. A) Maternal hemorrhage B) Insomnia C) Failure of the placenta to separate following birth of the infant D) Autonomic dysreflexia E) Shoulder dystocia

A, C

What are the primary complications of placenta accrete? Note: Credit will be given only if all correct choices and no incorrect choices are selected. Select all that apply. 1. Maternal hemorrhage 2. Insomnia 3. Failure of the placenta to separate following birth of the infant 4. Autonomic dysreflexia 5. Shoulder dystocia

A, C

1) A client is being prepared to take the oral mifepristone-vaginal misoprostol treatment for an abortion. For which reasons should the nurse instruct the client to contact the healthcare provider within 24 hours? Select all that apply. A) Nausea B) Itchy skin C) Weakness D) Stomach pain E) Bloody discharge

A, C, D

1) The nurse is reviewing the spermicidal agent nonoxynol-9 (N-9) with a client planning to use the barrier method to prevent pregnancy. What should the nurse emphasize when teaching about this preparation? Select all that apply. A) It does not cause toxicity. B) It is inserted after intercourse. C) It has no systemic side effects. D) It can be purchased over-the-counter. E) It reduces the risk of sexually transmitted infections.

A, C, D

1) The nurse suspects that a client is experiencing adverse effects from the progestin within a combined oral contraceptive. What did the nurse assess to make this clinical determination? Select all that apply. A) Pruritus B) Headache C) Hirsutism D) Weight gain E) Hypertension

A, C, D

A nurse is instructing nursing students about the procedure for vitamin K administration. What information should be included? Note: Credit will be given only if all correct choices and no incorrect choices are selected. Select all that apply. 1. Gently massage the site after injection. 2. Use a 22-gauge, 1-inch needle. 3. Inject in the vastus lateralis muscle. 4. Cleanse the site with alcohol prior to injection. 5. Inject at a 45-degree angle.

A, C, D

A postpartum patient who received epidural morphine prior to a cesarean birth is concerned about a severe headache that has persisted for several days. What should the nurse suggest to this patient? Note: Credit will be given only if all correct choices and no incorrect choices are selected. Select all that apply. Ingest fluids with caffeine Engage in moderate exercise Increase the intake of all fluids Lie in bed in a quiet dark room Restrict the intake of warm fluids

A, C, D

During a client's initial prenatal visit, the nurse must assess and document the client's current medical history, including which information? Note: Credit will be given only if all correct and no incorrect choices are selected. Select all that apply. 1. Body mass index 2. Infections before the last menstrual period 3. Homeopathic or herbal medication use 4. Blood type and Rh factor 5. History of previous pregnancies

A, C, D

Which of the following are important behaviors to assess in the neurologic assessment? Note: Credit will be given only if all correct choices and no incorrect choices are selected. Select all that apply. State of alertness Active posture Quality of muscle tone Cry Motor activity

A, C, D, E

The nurse encourages a new mother to feed the newborn as soon as the newborn shows interest. The nurse bases this recommendation on which benefits of early feedings? Note: Credit will be given only if all correct choices and no incorrect choices are selected. Select all that apply. 1. Early feedings stimulate peristalsis. 2. Colostrum is thinner than mature milk. 3. Early feedings enhance maternal-infant bonding. 4. Early feedings promote the passage of meconium. 5. Colostrum contains a high number of calories.

A, C, D

The nurse in the OB-GYN clinic is working with a client who is seeking her initial prenatal visit. The nurse will use the acronym TPAL to document the client's number of which of the following? Note: Credit will be given only if all correct and no incorrect choices are selected. Select all that apply. 1. Term infants born 2. Children living in the home 3. Pregnancies ending in abortion 4. Preterm infants born 5. Pregnancies that occurred

A, C, D

The nurse initiates newborn admission procedures and evaluates the newborn's need to remain under observation by assessing which of the following? Note: Credit will be given only if all correct choices and no incorrect choices are selected. Select all that apply. 1. Respiratory rate 2. Skin texture 3. Airway clearance 4. Ability to feed 5. Head weight

A, C, D

The nurse is explaining clinical pelvimetry to a client. The nurse explains that the anteroposterior diameters consist of which of the following? Note: Credit will be given only if all correct and no incorrect choices are selected. Select all that apply. 1. Diagonal conjugate 2. Transverse diameter 3. Conjugata vera 4. Obstetric conjugate 5. Oblique diameter

A, C, D

1) During an assessment, the nurse determines that a female patient is at risk for developing osteoporosis. Which information did the nurse use to make this clinical determination? Select all that apply. A) Body weight of 120 lb B) Plays tennis twice a week C) Smokes 2 packs per day of cigarettes D) Ingests 2 to 3 cocktails every day E) Mother diagnosed with osteoporosis

A, C, D, E

During a routine physical examination a female patient asks the nurse what can be done to prevent the development of breast cancer. What should the nurse review with the patient? Select all that apply. Exercise regularly Discuss starting tamoxifen Reduce the intake of red meat Maintain a normal body weight Reduce the intake of dietary fat

A, C, D, E

The nurse is caring for a client and her spouse during the third stage of labor. Which action(s) support initial parental-newborn attachment at this time? Select all that apply. A. Minimizing assessments B. Delaying ophthalmic antibiotics for 2 hours C. Dimming the room lights D. Talking quietly E. Providing privacy

A, C, D, E

The nurse is conducting an initial prenatal assessment for a pregnant client. Which screenings should the nurse prepare the client for during this visit? Note: Credit will be given only if all correct choices and no incorrect choices are selected. Select all that apply. Complete blood count (C B C) Glucose tolerance test (G T T) A B O and Rh typing H I V screening Urinalysis

A, C, D, E

The nurse is monitoring a postpartum patient receiving methylergonovine maleate (Methergine). Which assessment findings should the nurse identify as being expected adverse effects of this medication? Note: Credit will be given only if all correct choices and no incorrect choices are selected. Select all that apply. Nausea Leg pain Headache Hypertension Uterine cramping

A, C, D, E

The nurse is preparing teaching materials for female clients who wish to perform breast self-examination. In which order should the nurse ensure that the teaching materials present the process of inspection? Compare the breasts Study the skin surface Analyze for symmetry Study the shape and direction Look at color, thickening, edema, and venous patterns

A, C, D, E, B

1) The nurse manager is consulting with a certified nurse-midwife about a client. What is the role of the CNM? Select all that apply. A) Be prepared to manage independently the care of women at low risk for complications during pregnancy and birth. B) Give primary care for high-risk clients who are in hospital settings. C) Give primary care for healthy newborns. D) Obtain a physician consultation for any technical procedures at delivery. E) Be educated in two disciplines of nursing.

A, C, E

6) The nurse is caring for a postpartum client who is at risk for developing early postpartum hemorrhage. What interventions would be included in the plan of care to detect this complication? Select all that apply. 1. Weigh perineal pads if the client has a slow, steady, free flow of blood from the vagina. 2. Massage the uterus every 2 hours. 3. Maintain vascular access. 4. Obtain blood specimens for hemoglobin and hematocrit. 5. Encourage the client to void if the fundus is displaced upward or to one side.

A, D

A clinic nurse is preparing diagrams of pelvic shapes. Which pelvic shapes are considered least adequate for vaginal childbirth? Note: Credit will be given only for all correct choices and no incorrect choices. Select all that apply. 1. Android 2. Anthropoid 3. Gynecoid 4. Platypelloid 5. Lambdoidal suture

A, D

The nurse is teaching a prenatal class about false labor. The nurse should teach clients that false labor most likely will include which of the following? Note: Credit will be given only for all correct choices and no incorrect choices. Select all that apply. 1. Contractions that do not intensify while walking 2. An increase in the intensity and frequency of contractions 3. Progressive cervical effacement and dilatation 4. Pain in the abdomen that does not radiate 5. Contractions are at regular intervals

A, D

During a home visit, a new mother who is breastfeeding complains that her nipples are sore and cracked. Which measures should the nurse suggest? Note: Credit will be given only if all correct choices and no incorrect choices are selected. Select all that apply. 1. Alternate the baby's nursing positions throughout the day. 2. Breastfeed the baby always in a seated position 3. Avoid placing the areola in the baby's mouth. 4. Insert a finger between the infant's gums to break the latch before removing from the breast. 5. Wash the breasts with warm water, and avoid drying soaps.

A, D, E

The nurse knows that the Bishop scoring system for cervical readiness includes which of the following? Note: Credit will be given only if all correct choices and no incorrect choices are selected. Select all that apply. 1. Fetal station 2. Fetal lie 3. Fetal presenting part 4. Cervical effacement 5. Cervical softness

A, D, E

1) In assessing a new family coming to the clinic, the nurse determines they are an extended kin family because the family exhibits what as characteristics of an extended kin network family? Select all that apply. A) A sharing of a social support network B) Each family establishes their own sources of goods and services C) Elderly parents share housing D) Children are members of two nuclear families E) A sharing of goods and services

A, E

A mother and her newborn are being discharged 2 days after delivery. The general discharge instructions provided by the nurse include which of the following? Note: Credit will be given only if all correct choices and no incorrect choices are selected. Select all that apply. 1. Always place the infant in a supine position in the crib. 2. Support the infant's head when carrying for the first week or two. 3. Do not allow the baby to fall asleep in someone's arms. 4. Cover the cord stump with a bandage. 5. Use a bulb syringe to suction mucus from the infant's nostrils as necessary

A, E

The nurse is caring for a client in the transition phase of labor and notes that the fetal monitor tracing shows average short-term and long-term variability with a baseline of 142 beats per minute. What actions should the nurse take in this situation? Note: Credit will be given only if all correct choices and no incorrect choices are selected. Select all that apply. 1. Provide caring labor support. 2. Administer oxygen via face mask. 3. Change the client's position. 4. Speed up the client's intravenous. 5. Reassure the client and her partner that she is doing fine.

A, E

The nurse is preparing an educational in-service presentation about jaundice in the newborn. What content should the nurse include in this presentation? Note: Credit will be given only if all correct choices and no incorrect choices are selected. Select all that apply. 1. Physiologic jaundice occurs after 24 hours of age. 2. Pathologic jaundice occurs after 24 hours of age. 3. Phototherapy increases serum bilirubin levels. 4. The need for phototherapy depends on the bilirubin level and age of the infant. 5. Kernicterus causes irreversible neurological damage.

A, E

A telephone triage nurse gets a call from a postpartum client who is concerned about jaundice. The clients newborn is 37 hours old. What data point should the nurse gather first? 1. Stool characteristics 2. Fluid intake 3. Skin color 4. Bilirubin level

C

The client presents to the labor and delivery unit stating that her water broke 2 hours ago. Barring any abnormalities, how often would the nurse expect to take the client's temperature? 1. Every hour 2. Every 2 hours 3. Every 4 hours 4. Every shift

C

The postpartum client has chosen to bottle-feed her infant. Nursing actions that aid in lactation suppression include which of the following? 1. Warm showers 2. Pumping milk 3. Ice packs to each breast 4. Avoiding wearing a bra for 5 to 7 days

C

Which of the following systems provides a uniform format and classification of terminology based on current understanding of cervical disease? Levonorgestrel intrauterine P A L M-C O E I N Bethesda B S E

C

A client arrives in the labor and delivery unit and describes her contractions as occurring every 10-12 minutes, lasting 30 seconds. She is smiling and very excited about the possibility of being in labor. On exam, her cervix is dilated 2 cm, 100% effaced, and -2 station. What best describes this labor? 1. Second phase 2. Latent phase 3. Active phase 4. Transition phase

B

A fetal weight is estimated at 4490 grams in a client at 38 weeks' gestation. Counseling should occur before labor regarding which of the following? 1. Mother's undiagnosed diabetes 2. Likelihood of a cesarean delivery 3. Effectiveness of epidural anesthesia with a large fetus 4. Need for early delivery

B

A fetus has been diagnosed with myelomeningocele. Which of the following surgeries would be performed to correct this condition? A) Tubal ligation B) Intrauterine fetal surgery C) Cesarean section D) Sterilization

B

A nurse is evaluating the diet plan of a breastfeeding mother, and determines that her intake of fruits and vegetables is inadequate. The nurse explains that the nutritional composition of the mother's breast milk can be adversely affected by this nutritional inadequacy. Which strategy should the nurse recommend to the mother? 1. Stop breastfeeding 2. Provide newborn supplements to the newborn 3. Offer whole milk 4. Supplement with skim milk

B

A nurse is evaluating the diet plan of a breastfeeding mother. Which beverage is most likely to cause intolerance in the infant? 1. Orange juice 2. Milk 3. Decaffeinated tea 4. Water

B

A nurse is providing guidance to a group of parents of children in the infant-to-preschool age group. After reviewing statistics on the most common cause of death in this age group, the nurse includes information about prevention of which of the following? A) Cancer by reducing the use of pesticides in the home B) Accidental injury by reducing the risk of pool and traffic accidents C) Heart disease by incorporating heart-healthy foods into the child's diet D) Pneumonia by providing a diet high in vitamin C from fruits and vegetables

B

A nurse is teaching a class on the different types of uterine bleeding. The nurse explains that which of the following is one of the causes of abnormal uterine bleeding? Iron-deficiency anemia Polyps Heavy periods every 2 months Spotting between periods

B

A patient with a rectocele is experiencing progressive pain and constipation. What should the nurse expect to be indicated for this patient? Enemas Surgery Laxatives Antibiotics

B

A postpartum mother is concerned that her newborn has not had a stool since birth. The newborn is 18 hours old. What is the nurse's best response? 1. "I will call your pediatrician immediately." 2. "Passage of the first stool within 48 hours is normal." 3. "Your newborn might not have a stool until the third day." 4. "Your newborn must be dehydrated."

B

A woman in labor asks the nurse to explain the electronic fetal heart rate monitor strip. The fetal heart rate baseline is 150 with accelerations to 165, variable decelerations to 140, and moderate long-term variability. Which statement indicates that the client understands the nurse's teaching? 1. "The most important part of fetal heart monitoring is the absence of variable decelerations." 2. "The most important part of fetal heart monitoring is the presence of variability." 3. "The most important part of fetal heart monitoring is the fetal heart rate baseline." 4. "The most important part of fetal heart monitoring is the depth of decelerations."

B

At birth, an infant weighed 8 pounds 4 ounces. Three days later, the newborn is being discharged. The parents note that the baby now weighs 7 pounds 15 ounces. The nurse explains that the change in the newborn's weight is which of the following? 1. Excessive 2. Within normal limits 3. Less than expected 4. Unusual

B

Before applying a cord clamp, the nurse assesses the umbilical cord. The mother asks why the nurse is doing this. What should the nurse reply? 1. "I'm checking the blood vessels in the cord to see whether it has one artery and one vein." 2. "I'm checking the blood vessels in the cord to see whether it has two arteries and one vein." 3. "I'm checking the blood vessels in the cord to see whether it has two veins and one artery." 4. "I'm checking the blood vessels in the cord to see whether it has two arteries and two veins."

B

During an assessment of a 12-hour-old newborn, the nurse notices pale pink spots on the nape of the neck. The nurse documents this finding as: A. Nevus vasculosus B. Telangiectatic nevi C. Nevus flammeus D. A Mongolian spot

B

Four minutes after the birth of a baby, there is a sudden gush of blood from the mother's vagina, and about 8 inches of umbilical cord slides out. What action should the nurse take first? 1. Place the client in McRoberts position. 2. Watch for the emergence of the placenta. 3. Prepare for the delivery of an undiagnosed twin. 4. Place the client in a supine position.

B

In succenturiate placenta, one or more accessory lobes of fetal villi have developed on the placenta, with vascular connections of fetal origin. What is the gravest maternal danger? 1. Cord prolapse 2. Postpartum hemorrhage 3. Paroxysmal hypertension 4. Brachial plexus injury

B

In utero, what is the organ responsible for gas exchange? 1. Umbilical vein 2. Placenta 3. Inferior vena cava 4. Right atrium

B

Late preterm infants have higher infant morbidity and mortality rates than term infants. Which of the following complications can they experience? 1. Hyperglycemia 2. Jaundice 3. Motor difficulties 4. Sensory complications

B

The need for forceps has been determined. The client's cervix is dilated to 10 cm, and the fetus is at +2 station. What category of forceps application would the nurse anticipate? 1. Input 2. Low 3. Mid 4. Outlet

B

The parents of a newborn tell the nurse that their infant seems to enjoy being held and that holding him helps calm him when he cries. They ask the nurse why this is so. After explaining newborn behavior, the nurse assesses the parents' learning. Which statement indicates that teaching was effective? A. "Our baby spends more time in the active alert phase." B. "Cuddliness is a social behavior that some babies have." C. "We are lucky to have a baby with a calm disposition." D. "Some babies are easier to deal with than others."

B

The physician has determined the need for forceps. The nurse should explain to the client that the use of forceps is indicated because of which of the following? 1. Her support person is exhausted 2. Premature placental separation 3. To shorten the first stage of labor 4. To prevent fetal distress

B

The physicians/CNM opts to use a vacuum extractor for a delivery. What does the nurse understand? 1. There is little risk with vacuum extraction devices. 2. There should be further fetal descent with the first two "pop-offs." 3. Traction is applied between contractions. 4. The woman often feels increased discomfort during the procedure.

B

The postpartum homecare client asks the nurse why the visit is taking place. Which response is best? 1. "We make homecare visits to reinforce any teaching that you didn't quite grasp in the hospital." 2. "We make homecare visits to verify that both you and the baby are safe and doing well." 3. "We make homecare visits to ensure you are breastfeeding correctly." 4. "We make homecare visits to thoroughly assess your baby to make sure he is growing."

B

The prenatal clinic nurse is designing a new prenatal intake information form for pregnant clients. Which question is best to include on this form? 1. Where was the father of the baby born? 2. Do genetic diseases run in the family of the baby's father? 3. What is the name of the baby's father? 4. Are you married to the father of the baby?

B

1) A patient is being instructed on adverse effects of gonorrhea. For which reason should the nurse instruct the patient to contact the healthcare provider? A) Dysuria B) Urinary frequency C) Sharp abdominal pain D) Purulent, greenish-yellow vaginal discharge

C

1) A patient with osteoporosis wants a medication that does not need to be taken every day. What should the nurse expect to be prescribed for this patient? A) Teriparatide (Forteo) B) Alendronate (Fosamax) C) Zoledronic acid (Zometa®) D) Salmon calcitonin (Miacalcin®)

C

1) The nurse is planning a community education program on the role of complementary and alternative therapies during pregnancy. Which statement about alternative and complementary therapies should the nurse include? A) "They bring about cures for illnesses and diseases." B) "They are invasive but effective for achieving health." C) "They emphasize prevention and wellness." D) "They prevent pregnancy complications."

C

1) The nurse suspects that a female patient is experiencing amenorrhea because of ovarian failure. For which situation should the nurse assess this patient? A) Severe stress B) Recent head trauma C) Treatment for cancer D) Antianxiety medication

C

1) The student nurse is to perform Leopold maneuvers on a laboring client. Which assessment requires intervention by the staff nurse? A) The client is assisted into supine position, and the position of the fetus is assessed. B) The upper portion of the uterus is palpated, then the middle section. C) After determining where the back is located, the cervix is assessed. D) Following voiding, the client's abdomen is palpated from top to bottom.

C

1) The transcultural nursing theory was developed in 1961 by Dr. Madeleine Leininger. Its foundation is in which of the following? A) The framework categorizes a family's progression over time B) The family life cycle of a traditional nuclear family C) Anthropology and nursing D) Holistic health beliefs

C

1) What is the term for when children alternate between two homes, spending varying amounts of time with each parent in a situation called co-parenting and usually involving joint custody? A) Blended or reconstituted nuclear family B) Extended kin network family C) Binuclear family D) Extended family

C

1) When the nurse is teaching a woman about the use of a diaphragm, it is important to instruct her that the diaphragm should be rechecked for correct size how often? A) Every five years routinely B) When weight gain or loss beyond five pounds has occurred C) After each birth D) Only after significant weight loss

C

A client from Mexico has just delivered a son, and the nurse offers to assist in putting the baby to breast. Although the client indicated before the birth that she wanted to breastfeed, she is very hesitant, and says she would like to bottle-feed for the first few days. After talking to her, the nurse understands that her primary reason for wanting to delay breastfeeding is based on what cultural belief? 1. Breast milk causes skin rashes. 2. It is harmful to breastfeed immediately. 3. Colostrum is bad for the baby. 4. Thin milk causes diarrhea.

C

A client is admitted to the labor and delivery unit with contractions that are 2 minutes apart, lasting 60 seconds. She reports that she had bloody show earlier that morning. A vaginal exam reveals that her cervix is 100 percent effaced and 8 cm dilated. The nurse knows that the client is in which phase of labor? 1. Active 2. Latent 3. Transition 4. Fourth

C

A client is admitted to the labor and delivery unit with contractions that are regular, are 2 minutes apart, and last 60 seconds. She reports that her labor began about 6 hours ago, and she had bloody show earlier that morning. A vaginal exam reveals a vertex presenting, with the cervix 100% effaced and 8 cm dilated. The client asks what part of labor she is in. The nurse should inform the client that she is in what phase of labor? 1. Latent phase 2. Active phase 3. Transition phase 4. Fourth stage

C

A client is consulting a certified nurse-midwife because she is hoping for a vaginal birth after cesarean (VBAC) with this pregnancy. Which statement indicates that the client requires more information about VBAC? 1. "I can try a vaginal birth because my uterine incision is a low segment transverse incision." 2. "The vertical scar on my skin doesn't mean that the scar on my uterus goes in the same direction." 3. "There is about a 90% chance of giving birth vaginally after a cesarean." 4. "Because my hospital has a surgery staff on call 24 hours a day, I can try a VBAC there."

C

A client who wishes to have an unmedicated birth is in the transition stage. She is very uncomfortable and turns frequently in the bed. Her partner has stepped out momentarily. How can the nurse be most helpful? 1. Talk to the client the entire time. 2. Turn on the television to distract the client. 3. Stand next to the bed with hands on the railing next to the client. 4. Sit silently in the room away from the bed.

C

A laboring client asks the nurse, "Why does the physician want to use an intrauterine pressure catheter (IUPC) during my labor?" The nurse would accurately explain that the best rationale for using an IUPC is which of the following? 1. The IUPC can be used throughout the birth process. 2. A tocodynamometer is invasive. 3. The IUPC provides more accurate data than does the tocodynamometer. 4. The tocodynamometer can be used only after the cervix is dilated 2 cm.

C

A laboring client's obstetrician has suggested amniotomy as a method for inducing labor. Which assessment(s) must be made just before the amniotomy is performed? 1. Maternal temperature, BP, and pulse 2. Estimation of fetal birth weight 3. Fetal presentation, position, and station 4. Biparietal diameter

C

A laboring mother has recurrent late decelerations. At birth, the infant has a heart rate of 100, is not breathing, and is limp and bluish in color. What nursing action is best? 1. Begin chest compressions. 2. Begin direct tracheal suctioning. 3. Begin bag-and-mask ventilation. 4. Obtain a blood pressure reading.

C

A mother states that her breasts leak between feedings. Which of the following can contribute to the letdown reflex in breastfeeding mothers? 1. Pain with breastfeeding 2. Number of hours passed since last feeding 3. The newborn's cry 4. Maternal fluid intake

C

For prenatal care, the client is attending a clinic held in a church basement. The client's care is provided by registered nurses and a certified nurse-midwife. What is this type of prenatal care? A) Secondary care B) Tertiary care C) Community care D) Unnecessarily costly care

C

If a woman has the pre-existing condition of diabetes, the nurse knows that she would be prone to what high-risk factor when pregnant? 1. Vasospasm 2. Postpartum hemorrhage 3. Episodes of hypoglycemia and hyperglycemia 4. Cerebrovascular accident (CVA)

C

If the physician indicates a shoulder dystocia during the delivery of a macrosomic fetus, how would the nurse assist? 1. Call a second physician to assist. 2. Prepare for an immediate cesarean delivery. 3. Assist the woman into McRoberts maneuver. 4. Utilize fundal pressure to push the fetus out.

C

Induction of labor is planned for a 31-year-old client at 39 weeks due to insulin-dependent diabetes. Which nursing action is most important? 1. Administer 100 mcg of misoprostol (Cytotec) vaginally every 2 hours. 2. Place dinoprostone (Prepidil) vaginal gel and ambulate client for 1 hour. 3. Begin Pitocin (oxytocin) 4 hours after 50 mcg misoprostol (Cytotec). 4. Prepare to induce labor after administering a tap water enema.

C

Major perineal trauma (extension to or through the anal sphincter) is more likely to occur if what type of episiotomy is performed? 1. Mediolateral 2. Episiorrhaphy 3. Midline 4. Medical

C

New parents decide not to have their newborn circumcised. What should the nurse teach regarding care for the uncircumcised infant? 1. The foreskin will be retractable at 2 months. 2. Retract the foreskin and clean thoroughly. 3. Avoid retracting the foreskin. 4. Use soap and Betadine to cleanse the penis daily.

C

The client requires vacuum extraction assistance. To provide easier access to the fetal head, the physician cuts a mediolateral episiotomy. After delivery, the client asks the nurse to describe the episiotomy. What does the nurse respond? 1. "The episiotomy goes straight back toward your rectum." 2. "The episiotomy is from your vagina toward the urethra." 3. "The episiotomy is cut diagonally away from your vagina." 4. "The episiotomy extends from your vagina into your rectum."

C

The client vaginally delivers an infant that weighs 4750 g. Moderate shoulder dystocia occurred during the birth. During the initial assessment of this infant, what should the nurse look for? 1. Bell's palsy 2. Bradycardia 3. Erb palsy 4. Petechiae

C

The current emphasis on federal healthcare reform has yielded what unexpected benefit? A) Assessment of the details of the family's income and expenditures B) Case management to limit costly, unnecessary duplication of services C) Many healthcare providers and consumers are becoming more aware of the vitally important role nurses play in providing excellent care to clients and families D) Education of the family about the need for keeping regular well-child visit appointments

C

The nurse is planning an educational presentation on hyperbilirubinemia for nursery nurses. Which statement is most important to include in the presentation? 1. Conjugated bilirubin is eliminated in the conjugated state. 2. Unconjugated bilirubin is neurotoxic, and cannot cross the placenta. 3. Total bilirubin is the sum of the direct and indirect levels. 4. Hyperbilirubinemia is a decreased total serum bilirubin level.

C

The nurse is planning care for a newborn. Which nursing intervention would best protect the newborn from the most common form of heat loss? 1. Placing the newborn away from air currents 2. Pre-warming the examination table 3. Drying the newborn thoroughly 4. Removing wet linens from the isolette

C

The nurse is preparing a client education handout on the differences between false labor and true labor. What information is most important for the nurse to include? 1. True labor contractions begin in the back and sweep toward the front. 2. False labor often feels like abdominal tightening, or "balling up." 3. True labor can be diagnosed only if cervical change occurs. 4. False labor contractions do not increase in intensity or duration.

C

The nurse is providing care to a pregnant client diagnosed with a urinary tract infection (U T I) during a routine prenatal visit. What will the nurse educate the client about based on this data? Gestational hypertension Gestational diabetes mellitus Preterm labor Anemia

C

The nurse is providing guidance for a woman in her second trimester of pregnancy and telling her about some of the signs and symptoms that she might experience. Which statement by the client indicates that further teaching is necessary? 1. "During the third trimester, I might have frequent urination." 2. "During the third trimester, I might have heartburn." 3. "During the third trimester, I might have back pain." 4. "During the third trimester, I might have a persistent headache."

C

1) A patient experiencing symptoms of menopause asks if there are any vitamin supplements she should take at this time. Which vitamins should the nurse suggest to this patient? Select all that apply. A) Vitamin A B) Vitamin C C) Vitamin D D) Vitamin E E) Vitamin B complex

C, D, E

1) A client is planning to use condoms with a spermicidal cream as contraception. What should the nurse include when reviewing this method with the client? A) Coat the condom with spermicide before using B) Insert the spermicide 1 hour before having intercourse C) Insert the cream high into the vagina and remain supine D) Wait 15 minutes after inserting the spermicide into the vagina

D

1) A couple asks the nurse what is the safest method of sterilization. What should the nurse reply? A) "Laparotomy tubal ligation." B) "Laparoscopy tubal ligation." C) "Minilaparotomy." D) "Vasectomy."

D

1) The labor and delivery nurse is caring for a laboring client who has asked for a priest to visit her during labor. The client's mother died during childbirth, and although there were no complications during her pregnancy, the client is fearful of her own death during labor. What would be the best way for the nurse to respond? A) "Nothing is going to happen to you. We'll take very good care of you during your birth." B) "Would you like to have an epidural so that you won't feel the pain of the contractions?" C) "The priest won't be able to prevent complications, and might get in the way of your providers." D) "Would you like me to contact someone from your parish or our hospital chaplain to come see you?"

D

1) The nurse is analyzing several fetal heart rate patterns. The pattern that would be of most concern to the nurse would be which of the following? A) Moderate variability B) Early decelerations C) Late decelerations D) Accelerations

D

A nurse is caring for a newborn on a ventilator who has respiratory distress syndrome (RDS). The nurse informs the parents that the newborn is improving. Which data support the nurse's assessment? 1. Decreased urine output 2. Pulmonary vascular resistance increases 3. Increased PCO2 4. Increased urination

D

A nursing instructor is demonstrating an assessment on a newborn for the nursing students using the Ballard gestational assessment tool. The nurse explains that which of the following tests should be performed after the first hour of birth? A. Popliteal angle B. Square window sign C. Scarf sign D. Arm recoil

D

A patient is demonstrating manifestations of acute cervicitis. Which laboratory test should the nurse expect to be completed for this patient? Sedimentation rate Blood test for V D R L White blood cell count Vaginal smear for S T Is

D

A postpartum client calls the nursery to report that her 3-day-old newborn has passed a green stool. What is the nurses best response? 1. Take your newborn to the pediatrician. 2. There might be a possible food allergy. 3. Your newborn has diarrhea. 4. This is a normal occurrence.

D

After delivery, it is determined that there is a placenta accreta. Which intervention should the nurse anticipate? 1. 2 L oxygen by mask 2. Intravenous antibiotics 3. Intravenous oxytocin 4. Hysterectomy

D

As compared with admission considerations for an adult woman in labor, the nurse's priority for an adolescent in labor would be which of the following? 1. Cultural background 2. Plans for keeping the infant 3. Support persons 4. Developmental level

D

Before the newborn and mother are discharged from the birthing unit, the nurse teaches the parents about newborn screening tests that includes which of the following? 1. Preeclampsia screening 2. Congenital kidney disease screening 3. Visual screening 4. Hearing screening

D

Before the nurse begins to dry the newborn off after birth, which of the following assessment findings should the nurse document to ensure an accurate gestational rating on the Ballard gestational assessment tool? A. Size of the areola B. Creases on the sole C. Body surface temperature D. Amount and area of vernix coverage

D

During a maternal assessment, the nurse determines the fetus to be in a left occiput anterior (LOA) position. Auscultation of the fetal heart rate should begin in what quadrant? 1. Right upper quadrant 2. Left upper quadrant 3. Right lower quadrant 4. Left lower quadrant

D

During the fourth stage of labor, the client's assessment includes a BP of 110/60, pulse 90, and the fundus is firm midline and halfway between the symphysis pubis and the umbilicus. What is the priority action of the nurse? 1. Turn the client onto her left side. 2. Place the bed in Trendelenburg position. 3. Massage the fundus. 4. Continue to monitor.

D

The client has delivered a 4200 g fetus. The physician performed a midline episiotomy, which extended into a third-degree laceration. The client asks the nurse where she tore. Which response is best? 1. "The episiotomy extended and tore through your rectal mucosa." 2. "The episiotomy extended and tore up near your vaginal mucous membrane." 3. "The episiotomy extended and tore into the muscle layer." 4. "The episiotomy extended and tore through your anal sphincter."

D

The client has stated that she wants to avoid an epidural and would like an unmedicated birth. Which nursing action is most important for this client? 1. Encourage the client to vocalize during contractions. 2. Perform vaginal exams only between contractions. 3. Provide a CD of soft music with sounds of nature. 4. Offer to teach the partner how to massage tense muscles.

D

The client is at 42 weeks' gestation. Which order should the nurse question? 1. Obtain biophysical profile today. 2. Begin nonstress test now. 3. Schedule labor induction for tomorrow. 4. Have the client return to the clinic in 1 week.

D

The client is having fetal heart rate decelerations. An amnioinfusion has been ordered for the client to alleviate the decelerations. The nurse understands that the type of decelerations that will be alleviated by amnioinfusion is which of the following? 1. Early decelerations 2. Moderate decelerations 3. Late decelerations 4. Variable decelerations

D

The client is undergoing lab work and ultrasound for a possible diagnosis of polycystic ovarian syndrome (P C O S). Which problem does the nurse expect to find in the client's history? Multiple first-trimester fetal losses Dyspareunia Vulvitis Oligomenorrhea

D

The nurse assesses the newborn and notes the following behaviors: nasal flaring, facial grimacing, and excessive mucus. What is the nurse most concerned about? 1. Neonatal jaundice 2. Neonatal hypothermia 3. Neonatal hyperthermia 4. Respiratory distress

D

The nurse has assessed four newborns respiratory rates immediately following birth. Which respiratory rate would require further assessment by the nurse? 1. 60 breaths per minute 2. 70 breaths per minute 3. 64 breaths per minute 4. 20 breaths per minute

D

The nurse is caring for a client in active labor. The membranes spontaneously rupture, with a large amount of clear amniotic fluid. Which nursing action is most important to undertake at this time? 1. Assess the odor of the amniotic fluid. 2. Perform Leopold maneuvers. 3. Obtain an order for pain medication. 4. Complete a sterile vaginal exam.

D

The nurse is caring for a laboring client. A cervical exam indicates 8 cm dilation. The client is restless, frequently changing position in an attempt to get comfortable. Which nursing action is most important? 1. Leave the client alone so she can rest. 2. Ask the family to take a coffee-and-snack break. 3. Encourage the client to have an epidural for pain. 4. Reassure the client that she will not be left alone.

D

The nurse is caring for a new breastfeeding mother who is from Pakistan. The nurse plans her care so that the newborn is offered the breast on which of the following? 1. Day of birth 2. First day after birth 3. Second day after birth 4. Third to fourth day after birth

D

The nurse is caring for a premature infant in the NICU, and is going to attempt a bottle feeding with thawed breast milk. How long can thawed breast milk be stored in the refrigerator before the nurse must discard it? 1. 4 hours 2. 8 hours 3. 12 hours 4. 24 hours

D

The nurse is providing health teaching to a group of women of childbearing age. One woman states that she is a smoker, and asks about the effect of smoking on her fetus. The nurse tells her that which fetal complication can occur when the mother smokes? 1. Genetic changes in the fetal reproductive system 2. Extensive central nervous system damage 3. Addiction to the nicotine inhaled from the cigarette 4. Low birth rate

D

The nurse is speaking to a community group about the controversy regarding the length of the hospital stay for postpartum clients. Which statement indicates that a participant needs additional information? 1. "As of 1998, there's a law that requires insurance to pay for a 48-hour stay after an uncomplicated birth." 2. "The length of stay was shortened by insurance companies to decrease healthcare costs." 3. "Early discharge became more popular in the 1980s as an alternative to having a home birth." 4. "With current length-of-stay laws, newborns have no problems at home, and get recommended follow-up care."

D

The nurse is teaching a group of new parents about newborn behavior. Which statement made by a parent would indicate a need for additional information? 1. Sleep and alert states cycle throughout the day. 2. We can best bond with our child during an alert state. 3. About half of the babys sleep time is in active sleep. 4. Babies sleep during the night right from birth.

D

The nurse is teaching a pregnant client the clinical manifestations associated with preterm labor. Which client statement indicates the need for further education? "Menstrual-like cramps are a sign of preterm labor." "A dull low backache is a sign of preterm labor." "Diarrhea is a sign of preterm labor." "Vomiting is a sign of preterm labor."

D

The nurse is working with a client from Southeast Asia. The client tells the nurse that she should not put the baby to breast until her milk comes in and her breasts are warm, because "cold milk" (colostrum) is bad for the baby. After the nurse explains the benefits of colostrum, the client still insists that "cold milk" is bad. Which response by the nurse is best? 1. "What kind of formula would you like to use?" 2. "That idea is folklore. Colostrum is good for the baby." 3. "Now that you are here, you need to feed your baby the right way." 4. "Let's give the baby formula after you breastfeed."

D

The nurse is working with new parents who have recently immigrated to the United States. The nurse is not familiar with the family's cultural background. Which approach is most appropriate when discussing the newborn? 1. "You appear to be Muslim. Do you want your son circumcised?" 2. "Let me explain newborn care here in the United States." 3. "Your baby is a United States citizen. You must be very happy about that." 4. "Could you explain your preferences regarding childrearing?"

D

Which of the following would be a newborn care procedure that will decrease the probability of high bilirubin levels? 1. Monitor urine for amount and characteristics. 2. Encourage late feedings to promote intestinal elimination. 3. All infants should be routinely monitored for iron intake. 4. Maintain the newborns skin temperature at 36.5C (97.8F) or above

D

Which third-trimester client would the nurse suspect might be having difficulty with psychological adjustments to her pregnancy? 1. A woman who says, "Either a boy or a girl will be fine with me" 2. A woman who puts her feet up and listens to some music for 15 minutes when she is feeling too stressed 3. A woman who was a smoker but who has quit at least for the duration of her pregnancy 4. A woman who has not investigated the kind of clothing or feeding methods the baby will need

D

The nurse is preparing an educational program for new parents. In which order should the nurse provide the Purple Crying information? Unexpected Long lasting Pain-like face Peak of crying Resists soothing Evening and late afternoon

D, A, E, C, B, F

During the initial intrapartal assessment of a client in early labor, the nurse performs a vaginal examination. The client's partner asks why this pelvic exam needs to be done. The nurse should explain that the purpose of the vaginal exam is to obtain information about which of the following? Note: Credit will be given only if all correct choices and no incorrect choices are selected. Select all that apply. 1. Uterine contraction pattern 2. Fetal position 3. Presence of the mucous plug 4. Cervical dilation and effacement 5. Presenting part

D, E

1) A patient is being assessed for recurrent vulvovaginal candidiasis (V V C) infections. What should the nurse instruct this patient to do to help reduce the incidence of infection? Select all that apply. 1. Avoid douching 2. Use vaginal sprays 3. Wear cotton underwear 4. Avoid tight-fitting clothing 5. Apply cornstarch to the vulva

1, 3, 4

Which of the following would be considered normal newborn urinalysis values? Select all that apply. 1. Color bright yellow 2. Bacteria 0 3. Red blood cells (RBC) 0 4. White blood cells (WBC) more than 4-5/hpf 5. Protein less than 5-10 mg/dL

2, 3, 5

1) A premenopausal patient is experiencing vaginal dryness. What pharmacological intervention should the nurse suggest for this patient's symptom? A) Local low-dose vaginal estrogen B) Testosterone replacement therapy C) Menopausal hormone therapy with testosterone D) Menopausal hormone therapy with estrogen alone

A

A nurse who tells family members the sex of a newborn baby without first consulting the parents would have committed which of the following? A) A breach of privacy B) Negligence C) Malpractice D) A breach of ethics

A

Upon assessing the FHR tracing, the nurse determines that there is fetal tachycardia. The fetal tachycardia would be caused by which of the following? Note: Credit will be given only if all correct choices and no incorrect choices are selected. Select all that apply. 1. Early fetalhypoxia 2. Prolonged fetal stimulation 3. Fetal anemia 4. Fetal sleep cycle 5. Infection

A, B, C, E

1) The nurse is discussing the use of contraception with a client who has just become sexually active. What factors should the nurse include when educating the client on contraceptive methods? Select all that apply. A) Contraindications in the client's health history B) Religious or moral beliefs C) Partner's belief in the effectiveness of the choice D) Personal preferences to use method E) Future childbearing plans

A, B, D, E

12) Which findings would indicate the presence of a perineal wound infection? Note: Credit will be given only if all correct choices and no incorrect choices are selected. Select all that apply. 1. Redness 2. Tender at the margins 3. Vaginal bleeding 4. Hardened tissue 5. Purulent drainage

A, B, D, E

1) The nurse is preparing an education session for women on the prevention of urinary tract infections (U T Is). Which statement should be included? A) Lower urinary tract infections rarely occur in women. B) The most common causative organism of cystitis is E. coli. C) Wiping from back to front after a B M will help prevent a U T I. D) Back pain often develops with a lower urinary tract infection.

B

The nurse begins a prenatal assessment on a 25-year-old primigravida at 20 weeks' gestation and immediately contacts the healthcare provider because of which finding? 1. Pulse 88/minute 2. Respirations 30/minute 3. Temperature 37.4° C (99.3° F) 4. Blood pressure 118/82

B

The nurse is teaching a new mother how to encourage a sleepy baby to breastfeed. Which of the following instructions would not be included in that teaching? 1. Providing skin-to-skin contact 2. Swaddling the newborn in a blanket 3. Unwrapping the newborn 4. Allowing the newborn to feel and smell the mother's breast

B

The visiting nurse evaluates a 2-day-old breastfed newborn at home and notes that the baby appears jaundiced. When explaining jaundice to the parents, what would the nurse tell them? 1. Jaundice is uncommon in newborns. 2. Some newborns require phototherapy. 3. Jaundice is a medical emergency. 4. Jaundice is always a sign of liver disease.

B

To promote infant security in the hospital, the nurse instructs the parents of a newborn to do which of the following? 1. Keep the baby in the room at all times. 2. Check the identification of all personnel who transport the newborn. 3. Place a "No Visitors" sign on the door. 4. Keep the baby in the nursery at all times.

B

Upon delivery of the newborn, what nursing intervention most promotes parental attachment? 1. Placing the newborn under the radiant warmer. 2. Placing the newborn on the mother's abdomen. 3. Allowing the mother a chance to rest immediately after delivery. 4. Taking the newborn to the nursery for the initial assessment.

B

What is required for any women receiving oxytocin (Pitocin)? 1. CPR 2. Continuous electronic fetal monitoring 3. Administering oxygen by mask 4. Nonstress test

B

26) The nurse suspects that a client has developed a perineal hematoma. What assessment findings would the nurse have detected to lead to this conclusion? 1. Facial petechiae 2. Large, soft hemorrhoids 3. Tense tissues with severe pain 4. Elevated temperature

C

The nurse is instructing a new mother on basic infant care. For which activities should the nurse suggest that the mother use the following infant hold? Burping Bottle feeding Shampooing hair Changing a diaper

C

1) The community nurse is planning to visit a family. The grandparents are helping the adult parents with child-rearing activities. For which type of family should the nurse plan care? A) Nuclear B) Blended C) Binuclear D) Extended

D

33) The nurse is beginning the postpartum teaching of a mother who has given birth to her first child. What aspect of teaching is most important? 1. Describe the likely reaction of siblings to the new baby. 2. Discuss adaptation to grandparenthood by her parents. 3. Determine whether father-infant attachment is taking place. 4. Assist the mother in identifying the babys behavior cues.

D

A newborn is determined to have physiological jaundice. The nurse explains the steps involved in conjugation and excretion of bilirubin to the parents. Which factors would the nurse include in the explanation? Select all that apply. 1. At birth, the newborns liver begins to conjugate bilirubin or convert it from a yellow lipid-soluble pigment to a water-soluble pigment. 2. Unconjugated bilirubin can leave the bloodstream and enter the tissues, causing a yellow hue to the skin and sclera. 3. Unconjugated bilirubin results from the destruction of white blood cells. 4. The infant is able to excrete conjugated bilirubin, but not unconjugated bilirubin. 5. The newborns liver has greater metabolic and enzymatic activity at birth than does an adult liver, increasing the newborns susceptibility to jaundice.

1, 2, 4

Marked changes occur in the cardiopulmonary system at birth include which of the following? Select all that apply. 1. Closure of the foramen ovale 2. Closure of the ductus venosus 3. Mean blood pressure of 31 to 61 mmHg in full-term resting newborns 4. Increased systemic vascular resistance and decreased pulmonary vascular resistance 5. Opening of the ductus arteriosus

1, 2, 4

1) The nurse is caring for a client diagnosed with cystitis. When teaching the client about self-care techniques, which foods or beverages will the nurse advise the client to avoid? Select all that apply. 1. Caffeine 2. Dairy products 3. Alcohol 4. Carbonated beverages 5. Acidic fruit juices

1, 3, 4

1) A patient in her late 40s asks the nurse what she should expect when entering menopause. In which order should the nurse identify changes that the patient will experience during menopause? 1. Amenorrhea 2. Anovulation 3. Reduced fertility 4. Changes in menstrual flow 5. Menstrual cycle irregularities

2, 3, 4, 5, 1

1) The nurse suspects that a newly admitted patient is experiencing manifestations of hepatitis A. What assessment findings did the nurse use to make this clinical determination? Select all that apply. 1. Rash 2. Fever 3. Jaundice 4. Joint pain 5. Gray-colored stool

2, 3, 5

A patient experiencing postpartum hemorrhage is prescribed to receive 741 m L of a crystalloid solution. How many m L of blood did this patient lose from the hemorrhage? (Calculate to the nearest whole number.)

247 mL

At birth a newborn weighed 7 pounds 10 ounces. If the average weight gain is 7 ounces every week for the first 6 months, what weight should the nurse expect when assessing an infant that is 20 weeks old? (Calculate the weight in ounces.)

260 oz

A patient using the calendar rhythm method of birth control asks for assistance to calculate her most fertile period. She states that her shortest cycle is 22 days and her longest cycle is 40 days. Using this information, which day should the nurse identify as being the end of the patient's fertile period?

29

At birth a newborn's head circumference is 13 inches. What should the nurse expect the chest circumference to be in c m? (Round to the nearest whole number.)

31 cm

A neonate weighing 5.5 l b is prescribed to receive intravenous cefotaxime (Claforan) 50 m g/k g/dose every 6 hours for treatment of sepsis. How many m g of the medication should the nurse anticipate the patient will need for 24 hours? (Calculate to the nearest whole number.)

500 mg

1) The nurse is admitting a Hispanic woman scheduled for a cholecystectomy. The nurse uses a cultural assessment tool during the admission. Which question would be most important for the nurse to ask? A) "What other treatments have you used for your abdominal pain?" B) "In what country were you were born?" C) "When you talk to family members, how close do you stand?" D) "How would you describe your role within your family?"

A

1) When assessing a client asking about birth control, the nurse knows that the client would not be a good candidate for Depo-Provera (D M P A) if which of the following is true? A) She wishes to get pregnant within 3 months. B) She is a nursing mother. C) She has a vaginal prolapse. D) She weighs 200 pounds.

A

1) While caring for a client in labor, the nurse notes the following F H R pattern: Which action should the nurse perform? A) Continue to monitor the client B) Fetal scalp stimulation C) Palpate contraction strength D) Discontinue oxytocin

A

1) While caring for a client in labor, the nurse notes the following persistent rhythm on the fetal heart rate monitor: Which action should the nurse take first? A) Notify the provider. B) Prepare for expedient delivery. C) Reposition the client. D) Discontinue oxytocin therapy.

A

10) A client who delivered 2 hours ago tells the nurse that she is exhausted and feels guilty because her friends told her how euphoric they felt after giving birth. How should the nurse respond? 1. Everyone is different, and both responses are normal. 2. Most mothers do feel euphoria; I dont know why you dont. 3. Its good for me to know that because it might indicate a problem. 4. Let me bring your baby to the nursery so that you can rest.

A

15) The community nurse is working with a client from Southeast Asia who has delivered her first child. Her mother has come to live with the family for several months. The nurse understands that the main role of the grandmother while visiting is to do which of the following? 1. Help the new mother by allowing her to focus on resting and caring for the baby. 2. Teach her son-in-law the right way to be a father because this is his first child. 3. Make sure that her daughter does not become abusive towards the infant. 4. Pass on the cultural values and beliefs to the newborn grandchild.

A

16) The home health nurse is visiting a new mother whose baby was delivered by emergency cesarean after a car accident. The mother seems dazed, irritable, and unaware of her surroundings. She tells the nurse she has had trouble sleeping. What would the nurse suspect that the mother has? 1. Post-traumatic stress disorder 2. Postpartum blues 3. Postpartum psychosis 4. Disenfranchised grief

A

19) The postpartum multipara is breastfeeding her new baby. The client states that she developed mastitis with her first child, and asks whether there is something she can do to prevent mastitis this time. What would the best response of the nurse be? 1. Massage your breasts on a daily basis, and if you find a hardened area, massage it towards the nipple. 2. Most first-time moms experience mastitis. It is really quite unusual for a woman having her second baby to get it again. 3. Apply cabbage leaves to any areas that feel thickened or firm to relieve the swelling. 4. Take your temperature once a day. This will help you to pick up the infection early, before it becomes severe.

A

24) Which of the following behaviors noted in the postpartum client would require the nurse to assess further? 1. Responds hesitantly to infant cries. 2. Expresses satisfaction about the sex of the baby. 3. Friends and family visit the client and give advice. 4. Talks to and cuddles with the infant frequently.

A

25) The postpartum client who delivered 2 days ago has developed endometritis. Which entry would the nurse expect to find in this clients chart? 1. Cesarean birth after extended labor with ruptured membranes. 2. Unassisted childbirth and afterbirth. 3. External fetal monitoring used throughout labor. 4. The client has history of pregnancy-induced hypertension.

A

26) The nurse is planning care for three newly delivered adolescents and their babies. What should the nurse keep in mind when planning their care? 1. The babys father should be encouraged to participate when the nurse is providing instruction. 2. A class for all the adolescents would decrease teaching effectiveness. 3. The schools that the adolescents attend will provide teaching on bathing. 4. Adolescents understand the danger signals in newborns.

A

26) Which of the following conditions would predispose a client for thrombophlebitis? 1. Severe anemia 2. Cesarean delivery 3. Anorexia 4. Hypocoagulability

A

28) The client delivered vaginally 2 hours ago after receiving an epidural analgesia. She has a slight tingling sensation in both lower extremities, but normal movement. She sustained a second-degree perineal laceration. Her perineum is edematous and ecchymotic. What should the nurse include in the plan of care for this client? 1. Assist the client to the bathroom in 2 hours to void. 2. Place a Foley catheter now. 3. Apply warm packs to the perineum three times a day. 4. Allow the client to rest for the next 8 hours.

A

3) The nurse would expect a physician to prescribe which medication to a postpartum client with heavy bleeding and a boggy uterus? 1. Methylergonovine maleate (Methergine) 2. Rh immune globulin (RhoGAM) 3. Terbutaline (Brethine) 4. Docusate (Colace)

A

31) The client delivered by cesarean birth 3 days ago and is being discharged. Which statement should the nurse include in the discharge teaching? 1. If your incision becomes increasingly painful, call the doctor. 2. It is normal for the incision to ooze greenish discharge in a few days. 3. Increasing redness around the incision is a part of the healing process. 4. A fever is to be expected because you had a surgical delivery.

A

33) The nurse is performing discharge teaching for a newly delivered first-time mother and her infant on the 2nd postpartum day. Which statement by the mother indicates that teaching has been successful? 1. Taking baths will help my perineum feel less sore each day. 2. If I develop heavy bleeding, I should take my temperature. 3. My bowel movements should resume in a week. 4. I will go back to the doctor in 4 days for my RhoGAM shot.

A

34) The nurse is assessing clients after delivery. For which client is early discharge at 24 hours after delivery appropriate? 1. Woman and baby who have had two successful breastfeedings 2. Woman who is bottle-feeding her infant and has not voided since delivery 3. Twins delivered at 35 weeks, bottle-feeding 4. Cesarean birth performed for fetal distress

A

4) A postpartum client has inflamed hemorrhoids. Which nursing intervention would be appropriate? 1. Encourage sitz baths. 2. Position the client in the supine position. 3. Avoid stool softeners. 4. Decrease fluid intake.

A

4) The client has experienced a postpartum hemorrhage at 6 hours postpartum. After controlling the hemorrhage, the clients partner asks what would cause a hemorrhage. How should the nurse respond? 1. Sometimes the uterus relaxes and excessive bleeding occurs. 2. The blood collected in the vagina and poured out when your partner stood up. 3. Bottle-feeding prevents the uterus from getting enough stimulation to contract. 4. The placenta had embedded in the uterine tissue abnormally.

A

7) A variety of drugs are used either alone or in combination to provide relief of postpartum pain. Which of the following would be an option for pain relief? 1. Nonsteroidal anti-inflammatory agents 2. Proquad 3. Methergine 4. Intravenous oxytocin

A

A new parent reports to the nurse that the baby looks cross-eyed several times a day. The nurse teaches the parents that this finding should resolve in: A. 4 months B. 1 year C. 2 weeks D. 2 months

A

A newborn delivered at term is being discharged. The parents ask the nurse how to keep their baby warm. The nurse knows additional teaching is necessary if a parent states which of the following? 1. "A quick cool bath will help wake up my son for feedings." 2. "I can check my son's temperature under his arm." 3. "My baby should be dressed warmly, with a hat." 4. "Cuddling my son will help to keep him warm."

A

A nurse needs to evaluate the progress of a woman's labor. The nurse obtains the following data: cervical dilatation 6 cm; contractions mild in intensity, occurring every 5 minutes, with a duration of 30-40 seconds. Which clue in this data does not fit the pattern suggested by the rest of the clues? 1. Cervical dilatation 6 cm 2. Mild contraction intensity 3. Contraction frequency every 5 minutes 4. Contraction duration 30-40 seconds

A

A nursing instructor is demonstrating how to perform a heel stick on a newborn. To obtain an accurate capillary hematocrit reading, what does the nursing instructor tell the student do? 1. Rub the heel vigorously with an isopropyl alcohol swab prior to obtaining blood. 2. Use a previous puncture site. 3. Cool the heel prior to obtaining blood. 4. Use a sterile needle and aspirate.

A

The nurse attempts to elicit the Moro reflex on a newborn, and assesses movement of the right arm only. Based on this finding, the nurse immediately assesses: A. The clavicle B. Babinski's reflex C. Ortolani's maneuver D. The Gallant reflex

A

The nurse is conducting several home visits over the course of a week. Which action is appropriate to maintain safety? 1. Provide a daily schedule of visits to supervisors, including client addresses and phone numbers. 2. Maintain distance from threatening pets but do not insist that they be kept out of the room. 3. If an unsafe situation arises, discuss safety concerns with the client before continuing with the visit. 4. Lock personal belongings in the car trunk prior to entering the client's home.

A

The nurse is explaining to a new prenatal client that the certified nurse-midwife will perform clinical pelvimetry as a part of the pelvic exam. The nurse knows that teaching has been successful when the client makes which statement about the reason for the exam? 1. "It will help us know how big a baby I can deliver vaginally." 2. "Doing this exam is a part of prenatal care at this clinic." 3. "My sister had both of her babies by cesarean." 4. "I am pregnant with my first child."

A

The nurse is preparing to assess the fetus of a laboring client. Which assessment should the nurse perform first? 1. Perform Leopold maneuvers to determine fetal position. 2. Count the fetal heart rate between, during, and for 30 seconds following a uterine contraction (UC). 3. Dry the maternal abdomen before using the Doppler. 4. The diaphragm should be cooled before using the Doppler.

A

The nurse is reviewing charts of clients who underwent cesarean births by request in the last two years. The hospital is attempting to decrease costs of maternity care. What findings contribute to increased health care costs in clients undergoing cesarean birth by request? 1. Increased abnormal placenta implantation in subsequent pregnancies 2. Decreased use of general anesthesia with greater use of epidural anesthesia 3. Prolonged anemia, requiring blood transfusions every few months 4. Coordination of career projects of both partners leading to increased income

A

The nurse is seeing prenatal clients in the clinic. Which client is exhibiting expected findings? 1. 12 weeks' gestation, with fetal heart tones heard by Doppler fetoscope 2. 22 weeks' gestation, client reports no fetal movement felt yet 3. 16 weeks' gestation, fundus three finger breadths above umbilicus 4. Marked edema

A

The nurse prepares to admit to the nursery a newborn whose mother had meconium-stained amniotic fluid. The nurse knows this newborn might require which of the following? 1. Initial resuscitation 2. Vigorous stimulation at birth 3. Phototherapy immediately 4. An initial feeding of iron-enriched formula

A

The postpartum homecare nurse has performed home visits to four breastfeeding mothers. Which mother is experiencing an expected outcome? 1. Breasts are engorged; placing fresh cabbage leaves inside her bra 2. Sore and cracked nipples; using hydrogel dressings to facilitate healing 3. Breast engorgement; accompanied by erythema 4. Concerns about milk supply; supplementing with formula

A

The student nurse attempts to take the vital sign of the newborn, but the newborn is crying. What nursing intervention would be appropriate? A. Placing a gloved finger in the newborn's mouth B. Taking the vital signs C. Swaddling the newborn with several warm blankets in an attempt to calm D. Waiting until the newborn stops crying

A

The student nurse notices that a newborn weighs less today compared with the newborns birth weight three days ago. The nursing instructor explains that newborns lose weight following birth due to which of the following? 1. A shift of intracellular water to extracellular spaces. 2. Loss of meconium stool. 3. A shift of extracellular water to intracellular spaces. 4. The sleep-wake cycle.

A

To identify the duration of a contraction, the nurse would do which of the following? 1. Start timing from the beginning of one contraction to the completion of the same contraction. 2. Time between the beginning of one contraction and the beginning of the next contraction. 3. Palpate for the strength of the contraction at its peak. 4. Time from the beginning of the contraction to the peak of the same contraction.

A

What information should the nurse include when teaching a new mother how to successfully bottle-feed her newborn? 1. Proper dilution of powdered formula is essential to provide adequate nutrition. 2. Keep formula at room temperature for at least 4 hours to warm it, instead of microwaving it. 3. Use enough water to dilute the nutrient and calorie density so the infant will drink more formula. 4. Freeze newly prepared formula for up to 3 months.

A

While completing the medical and surgical history during the initial prenatal visit, the 16-year-old primigravida interrupts with "Why are you asking me all these questions? What difference does it make?" Which statement would best answer the client's questions?" 1. "We ask these questions to detect anything that happened in your past that might affect the pregnancy." 2. "We ask these questions to see whether you can have prenatal visits less often than most clients do." 3. "We ask these questions to make sure that our paperwork and records are complete and up to date." 4. "We ask these questions to look for any health problems in the past that might affect your parenting."

A

1) In assessing a family, the community nurse uses a family assessment tool, which provides an organized framework to collect data concerning which of the following? Select all that apply. A) Access to laundry and grocery facilities B) Access to health care C) Sharing of religious beliefs and values D) Acculturation to traditional lifestyles E) Ability to include a new spouse into the family unit

A, B, C

1) In working with immigrants in an inner-city setting, the nurse recognizes that acculturation of immigrants often brings with it which of the following benefits? Select all that apply. A) Improved socioeconomic status B) Use of preventive care services C) Better nutrition D) Increase in substance abuse over time E) More physician visits due to language barriers

A, B, C

1) A client at 10 weeks' gestation is scheduled for a surgical abortion. Which approaches may be used to dilate the cervix for the procedure? Select all that apply. A) Misoprostol B) Mifepristone C) Metal dilators D) Sterile seaweed E) Paracervical block

A, B, C, D

The nurse caring for a client in labor anticipates fetal macrosomia and shoulder dystocia. Appropriate management of shoulder dystocia is essential in order to prevent which fetal complications? Note: Credit will be given only if all correct choices and no incorrect choices are selected. Select all that apply. 1. Brachial plexus injury 2. Fractured clavicle 3. Asphyxia 4. Neurological damage 5. Puerperal infection

A, B, C, D

The nurse caring for a newborn with anemia would expect which initial laboratory data to be included in the initial assessment? Note: Credit will be given only if all correct choices and no incorrect choices are selected. Select all that apply. 1. Hemoglobin 2. Hematocrit 3. Reticulocyte count 4. Direct Coombs' test 5. Cord serum OgM

A, B, C, D

The nurse is caring for a client undergoing fetal heart rate monitoring, and the FHR is greater than 162 beats/min for 12 minutes. For what cause(s) should the nurse anticipate treatment? Note: Credit will be given only if all correct choices and no incorrect choices are selected. Select all that apply. Maternal anxiety Fetal asphyxia Prematurity Fetal anemia Maternal hypotension

A, B, C, D

The nurse is caring for a client who is having fetal tachycardia. The nurse knows that possible causes include which of the following? Note: Credit will be given only if all correct choices and no incorrect choices are selected. Select all that apply. 1. Maternal dehydration 2. Maternal hyperthyroidism 3. Fetal hypoxia 4. Prematurity 5. Anesthesia or regional analgesia

A, B, C, D

The nurse is planning a home visit for a new mother and newborn who were discharged prior to 48 hours after delivery. In preparation for the home visit, what should the nurse do? Note: Credit will be given only if all correct choices and no incorrect choices are selected. Select all that apply. 1. Review the client's records. 2. Gather materials and equipment that might be needed. 3. Make a pre-visit telephone call to determine time and day of visit. 4. Contact the healthcare provider about any special concerns. 5. Schedule additional home visits or follow-up contacts with community agencies.

A, B, C, D

The nurse is reviewing the contents of the birthing unit's emergency pack for use in case of a precipitous birth. Which item(s) should the nurse ensure is (are) included in the pack? Select all that apply. A. Sterile drape B. Bulb syringe C. Two sterile clamps D. Sterile gloves E. Forceps

A, B, C, D

When teaching the new mother about the composition of breast milk, the nurse explains that the fat content can range from 30 to 50 grams/liter. Which factors affect the fat content of breast milk? Note: Credit will be given only if all correct choices and no incorrect choices are selected. Select all that apply. 1. Maternal parity 2. Duration of pregnancy 3. Stage of lactation 4. Time of day 5. Vitamin C intake

A, B, C, D

Which serum markers are assessed when conducting a quadruple screen? Note: Credit will be given only if all correct choices and no incorrect choices are selected. Select all that apply. Alpha-fetoprotein (A F P) Human chorionic gonadotropin (h C G) Unconjugated estriol (U E) Inhibin-A Glycated hemoglobin

A, B, C, D

) In caring for a client with a uterine rupture, the nurse determines which nursing diagnoses to be appropriate? Note: Credit will be given only if all correct choices and no incorrect choices are selected. Select all that apply. 1. Gas Exchange, Impaired 2. Fear related to unknown outcome 3. Coping, Ineffective 4. Mobility: Physical, Impaired 5. Anxiety

A, B, C, E

1) The public health nurse is working with a student nurse. The student nurse asks which of the six groups of people they have seen today are considered to be families. How should the nurse respond? Select all that apply. A) "The married heterosexual couple without children" B) "The gay couple with two adopted children" C) "The unmarried heterosexual couple with two biological children" D) "The lesbian couple not living together that have no children" E) "The married heterosexual couple with three children, living with grandparents"

A, B, C, E

1) Upon assessing the F H R tracing, the nurse determines that there is fetal tachycardia. The fetal tachycardia would be caused by which of the following? Note: Credit will be given only if all correct choices and no incorrect choices are selected. Select all that apply. A) Early fetal hypoxia B) Prolonged fetal stimulation C) Fetal anemia D) Fetal sleep cycle E) Infection

A, B, C, E

13) The nurse is teaching a new mother about ways to manage fatigue after she returns home. Which instructions should the nurse include? Select all that apply. 1. Take frequent rest periods. 2. Nap when the newborn is sleeping. 3. Avoid overdoing housework and unnecessary chores. Do not clean when infant is sleeping. 4. Avoid having others come to the house to do housework and interfere with rest. 5. Utilize friends and family to provide help and support, such as cooking a meal.

A, B, C, E

28) The nurse is teaching a prenatal class about postpartum changes. The nurse explains that factors that might interfere with uterine involution include which of the following? Select all that apply. 1. Prolonged labor 2. Difficult birth 3. Full bladder 4. Breastfeeding 5. Infection

A, B, C, E

34) At her 6-week postpartum checkup, a new mother voices concerns to the nurse. She states that she is finding it hard to have time alone to even talk on the phone without interruption. Her family lives in another state, and she has contact with them only by phone. She is still having difficulty getting enough sleep and worries that she will not be a good mother. Appropriate nursing interventions would include providing which of the following? Select all that apply. 1. Anticipatory guidance about the realities of being a parent. 2. Parenting literature and reference manuals. 3. Phone numbers and locations of local parenting groups. 4. Referral for specialized interventions related to postpartum blues. 5. Phone numbers and names of postpartum doulas.

A, B, C, E

34) Which interventions can the nurse utilize to provide continuity of care for the postpartal client who experienced a complication and is now ready to return home? Select all that apply. 1. Encourage the client to take advantage of home visits. 2. Make telephone calls as a follow-up to check on the client and newborn. 3. Provide information about postpartal support groups. 4. Refer to mental health professionals to help screen the client for any mental health problems as a result of the complications experienced in the hospital. 5. Supply information about postpartum expectations designed to meet the specific needs of a variety of families.

A, B, C, E

A prenatal client asks the nurse about conditions that would necessitate a cesarean delivery. The nurse explains that cesarean delivery generally is performed in the presence of which of the following? Note: Credit will be given only if all correct choices and no incorrect choices are selected. Select all that apply. 1. Complete placenta previa 2. Placental abruption 3. Umbilical cord prolapse 4. Precipitous labor 5. Failure to progress

A, B, C, E

The nurse is teaching a class on reading a fetal monitor to nursing students. The nurse explains that bradycardia is a fetal heart rate baseline below 110 and can be caused by which of the following? Note: Credit will be given only if all correct choices and no incorrect choices are selected. Select all that apply. 1. Maternal hypotension 2. Prolonged umbilical cord compression 3. Fetal dysrhythmia 4. Central nervous system malformation 5. Late fetal asphyxia

A, B, C, E

16) Which strategies would the nurse utilize to promote culturally competent care for the postpartum client? Select all that apply. 1. Examine ones own cultural beliefs, biases, stereotypes, and prejudices. 2. Respect the values and beliefs of others. 3. Limit the alternative food choices offered clients to minimize conflicts. 4. Incorporate the familys cultural practices into the care. 5. Evaluate whether the familys cultural practices fit into Western norms.

A, B, D

17) What maternity unit policies promote postpartal family wellness and shared parenting? Select all that apply. 1. Mother-baby care or couplet care on the postpartum unit 2. Skin-to-skin contact between the mother and baby and the father and baby 3. Newborn kept in the nursery to allow mother to rest between feedings 4. On-demand feeding schedule for both breastfed and bottle-fed infants 5. Limited visiting hours for the father so that the mother can sleep as needed

A, B, D

The client with polycystic ovarian syndrome (P C O S) has been prescribed metformin (Glucophage). The nurse tells the client that the medication will do which of the following? Note: Credit will be given only if all correct choices and no incorrect choices are selected. Select all that apply. "Decrease your excessive hair growth." "Make it easier to lose weight." "Increase your acne." "Improve your chances of pregnancy." "Make your menstrual periods irregular."

A, B, D

The nurse is caring for a client in labor. Which signs and symptoms would indicate the client is progressing into the second stage of labor? Note: Credit will be given only for all correct choices and no incorrect choices. Select all that apply. 1. Bulging perineum 2. Increased bloody show 3. Spontaneous rupture of the membranes 4. Uncontrollable urge to push 5. Inability to breathe through contractions

A, B, D

The nurse tells a mother that the doctor is preparing to circumcise her newborn. The mother expresses concern that the infant will be uncomfortable during the procedure. The nurse explains that the physician will numb the area before the procedure. Additional methods of comfort often used during the procedure include which of the following? Note: Credit will be given only if all correct choices and no incorrect choices are selected. Select all that apply. 1. Providing a pacifier 2. Stroking the head 3. Restraining both arms and legs 4. Talking to the infant 5. Giving the infant a sedative before the procedure

A, B, D

1) A 30-year-old patient who experiences severe premenstrual syndrome every month asks for nonpharmacologic suggestions to treat this disorder. What should the nurse recommend? Select all that apply. A) Eat more frequent meals B) Engage in aerobic activity C) Limit alcohol to two drinks per day D) Restrict the intake of chocolate and coffee E) Increase the intake of fruits and vegetables

A, B, D, E

1) The nurse is preparing an educational session for high school female students on self-care during menstruation. What should the nurse include regarding care when using a tampon? Select all that apply. A) Wash hands before inserting a tampon B) Wash hands after inserting the tampon C) Change the tampon every 8 to 12 hours D) Use tampons with the minimum amount of absorbency E) Avoid touching the part that will be inserted into the vagina

A, B, D, E

A mother and her newborn have been discharged after a hospital stay of less than 48 hours. What are essential components the nurse must include in the first postpartum home visit? Note: Credit will be given only if all correct choices and no incorrect choices are selected. Select all that apply. 1. Assessment of color 2. Measurement of weight 3. Measurement of height 4. Assessment of mother-newborn interaction 5. Reinforcement of information about feeding and sleep patterns

A, B, D, E

A pregnant client calls the clinic nurse to say she is worried about symptoms she is experiencing. The nurse advises the client to come immediately to the clinic because of which reported symptoms? Note: Credit will be given only if all correct and no incorrect choices are selected. Select all that apply. 1. Vaginal bleeding 2. Abdominal pain 3. Constipation 4. Epigastric pain 5. Blurring of vision

A, B, D, E

The nurse is caring for a newborn with jaundice. The parents question why the newborn is not under phototherapy lights. The nurse explains that the fiber-optic blanket is beneficial because of which of the following? Note: Credit will be given only if all correct choices and no incorrect choices are selected. Select all that apply. 1. Lights can stay on all the time. 2. The eyes do not need to be covered. 3. The lights will need to be removed for feedings. 4. Newborns do not get overheated. 5. Weight loss is not a complication of this system.

A, B, D, E

1) In learning about Duvall's life-cycle stages ascribed to traditional families, the nursing student recognizes that developmental tasks of each stage include which of the following? Select all that apply. A) Adjusting to new roles as mother and father B) Working out authority and socialization roles with the school C) Becoming a single parent with custodial responsibilities D) Becoming a couple and dating E) Adjusting to the loss of a spouse

A, B, E

A new mother is questioning the nurse about the advantages of breastfeeding her newborn. Which information should the nurse include in the teaching session? Note: Credit will be given only if all correct choices and no incorrect choices are selected. Select all that apply. 1. Breast milk has immunological advantages, including varying degrees of protection from bacterial and viral infections. 2. Breastfeeding has been shown to increase maternal-infant attachment. 3. Breastfeeding can be initially supplemented with bottle feedings so that the father does not feel left out of the infant's care. 4. Breastfeeding often causes nipple tenderness, and may be discouraged until healing occurs. 5. Breastfeeding provides a psychologic advantage to the mother, who derives satisfaction knowing that she is providing her infant with the optimal nutritional start in life.

A, B, E

The client presents to the labor and delivery unit stating that her water broke 2 hours ago. Indicators of normal labor include which of the following? Select all that apply. 1. Fetal heart rate of 130 with average variability 2. Blood pressure of 130/80 3. Maternal pulse of 160 4. Protein of +1 in urine 5. Odorless, clear fluid on underwear

A, B, E

The nurse is administering erythromycin (Ilotycin) ointment to a newborn. What factors are associated with administration of this medication? Note: Credit will be given only if all correct choices and no incorrect choices are selected. Select all that apply. 1. The medication should be instilled in the lower conjunctival sac of each eye. 2. The eyelids should be massaged gently to distribute the ointment. 3. The medication must be given immediately after delivery. 4. The medication does not cause any discomfort to the infant. 5. The medication can interfere with the baby's ability to focus.

A, B, E

The nurse is teaching the pregnant client about the symptoms of preeclampsia. Which clinical manifestations will the nurse include in the teaching session? Note: Credit will be given only if all correct choices and no incorrect choices are selected. Select all that apply. Dizziness Blurred vision Abdominal pain Vaginal bleeding Severe headache

A, B, E

Which of the following are potential disadvantages to breastfeeding? Note: Credit will be given only if all correct choices and no incorrect choices are selected. Select all that apply. 1. Pain with breastfeeding 2. Leaking milk 3. Equal feeding responsibilities with fathers 4. Vaginal wetness 5. Embarrassment

A, B, E

Which statements by a breastfeeding class participant indicate that teaching by the nurse was effective? Note: Credit will be given only if all correct choices and no incorrect choices are selected. Select all that apply. 1. "Breastfed infants get more skin-to-skin contact and sleep better." 2. "Breastfeeding raises the level of a hormone that makes me feel good." 3. "Breastfeeding is complex and difficult, and I probably won't succeed." 4. "Breastfeeding is worthwhile, even if it costs more overall." 5. "Breastfed infants have fewer digestive and respiratory illnesses."

A, B, E

1) Before performing Leopold maneuvers, what would the nurse do? Note: Credit will be given only if all correct choices and no incorrect choices are selected. Select all that apply. A) Have the client empty her bladder. B) Place the client in Trendelenburg position. C) Have the client lie on her back with her feet on the bed and knees bent. D) Turn the client to her left side. E) This is not the optimal position for the client when performing Leopold maneuvers.

A, C

3) A client is experiencing excessive bleeding immediately after the birth of her newborn. After speeding up the IV fluids containing oxytocin, with no noticeable decrease in the bleeding, the nurse should anticipate the physician requesting which medications? Select all that apply. 1. Methergine 2. Coumadin 3. Misoprostol 4. Serotonin reuptake inhibitors (SSRIs) 5. Nonsteroidal anti-inflammatory drugs

A, C

A pregnant client at 30 weeks' gestation has had a steady rise in blood pressure. She is now 20 mmHg above her systolic baseline. The nurse advises her to immediately report which symptoms? Note: Credit will be given only if all correct and no incorrect choices are selected. Select all that apply. 1. Dizziness 2. Even a small amount of dependent edema 3. Spots before her eyes 4. Persistent nausea and vomiting 5. Vaginal spotting

A, C

Before performing Leopold maneuvers, what would the nurse do? Note: Credit will be given only if all correct choices and no incorrect choices are selected. Select all that apply. 1. Have the client empty her bladder. 2. Place the client in Trendelenburg position. 3. Have the client lie on her back with her feet on the bed and knees bent. 4. Turn the client to her left side. 5. Have the client lie flat with her ankles crossed.

A, C

The nurse is at the home of a postpartum client for an initial assessment. The client gave birth by cesarean section 1 week earlier. Which statements should the nurse include? Note: Credit will be given only if all correct choices and no incorrect choices are selected. Select all that apply. 1. "Because you had a cesarean, I'd like to assess your incision." 2. "You aren't having any problems nursing, right?" 3. "How rested do you feel since you came home?" 4. "Because you are bottle-feeding, I won't assess your breasts." 5. "You should remain at home for the first 3 weeks after delivery."

A, C

What interventions would the nurse apply to support the breastfeeding mother? Note: Credit will be given only if all correct choices and no incorrect choices are selected. Select all that apply. 1. Assist the mother to begin breastfeeding within the first hour after birth. 2. Have the baby returned to the nursery after feeding so that the mother can get adequate rest. 3. Teach the mother to recognize and respond to early infant feeding cues. 4. Inform the mother about community resources that support breastfeeding. 5. Instruct the mother to avoid eating foods that might upset the newborn's stomach

A, C, D

6) The incidence of complications and discomforts in the first year postpartum is common and women may experience which of the following? Select all that apply. 1. Pain 2. Excess energy 3. Urinary incontinence 4. Changes in mental health status 5. Sleep deprivation

A, C, D, E

9) The nurse is caring for a client who had a cesarean birth 4 hours ago. Which interventions would the nurse implement at this time? Select all that apply. 1. Administer analgesics as needed. 2. Encourage the client to ambulate to the bathroom to void. 3. Encourage leg exercises every 2 hours. 4. Encourage the client to cough and deep-breathe every 2 to 4 hours. 5. Encourage the use of breathing, relaxation, and distraction.

A, C, D, E

A new mother inquires about postpartum resources. What resources can the nurse suggest to provide assistance to the new mother? Note: Credit will be given only if all correct choices and no incorrect choices are selected. Select all that apply. 1. Breastfeeding support groups 2. Meals on Wheels 3. Lactation consultants 4. Postpartum classes 5. Internet sites

A, C, D, E

A nurse is performing a postpartum assessment during the first home visit to a client 3 days postdelivery. Which actions will the nurse perform? Note: Credit will be given only if all correct choices and no incorrect choices are selected. Select all that apply. 1. Palpate the breasts. 2. Auscultate the carotid. 3. Check vaginal discharge. 4. Assess the extremities. 5. Inspect the perineum.

A, C, D, E

Antibiotics have been ordered for a newborn with an infection. Which interventions would the nurse prepare to implement? Note: Credit will be given only if all correct choices and no incorrect choices are selected. Select all that apply. 1. Obtain skin cultures. 2. Restrict parental visits. 3. Evaluate bilirubin levels. 4. Administer oxygen as ordered. 5. Observe for signs of hypoglycemia.

A, C, D, E

The nurse is preparing to give an injection of vitamin K to a newborn. Which considerations would be appropriate? Note: Credit will be given only if all correct choices and no incorrect choices are selected. Select all that apply. 1. Administer a dose of 0.5 to 1 mg within 1 hour of birth. 2. Administer the injection subcutaneously. 3. Use a 25-gauge, 5/8-inch needle for the injection. 4. Protect the medication bottle from light. 5. Give vitamin K prior to a circumcision procedure.

A, C, D, E

The nurse is providing discharge teaching to the parents of a newborn. The nurse should instruct the parents to notify the healthcare provider in case of which of the following? Note: Credit will be given only if all correct choices and no incorrect choices are selected. Select all that apply. 1. More than one episode of forceful vomiting. 2. More than 6 to 10 wet diapers per day. 3. A bluish discoloration of the skin with or without a feeding. 4. Refusal of two feedings in a row. 5. Development of eye drainage.

A, C, D, E

The nurse is reviewing data collected during a health history and physical assessment and suspects that the patient could be experiencing polycystic ovarian syndrome (P C O S). What information did the nurse use to make this clinical determination? Select all that apply. Body mass index 31 Hair loss and warm moist skin Periods occur every 3 to 4 months Fasting capillary blood glucose 123 m g/d L Inability to become pregnant after 2 years of unprotected intercourse

A, C, D, E

The nurse should inform the parents of a newborn that they should call their healthcare provider when which of the following occurs? Note: Credit will be given only if all correct choices and no incorrect choices are selected. Select all that apply. 1. Continual rise in temperature 2. Decreased frequency of stools 3. Absence of breathing longer than 20 seconds 4. Lethargy 5. Refusal of two feedings in a row

A, C, D, E

Usually, the family is advised to arrive at the birth setting at the beginning of the active phase of labor or when which of the following occur? Select all that apply. 1. Rupture of membranes (ROM) 2. Increased fetal movement 3. Decreased fetal movement 4. Any vaginal bleeding 5. Regular, frequent uterine contractions (UCs)

A, C, D, E

When planning care for the premature newborn diagnosed with respiratory distress syndrome, which potential complications would the nurse anticipate? Note: Credit will be given only if all correct choices and no incorrect choices are selected. Select all that apply. 1. Hypoxia 2. Respiratory alkalosis 3. Metabolic acidosis 4. Massive atelectasis 5. Pulmonary edema

A, C, D, E

Which fetal/neonatal risk factors would lead the nurse to anticipate a potential need to resuscitate a newborn? Note: Credit will be given only if all correct choices and no incorrect choices are selected. Select all that apply. 1. Nonreassuring fetal heart rate pattern/sustained bradycardia 2. Fetal scalp/capillary blood sample pH greater than 7.25 3. History of meconium in amniotic fluid 4. Prematurity 5. Significant intrapartum bleeding

A, C, D, E

35) Clinical features of posttraumatic stress disorder (PTSD) include which of the following? 1. Difficulty sleeping 2. Acute awareness 3. Flashbacks 4. The need to be constantly around others 5. Irritability

A, C, E

A first-time 22-year-old single labor client, accompanied by her boyfriend, is admitted to the labor unit with ruptured membranes and mild to moderate contractions. She is determined to be 2 centimeters dilated. Which nursing diagnoses might apply during the current stage of labor? Select all that apply. 1. Fear/Anxiety related to discomfort of labor and unknown labor outcome 2. Knowledge, Deficient, related to lack of information about pushing methods 3. Pain, Acute, related to uterine contractions, cervical dilatation, and fetal descent 4. Pain, Acute, related to perineal trauma 5. Coping: Family, Compromised, related to labor process

A, C, E

A patient who is hemorrhaging after a vaginal delivery is being considered for a uterine tamponade. What should the nurse instruct the patient about this process? Note: Credit will be given only if all correct choices and no incorrect choices are selected. Select all that apply. 1. A balloon is inserted into the uterus 2. The balloon is kept in place for 12 hours 3. The balloon is inflated with 300 to 500 m L of saline 4. After removal, the uterus is packed with sterile gauze 5. The tube has an open tip to permit bleeding to be visualized

A, C, E

The newborn's cry should have which of the following characteristics? Note: Credit will be given only if all correct choices and no incorrect choices are selected. Select all that apply. Medium pitch Shrillness Strength High pitch Lusty

A, C, E

The nurse is serving on a panel to evaluate the hospital staff's reliance on evidence-based practice in their decision-making processes. Which practices characterize the basic competencies related to evidence-based practice? Select all that apply. A) Recognizing which clinical practices are supported by good evidence B) Recognizing and including clinical practice supported by intuitive evidence C) Using data in clinical work to evaluate outcomes of care D) Including quality-improvement measures in clinical practice E) Appraising and integrating scientific bases into practice

A, C, E

1) A female patient asks what can be done to control vaginal odor. How should the nurse respond? Select all that apply. A) Wear cotton underwear B) Use a mild vaginal deodorant C) Schedule douching to occur weekly D) Cleanse from front to back when toileting E) Use soap and water to cleanse the perineum

A, D, E

2) The nurse is caring for a postpartum client who is experiencing afterpains following the birth of her third child. Which comfort measure should the nurse implement to decrease her pain? Select all that apply. 1. Offer a warm water bottle for her abdomen. 2. Call the physician to report this finding. 3. Inform her that this is not normal, and she will need an oxytocic agent. 4. Administer a mild analgesic to help with breastfeeding. 5. Administer a mild analgesic at bedtime to ensure rest.

A, D, E

39) Which factors would the nurse observe that would indicate a new mothers early attachment to the newborn? Select all that apply. 1. Face-to-face contact and eye contact 2. Failure to choose a name for the baby 3. Decreased interest in the infants cues 4. Pointing out familial traits of the newborn 5. Displaying satisfaction with the infants sex

A, D, E

5) The nurse is assessing a client who has been diagnosed with an early postpartum hemorrhage. Which findings would the nurse expect? Select all that apply. 1. A boggy fundus that does not respond to massage 2. Small clots and a moderate amount of lochia rubra on the pad 3. Decreased pulse and increased blood pressure 4. Hematoma formation or bulging/shiny skin in the perineal area 5. Rise in the level of the fundus of the uterus

A, D, E

A client who is having false labor most likely would have which of the following? Note: Credit will be given only for all correct choices and no incorrect choices. Select all that apply. 1. Contractions that do not intensify while walking 2. An increase in the intensity and frequency of contractions 3. Progressive cervical effacement and dilatation 4. Pain in the abdomen that does not radiate 5. Contractions that lessen with rest and warm tub baths

A, D, E

The nurse is preparing to provide a newborn with an injection of vitamin K. In which order should the nurse complete the following steps? Cleanse skin with alcohol and allow to dry Aspirate and then inject the medication slowly Insert a 25-gauge 5/8 inch needle at a 90 degree angle Remove the needle and massage with an alcohol swab Bunch skin over mid-anterior lateral aspect of the thigh

A, E, C, B, D

1) A 7-year-old client tells the nurse that "Grandpa, Mommy, Daddy, and my brother live at my house." The nurse identifies this as what type of family? A) Binuclear B) Extended C) Gay or lesbian D) Traditional

B

1) A client comes to the clinic complaining of severe menstrual cramps. She has never been pregnant, has been diagnosed with ovarian cysts, and has had an intrauterine device (I U D) for 2 years. What is the most likely cause for the client's complaint? A) Primary dysmenorrhea B) Secondary dysmenorrhea C) Menorrhagia D) Hypermenorrhea

B

1) A client who has been using transdermal hormonal contraception comes in for a routine wellness visit. Which finding should cause the nurse to question if the client should continue to use this form of contraception? A) Body weight of 179 lb B) Skin breakdown at the site of the patch C) Drinks 2 cups of caffeinated coffee a day D) Bicycles at the gym three evenings a week

B

1) A female client comes into the clinic for a pregnancy test because she took the morning after pill immediately after having unprotected intercourse 3 days ago and has not had a menstrual period. What should the nurse respond to this client? A) "I'll make sure you have one during this visit." B) "You should wait for two weeks before having a pregnancy test." C) "It's unlikely that you are pregnant. Wait a few days and then take a pregnancy test." D) "How long did you wait to take the morning after pill after having unprotected intercourse?"

B

1) A menopausal woman tells her nurse that she experiences discomfort from vaginal dryness during sexual intercourse, and asks, "What should I use as a lubricant?" The nurse should recommend which of the following? A) Petroleum jelly B) A water-soluble lubricant C) Body cream or body lotion D) Less-frequent intercourse

B

1) A nurse is performing an assessment on a family with a father and mother who both work. What type of family does she record this family as being? A) A traditional nuclear family B) A dual-career/dual-earner family C) An extended family D) An extended kin family

B

1) A nurse is working in a clinic where clients from several cultures are seen. As a first step toward the goal of personal cultural competence, the nurse will do which of the following? A) Enhance cultural skills. B) Gain cultural awareness. C) Seek cultural encounters. D) Acquire cultural knowledge.

B

1) A nursing student investigating potential career goals is strongly considering becoming a nurse practitioner (N P). The major focus of the N P is on which of the following? A) Leadership B) Physical and psychosocial clinical assessment C) Independent care of the high-risk pregnant client D) Tertiary prevention

B

1) A woman in labor asks the nurse to explain the electronic fetal heart rate monitor strip. The fetal heart rate baseline is 150 with accelerations to 165, variable decelerations to 140, and moderate long-term variability. Which statement indicates that the client understands the nurse's teaching? A) "The most important part of fetal heart monitoring is the absence of variable decelerations." B) "The most important part of fetal heart monitoring is the presence of variability." C) "The most important part of fetal heart monitoring is the fetal heart rate baseline." D) "The most important part of fetal heart monitoring is the depth of decelerations."

B

1) After reviewing approaches for contraception with a female client, the nurse is concerned that barrier methods will not achieve the client's goal to prevent pregnancy. What did the client say to cause the nurse to draw this conclusion? A) "My partner doesn't mind wearing condoms." B) "I don't want to have to put anything in myself." C) "We should use a condom even with a diaphragm." D) "I know that spermicides are inserted before intercourse."

B

1) Extended use of combined oral contraceptives (C O Cs) reduces the side effects of C O Cs such as which of the following? A) Cramping B) Hypertension C) Breast tenderness D) Bloating

B

1) The client is in the second stage of labor. The fetal heart rate baseline is 170, with minimal variability present. The nurse performs fetal scalp stimulation. The client's partner asks why the nurse did that. What is the best response by the nurse? A) "I stimulated the top of the fetus's head to wake him up a little." B) "I stimulated the top of the fetus's head to try to get his heart rate to accelerate." C) "I stimulated the top of the fetus's head to calm the fetus down before birth." D) "I stimulated the top of the fetus's head to find out whether he is in distress."

B

1) The clinic nurse is returning phone calls. Which call should the nurse return first? A) The call from a 22-year-old reporting that she has menstrual cramps and vomiting every month B) The call from a 17-year-old asking whether there is a problem with using one tampon for a whole day C) The call from a 46-year-old mother of a teen wondering if her daughter should be on birth control D) The call from a 34-year-old requesting information on douching after intercourse

B

1) The nurse is admitting a client to the birthing unit. What question should the nurse ask to gain a better understanding of the client's psychosocial status? A) "How did you decide to have your baby at this hospital?" B) "Who will be your labor support person?" C) "Have you chosen names for your baby yet?" D) "What feeding method will you use for your baby?"

B

1) The nurse is caring for a client hospitalized for pelvic inflammatory disease. Which nursing intervention would have priority? A) Encourage oral fluids B) Administer cefotetan Ⅳ C) Enforce bed rest D) Remove I U C, if present

B

1) The nurse is planning teaching for a patient diagnosed with hepatitis A. What should the nurse emphasize when instructing the patient about this disease process? A) It is a chronic illness B) It is not a chronic illness C) A vaccination is not available D) It occurs in East and South Asia

B

1) The nurse is providing follow-up education to a client just diagnosed with vaginal herpes. What statement by the client verifies correct knowledge about vaginal herpes? A) "I should douche daily to prevent infection." B) "I could have another breakout during my period." C) "I am more likely to develop cancer of the cervix." D) "I should use sodium bicarbonate on the lesions to relieve discomfort."

B

1) The nurse is reviewing laboratory testing completed for a patient with suspected pelvic inflammatory disease. Which test result should the nurse identify as supporting this diagnosis? A) Decreased hematocrit level B) Elevated sedimentation rate C) Decreased hemoglobin level D) Elevated white blood cell count

B

1) The nurse is seeing clients in the women's clinic. Which client should be treated with ceftriaxone I M and doxycycline orally? A) A pregnant client with gonorrhea and a yeast infection B) A nonpregnant client with gonorrhea and chlamydia C) A pregnant client with syphilis D) A nonpregnant client with chlamydia and trichomoniasis

B

1) The nurse is teaching a community education class on complementary and alternative therapies. To assess learning, the nurse asks, "In traditional Chinese medicine, what is the invisible flow of energy in the body that maintains health and ensures physiologic functioning?" Which answer indicates that teaching was successful? A) Meridians B) Chi C) Yin D) Yang

B

1) The nurse is working with a client whose religious beliefs differ from those of the general population. What is the best nursing intervention to use to meet the specific spiritual needs of this family? A) Ask how important the client's religious and spiritual beliefs are when making decisions about health care. B) Show respect while allowing time and privacy for religious rituals. C) Ask for the client's opinion on what caused the illness. D) Identify healthcare practices forbidden by religious or spiritual beliefs.

B

1) The nurse manager in a hospital with a large immigrant population is planning an in-service. Aware of how ethnocentrism affects nursing care, the nurse manager asks, "The belief that one's own values and beliefs are the only or the best values has which of the following results?" A) It implies newcomers to the United States should adopt the norms and values of the country. B) It can create barriers to communication through misunderstanding. C) It leads to an expectation that all clients will exhibit pain the same way. D) It improves the quality of care provided to culturally diverse client bases.

B

1) The nurse who is taking a sexual history from a client should do which of the following? A) Ask questions that the client can answer with "yes" or "no." B) Ask mostly open-ended questions. C) Have the client fill out a comprehensive questionnaire and review it after the client leaves. D) Try not to make much direct eye contact.

B

1) Which of the following best describes a nuclear family? A) An unmarried woman who chooses to conceive or adopt without a life partner. B) Children live in a household with both biologic parents and no other relatives or persons. C) A couple shares household and childrearing responsibilities with parents, siblings, or other relatives. D) The head of the household is widowed, divorced, abandoned, separated, or most often, the mother remains unmarried.

B

10) Every time the nurse enters the room of a postpartum client who gave birth 3 hours ago, the client asks something else about her birth experience. What action should the nurse take? 1. Answer questions quickly and try to divert her attention to other subjects. 2. Review the documentation of the birth experience and discuss it with her. 3. Contact the physician to warn him the client might want to file a lawsuit, based on her preoccupation with the birth experience. 4. Submit a referral to Social Services because of possible obsessive behavior.

B

11) Which of the following would be considered a clinical sign of hemorrhage? 1. Increased blood pressure 2. Increasing pulse 3. Increased urinary output 4. Hunger

B

12) The postpartum client expresses concern about getting back to her prepregnant shape, and asks the nurse when she will be able to run again. Which statement by the client indicates that teaching was effective? 1. I can start running in 2 weeks, and can breastfeed as soon as I am done. 2. I should see how my energy level is at home, and increase my activity slowly. 3. Running is not recommended for breastfeeding women. 4. If I am getting 8 hours of sleep per day, I can start running.

B

13) A postpartum woman is at increased risk for developing urinary tract problems because of which of the following? 1. Decreased bladder capacity 2. Inhibited neural control of the bladder following the use of anesthetic agents 3. Increased bladder sensitivity 4. Abnormal postpartum diuresis

B

13) The nursing instructor is conducting a class about attachment behaviors. Which statement by a student indicates the need for further instruction? 1. The en face position promotes bonding and attachment. 2. Ideally, initial skin-to-skin contact occurs after the baby has been assessed and bathed. 3. In reciprocity, the interaction of mother and infant is mutually satisfying and synchronous. 4. The needs of the mother and of her infant are balanced during the phase of mutual regulation.

B

15) The nurse is calling clients at 4 weeks postpartum. Which of the following clients should be seen immediately? 1. The client who describes feeling sad all the time 2. The client who reports hearing voices talking about the baby 3. The client who states she has no appetite and wants to sleep all day 4. The client who says she needs a refill on her sertraline (Zoloft) next week

B

15) Which statement by a nursing student preparing to care for a postpartum lesbian mother would indicate the need for further teaching? 1. I cant let the client know Ive never worked with lesbian mothers. 2. I will have to adjust some of my discharge instruction for this mother. 3. I dont need to include the partner when I provide care and instruction. 4. Discharge teaching is exactly the same for lesbian mothers as for all others.

B

16) A new grandmother comments that when her children were born, they stayed in the nursery. The grandmother asks the nurse why her daughters baby stays mostly in the room instead of the nursery. How should the nurse respond? 1. Babies like to be with their mothers more than they like to be in the nursery. 2. Contact between parents and babies increases attachment. 3. Budget cuts have decreased the number of nurses in the nursery. 4. Why do you ask? Do you have concerns about your daughters parenting?

B

17) To prevent the spread of infection, the nurse teaches the postpartum client to do which of the following? 1. Address pain early 2. Change peri-pads frequently 3. Avoid overhydration 4. Report symptoms of uterine cramping

B

2) The nurse is assisting a multiparous woman to the bathroom for the first time since her delivery 3 hours ago. When the client stands up, blood runs down her legs and pools on the floor. The client turns pale and feels weak. What would be the first action of the nurse? 1. Assist the client to empty her bladder 2. Help the client back to bed to check the fundus 3. Assess her blood pressure and pulse 4. Begin an IV of lactated Ringers solution

B

20) The client having her second child is scheduled for a cesarean birth because the baby is in a breech presentation. The client states, Im wondering what will be different this time compared with my first birth, which was vaginal. What response is best? 1. Well take good care of you and your baby. Youll be home before you know it. 2. Youll be wearing a sequential compression device until you start walking. 3. You will have a lot of pain, but there are medications that we give when it gets really bad. 4. You wont be able to nurse until the baby is 12 hours old, because of your epidural.

B

22) A multiparous client delivered her first child vaginally 2 years ago, and delivered an infant by cesarean yesterday due to breech presentation. Which statement would the nurse expect the client to make? 1. I cant believe how much more tired I was with the first baby. 2. Im having significantly more pain this time than with my last birth. 3. It is disappointing that I cant breastfeed because of the cesarean. 4. Getting in and out of bed feels more comfortable than last time.

B

27) The community nurse is meeting a new mother for the first time. The client delivered her first child 5 days ago after a 12-hour labor. Neither the mother nor the infant had any complications during the birth or postpartum period. Which statement by the client would indicate to the nurse that the client is experiencing postpartum blues? 1. I am so happy and blessed to have my new baby. 2. One minute Im laughing and the next Im crying. 3. My husband is helping out by changing the baby at night. 4. Breastfeeding is going quite well now that the engorgement is gone.

B

4) A client is preparing to take a sitz bath for the first time. What will the nurse do? 1. Allow the client privacy during the sitz bath. 2. Place a call bell well within reach and check on the client frequently. 3. Discourage the client from taking a sitz bath. 4. Check on the client after the sitz bath.

B

5) A postpartum client has just received a rubella vaccination. The client demonstrates understanding of the teaching associated with administration of this vaccine when she states which of the following? 1. I will need another vaccination in 3 months. 2. I must avoid getting pregnant for 1 month. 3. This will prevent me from getting chickenpox. 4. This will protect my newborn from getting the measles.

B

6) The nurse has received the end-of-shift report on the postpartum unit. Which client should the nurse see first? 1. Woman who is 2nd day post-cesarean, moderate lochia serosa 2. Woman day of delivery, fundus firm 2 cm above umbilicus 3. Woman who had a cesarean section, 1st postpartum day, 4 cm diastasis recti abdominis 4. Woman who had a cesarean section, 1st postpartum day, hypoactive bowel sounds all quadrants

B

7) The nurse expects an initial weight loss for the average postpartum client to be which of the following? 1. 5 to 8 pounds 2. 10 to 12 pounds 3. 12 to 15 pounds 4. 15 to 20 pounds

B

A 25-year-old primigravida is at 20 weeks' gestation. The nurse takes her vital signs and notifies the healthcare provider immediately because of which finding? 1. Pulse 88/minute 2. Rhonchi in both bases 3. Temperature 37.4° C (99.3° F) 4. Blood pressure 122/78

B

A client calls the labor and delivery unit and tells the nurse that she is 39 weeks pregnant and that over the last 4 or 5 days, she has noticed that although her breathing has become easier, she is having leg cramps, a slight amount of edema in her lower legs, and an increased amount of vaginal secretions. The nurse tells the client that she has experienced which of the following? 1. Engagement 2. Lightening 3. Molding 4. Braxton Hicks contractions

B

A client in labor is found to have meconium-stained amniotic fluid upon rupture of membranes. At delivery, the nurse finds the infant to have depressed respirations and a heart rate of 80. What does the nurse anticipate? 1. Delivery of the neonate on its side with head up, to facilitate drainage of secretions. 2. Direct tracheal suctioning by specially trained personnel. 3. Preparation for the immediate use of positive pressure to expand the lungs. 4. Suctioning of the oropharynx when the newborn's head is delivered.

B

A client is admitted to the labor unit with contractions 1-2 minutes apart lasting 60-90 seconds. The client is apprehensive and irritable. This client is most likely in what phase of labor? 1. Active 2. Transition 3. Latent 4. Second

B

A client is concerned about her risk for breast cancer. Following the initial history, the nurse identifies which of the following as a high risk factor for breast cancer? History of late menarche and early menopause Sister who has had breast cancer Mother with fibrocystic breast disease Multiparity

B

A couple who came to the United States two years ago with their two children are seeing the nurse in the community clinic. The nurse knows their family is acculturating when the mother makes which statement? A) "The children are much less well-behaved than they used to be." B) "Our diet now includes hamburgers and French fries." C) "We celebrate the same holidays that we used to at home." D) "When the children leave the house, I worry about them."

B

A maternity client is in need of surgery. Which healthcare member is legally responsible for obtaining informed consent for an invasive procedure? A) The nurse B) The physician C) The unit secretary D) The social worker

B

A new mother is concerned about spoiling her newborn. The home care nurse teaches the mother which of the following? 1. Newborns can be manipulative, so caution is advised. 2. Meeting the infant's needs develops a trusting relationship. 3. An infant who is rocked to sleep every night is being spoiled. 4. Crying is good for babies, and letting them cry it out is advised.

B

A new mother is holding her 2-hour-old son. The delivery occurred on the due date. His Apgar score was 9 at both 1 and 5 minutes. The mother asks the nurse why her son was so wide awake right after birth, and now is sleeping so soundly. What is the nurses best response? 1. Dont worry. Babies go through a lot of these little phases. 2. Your son is in the sleep phase. Hell wake up soon. 3. Your son is exhausted from being born, and will sleep 6 more hours. 4. Your breastfeeding efforts have caused excessive fatigue in your son.

B

A new mother who is breastfeeding tells the nurse that her infant is spitting up frequently, has very loose stools and copious gas, and feeds for only short periods of time. The nurse suspects a feeding intolerance and, after questioning the mother about her diet, suggests that she do which of the following? 1. Stop breastfeeding and switch to formula. 2. Eliminate dairy products from her diet. 3. Supplement breastfeeding with a soy-based formula. 4. Offer the baby water between feedings.

B

A nurse examining a prenatal client recognizes that a lag in progression of measurements of fundal height from week to week and month to month could signal what condition? 1. Twin pregnancy 2. Intrauterine growth restriction 3. Hydramnios 4. Breech position

B

A woman has been admitted for an external version. She has completed an ultrasound exam and is attached to the fetal monitor. Prior to the procedure, why will terbutaline be administered? 1. To provide analgesia 2. To relax the uterus 3. To induce labor 4. To prevent hemorrhage

B

During labor, the fetus was in a brow presentation, but after a prolonged labor, the fetus converted to face presentation and was delivered vaginally with forceps assist. What should the nurse explain to the parents? 1. The infant will need to be observed for meconium aspiration. 2. Facial edema and head molding will subside in a few days. 3. The infant will be given prophylactic antibiotics. 4. Breastfeeding will need to be delayed for a day or two.

B

During the examination of a patient who is pregnant 24 weeks, the healthcare provider performs the maneuver in the diagram below. What is being measured on this patient? Subpubic angle Transverse diameter Anteroposterior sagittal diameter Height and angle of the symphysis pubis

B

Dystocia encompasses many problems in labor. What is the most common? 1. Meconium-stained amniotic fluid 2. Dysfunctional uterine contractions 3. Cessation of contractions 4. Changes in the fetal heart rate

B

Forceps are being used to deliver the fetus of a laboring patient, as identified in the diagram. In which direction should the healthcare provider use the forceps to guide the fetus for delivery? Upward and outward Downward and outward Midline and towards the left Midline and toward the right

B

Mild or chronic anemia in an infant may be treated adequately which of the following? 1. Transfusions with O-negative or typed and cross-matched packed red cells 2. Iron supplements or iron-fortified formulas 3. Steroid therapy 4. Antibiotics or antivirals

B

One day after giving birth vaginally, a client develops painful vesicular lesions on her perineum and vulva. She is diagnosed with a primary herpes simplex 2 infection. What is the expected care for her neonate? 1. Meticulous hand washing and antibiotic eye ointment administration. 2. Intravenous acyclovir (Zovirax) and contact precautions. 3. Cultures of blood and CSF and serial chest x-rays every 12 hours. 4. Parental rooming-in and four intramuscular injections of penicillin.

B

Placing the baby at mother's breast facilitates early latch and promotes successful breastfeeding. When should breastfeeding be initiated? 1. 6 to 12 hours after birth 2. Within 1 hour of birth 3. 24 hours after birth 4. 48 hours after birth

B

The client at 40 weeks' gestation reports to the nurse that she has had increased pelvic pressure and increased urinary frequency. Which response by the nurse is best? 1. "Unless you have pain with urination, we don't need to worry about it." 2. "These symptoms usually mean the baby's head has descended further." 3. "Come in for an appointment today and we'll check everything out." 4. "This might indicate that the baby is no longer in a head-down position."

B

The client delivered 30 minutes ago. Her blood pressure and pulse are stable. Vaginal bleeding is scant. The nurse should prepare for which procedure? 1. Abdominal hysterectomy 2. Manual removal of the placenta 3. Repair of perineal lacerations 4. Foley catheterization

B

The client has been pushing for two hours, and is exhausted. The fetal head is visible between contractions. The physician informs the client that a vacuum extractor could be used to facilitate the delivery. Which statement indicates that the client needs additional information about vacuum extraction assistance? 1. "A small cup will be put onto the baby's head, and a gentle suction will be applied." 2. "I can stop pushing and just rest if the vacuum extractor is used." 3. "The baby's head might have some swelling from the vacuum cup." 4. "The vacuum will be applied for a total of ten minutes or less."

B

The client has undergone an ultrasound, which estimated fetal weight at 4500 g (9 pounds 14 ounces). Which statement indicates that additional teaching is needed? 1. "Because my baby is big, I am at risk for excessive bleeding after delivery." 2. "Because my baby is big, his blood sugars could be high after he is born." 3. "Because my baby is big, my perineum could experience trauma during the birth." 4. "Because my baby is big, his shoulders could get stuck and a collarbone broken."

B

The client is in the second stage of labor. The fetal heart rate baseline is 170, with minimal variability present. The nurse performs fetal scalp stimulation. The client's partner asks why the nurse did that. What is the best response by the nurse? 1. "I stimulated the top of the fetus's head to wake him up a little." 2. "I stimulated the top of the fetus's head to try to get his heart rate to accelerate." 3. "I stimulated the top of the fetus's head to calm the fetus down before birth." 4. "I stimulated the top of the fetus's head to find out whether he is in distress."

B

The client presents for cervical ripening in anticipation of labor induction tomorrow. What should the nurse include in her plan of care for this client? 1. Apply an internal fetal monitor. 2. Monitor the client using electronic fetal monitoring. 3. Withhold oral intake and start intravenous fluids. 4. Place the client in a upright, sitting position.

B

The client with blood type O Rh-negative has given birth to an infant with blood type O Rh-positive. The infant has become visibly jaundiced at 12 hours of age. The mother asks why this is happening. What is the best response by the nurse? 1. "The RhoGAM you received at 28 weeks' gestation did not prevent alloimmunization." 2. "Your body has made antibodies against the baby's blood that are destroying her red blood cells." 3. "The red blood cells of your baby are breaking down because you both have type O blood." 4. "Your baby's liver is too immature to eliminate the red blood cells that are no longer needed."

B

The clinic nurse is assisting with an initial prenatal assessment. The following findings are present: spider nevi present on lower legs; dark pink, edematous nasal mucosa; mild enlargement of the thyroid gland; mottled skin and pallor on palms and nail beds; heart rate 88 with murmur present. What is the best action for the nurse to take based on these findings? 1. Document the findings on the prenatal chart. 2. Have the physician see the client today. 3. Instruct the client to avoid direct sunlight. 4. Analyze previous thyroid hormone lab results.

B

The community nurse is working with poor women who are formula-feeding their infants. Which statement indicates that the nurse's education session was effective? 1. "I should use only soy-based formula for the first year." 2. "I follow the instructions for mixing the powdered formula exactly." 3. "It is okay to add more water to the formula to make it last longer." 4. "The mixed formula can be left on the counter for a day."

B

The homecare nurse is examining a newborn who is sleeping on a pillow in a basket, covered with a fluffy blanket. There is also a stuffed animal in the basket. The most important nursing action is to do which of the following? 1. Remove the stuffed animal from the basket and place it on the floor. 2. Teach the parents the risk of SIDS from soft items in the infant's bed. 3. Make certain that the blanket is firmly tucked under the baby. 4. Ask whether the color of the blanket has cultural significance.

B

The homecare nurse is seeing a client at 6 weeks postpartum. Which statement by the client indicates the need for immediate intervention? 1. "The baby sleeps 7 hours each night now." 2. "My flow is red, and I need to wear a pad." 3. "My breasts no longer leak between feedings." 4. "I started back on the pill 2 weeks ago."

B

The labor and delivery nurse is reviewing charts. The nurse should inform the supervisor about which client? 1. Client at 5 cm requesting labor epidural analgesia 2. Client whose cervix remains at 6 cm for 4 hours 3. Client who has developed nausea and vomiting 4. Client requesting her partner to stay with her

B

The laboring client and her partner have arrived at the birthing unit. Which step of the admission process should be undertaken first? 1. The sterile vaginal exam 2. Welcoming the couple 3. Auscultation of the fetal heart rate 4. Checking for ruptured membranes

B

The laboring client is at 7 cm, with the vertex at a +1 station. Her birth plan indicates that she and her partner took Lamaze prenatal classes, and they have planned on a natural, unmedicated birth. Her contractions are every 3 minutes and last 60 seconds. She has used relaxation and breathing techniques very successfully in her labor until the last 15 minutes. Now, during contractions, she is writhing on the bed and screaming. Her labor partner is rubbing the client's back and speaking to her quietly. Which nursing diagnosis should the nurse incorporate into the plan of care for this client? 1. Fear/Anxiety related to discomfort of labor and unknown labor outcome 2. Pain, Acute, related to uterine contractions, cervical dilatation, and fetal descent 3. Coping: Family, Compromised, related to labor process 4. Knowledge, Deficient, related to lack of information about normal labor process and comfort measures

B

The laboring client is having moderately strong contractions lasting 60 seconds every 3 minutes. The fetal head is presenting at a -2 station. The cervix is 6 cm and 100% effaced. The membranes spontaneously ruptured prior to admission, and clear fluid is leaking. Fetal heart tones are in the 140s with accelerations to 150. Which nursing action has the highest priority? 1. Encourage the husband to remain in the room. 2. Keep the client on bed rest at this time. 3. Apply an internal fetal scalp electrode. 4. Obtain a clean-catch urine specimen.

B

The laboring client participated in childbirth preparation classes that strongly discouraged the use of medications and intervention during labor. The client has been pushing for two hours, and is exhausted. The physician requests that a vacuum extractor be used to facilitate the birth. The client first states that she wants the birth to be normal, then allows the vacuum extraction. Following this, what should the nurse assess the client for after the birth? 1. Elation, euphoria, and talkativeness 2. A sense of failure and loss 3. Questions about whether or not to circumcise 4. Uncertainty surrounding the baby's name

B

The neonate was born 5 minutes ago. The body is bluish. The heart rate is 150. The infant is crying strongly. The infant cries when the sole of the foot is stimulated. The arms and legs are flexed, and resist straightening. What should the nurse record as this infant's Apgar score? 1. 7 2. 8 3. 9 4. 10

B

The nurse assesses a sleeping 1-hour-old, 39-weeks'-gestation newborn. The assessment data that would be of greatest concern would be which of the following? 1. Temperature 97.9°F 2. Respirations 68 breaths/minute 3. Vital signs stable for only 2 hours 4. Heart rate 156 beats/min

B

The nurse assesses four newborns. Which of the following assessment findings would place a newborn at risk for developing physiologic jaundice? A. Telangiectatic nevi B. Cephalohematoma C. Mongolian spots D. Molding

B

The nurse has just assisted the father in bathing the newborn 2 hours after birth. The nurse explains that the newborn must remain in the radiant warmer. This is based on which assessment data? 1. Heart rate 120 2. Temperature 96.8°F 3. Respiratory rate 50 4. Temperature 99.6°F

B

The nurse has presented a community education class on recommended health screenings for women. Which statement about the Pap smear by a class member indicates that additional teaching is necessary? "It is recommended for women 21 years of age and older." "It diagnoses cervical cancer." "Intercourse at a young age is a risk factor for an abnormal Pap smear." "Detects abnormal cells."

B

The nurse is admitting a client to the birthing unit. What question should the nurse ask to gain a better understanding of the client's psychosocial status? 1. "How did you decide to have your baby at this hospital?" 2. "Who will be your labor support person?" 3. "Have you chosen names for your baby yet?" 4. "What feeding method will you use for your baby?"

B

The nurse is assessing a 2-hour-old newborn delivered by cesarean at 38 weeks. The amniotic fluid was clear. The mother had preeclampsia. The newborn has a respiratory rate of 80, is grunting, and has nasal flaring. What is the most likely cause of this infant's condition? 1. Meconium aspiration syndrome 2. Transient tachypnea of the newborn 3. Respiratory distress syndrome 4. Prematurity of the neonate

B

The nurse is assessing a primiparous client who indicates that her religion is Judaism. Why is this information is pertinent for the nurse to assess? 1. Religious and cultural background can impact what a client eats during pregnancy. 2. It provides a baseline from which to ask questions about the client's religious and cultural background. 3. Knowing the client's beliefs and behaviors regarding pregnancy is not important. 4. Clients sometimes encounter problems in their pregnancies based on what religion they practice.

B

The nurse is assisting a mother to bottle-feed her newborn, who has been crying. The nurse suggests that prior to feeding, the mother should do which of the following? 1. Offer a pacifier 2. Burp the newborn 3. Unwrap the newborn 4. Stroke the newborn's spine and feet

B

The nurse is assisting in the delivery of a pregnant patient's placenta post-delivery. The placenta appears as follows. What term should the nurse use to document this placental delivery? Schultz mechanism of delivery Duncan mechanism of delivery Complete mechanism of delivery Incomplete mechanism of delivery

B

The nurse is caring for a jaundiced infant receiving bank light phototherapy in an isolette. Which finding requires an immediate intervention? 1. Eyes are covered, no clothing on, diaper in place 2. Axillary temperature 99.7°F 3. Infant removed from the isolette for breastfeeding 4. Loose bowel movement

B

The nurse is caring for a newborn who was recently circumcised. Which nursing intervention is appropriate following the procedure? 1. Keep the infant NPO for 4 hours following the procedure. 2. Observe for urine output. 3. Wrap dry gauze tightly around the penis. 4. Clean with cool water with each diaper change.

B

The nurse is completing the gestational age assessment on a newborn while in the mother's postpartum room. During the assessment, the mother asks what aspects of the baby are being checked. What is the nurse's best response? 1. "I'm checking to make sure the baby has all of its parts." 2. "This assessment looks at both physical aspects and the nervous system." 3. "This assessment checks the baby's brain and nerve function." 4. "Don't worry. We perform this check on all the babies."

B

The nurse is explaining the difference between descriptive statistics and inferential statistics to a group of student nurses. To illustrate descriptive statistics, what would the nurse use as an example? A) A positive correlation between breastfeeding and infant weight gain B) The infant mortality rate in the state of Oklahoma C) A causal relationship between the number of sexual partners and sexually transmitted infections D) The total number of spontaneous abortions in drug-abusing women as compared with non-drug-abusing women

B

The nurse is making a postpartum home visit in the summer. The new father asks about taking the baby to a family outing this weekend. The nurse should encourage the father to do which of the following? 1. Cover the infant with dark blankets to block the sun. 2. Keep the infant in the shade. 3. Uncover the infant's head to prevent hyperthermia. 4. Avoid taking the infant outdoors for 6 months.

B

The nurse is making an initial assessment of the newborn. Which of the following data would be considered normal? A. Chest circumference 38 cm, head circumference 31.5 cm B. Chest circumference 31.5 cm, head circumference 33.5 cm C. Chest circumference 32.5 cm, head circumference 36 cm D. Chest circumference 30 cm, head circumference 29 cm

B

The nurse is planning care for four infants who were born on this shift. The infant who will require the most detailed assessment is the one whose mother has which of the following? 1. A history of obsessive-compulsive disorder (OCD) 2. Chlamydia 3. Delivered six other children by cesarean section 4. A urinary tract infection (UTI)

B

The nurse is preparing a report on the number of births by three service providers at the facility (certified nurse-midwives, family practitioners, and obstetricians). What is this an example of? A) Inferential statistics B) Descriptive statistics C) Evidence-based practice D) Secondary use of data

B

The nurse is preparing to assess the pregnant client's fundal height during a routine prenatal visit. Which nursing action is appropriate in this situation? Telling the client not to eat or drink for one hour after the procedure Asking the client to empty her bladder prior to the procedure Obtaining informed consent for the procedure Assessing blood pressure after the procedure

B

The nurse is reviewing care of clients on a mother-baby unit. Which situation should be reported to the supervisor? A) A 2-day-old infant has breastfed every 2-3 hours and voided four times. B) An infant was placed in the wrong crib after examination by the physician. C) The client who delivered by cesarean birth yesterday received oral narcotics. D) A primiparous client who delivered today is requesting discharge within 24 hours.

B

The nurse is teaching a newborn care class to parents who are about to give birth to their first babies. Which statement by a parent indicates that teaching was effective? 1. My baby will be able to focus on my face when she is about a month old. 2. My baby might startle a little if a loud noise happens near him. 3. Newborns prefer sour tastes. 4. Our baby wont have a sense of smell until she is older.

B

The nurse is training a nurse new to the labor and delivery unit. They are caring for a laboring client who will have a forceps delivery. Which action or assessment finding requires intervention? 1. Regional anesthesia is administered via pudendal block. 2. The client is instructed to push between contractions. 3. Fetal heart tones are consistently between 110 and 115. 4. The client's bladder is emptied using a straight catheter.

B

The nurse is working with a mother who has just delivered her third child at 33 weeks' gestation. The mother says to the nurse, "This baby doesn't turn her head and suck like the older two children did. Why?" What is the best response by the nurse? "Every baby is different. This is just one variation of normal that we see on a regular basis." "This baby might not have a rooting or sucking reflex because she is premature." "When she is wide awake and alert, she will probably root and suck even if she is early." "She might be too tired from the birthing process and need a couple of days to recover."

B

The nurse is working with a new mother who delivered yesterday. The mother has chosen to breastfeed her infant. Which demonstration of skill is the best indicator that the client understands breastfeeding? 1. She puts the infant to breast when he is asleep to help wake him up. 2. She takes off her gown to achieve skin-to-skin contact. 3. She leans toward the infant so that he turns his head to access the nipple. 4. The infant is crying when he is brought to the breast.

B

The nurse is working with a pregnant adolescent. The client asks the nurse how the baby's condition is determined during labor. The nurse's best response is that during labor, the nurse will do which of the following? 1. Check the client's cervix by doing a pelvic exam every 2 hours. 2. Assess the fetus's heart rate with an electronic fetal monitor. 3. Look at the color and amount of bloody show that the client has. 4. Verify that the client's contractions are strong but not too close together.

B

The nurse notes that a 36-hour-old newborn's serum bilirubin level has increased from 14 mg/dL to 16.6 mg/dL in an 8-hour period. What nursing intervention would be included in the plan of care for this newborn? 1. Continue to observe 2. Begin phototherapy 3. Begin blood exchange transfusion 4. Stop breastfeeding

B

The nurse obtains the following device to be used on a newborn. For which procedure is the nurse preparing this infant? Hearing test Circumcision Neurological exam Assess for umbilical hernia

B

The nurse receives a phone call from a 25-year-old woman experiencing breast tenderness in the week prior to her menses, with palpable breast nodularity, without nipple discharge. What is the best response by the nurse? "Please make an appointment at the breast cancer center as soon as possible." "How much salty food do you regularly consume?" "As long as you don't have nipple discharge, it isn't a serious condition." "Eliminate caffeine and chocolate from your diet."

B

The nurse receives a phone call from a client who claims she is pregnant. The client reports that she has regular menses that occur every 28 days and last 5 days. The first day of her last menses was April 10. What would the client's estimated date of delivery (EDD) be if she is pregnant? 1. Nov. 13 2. Jan. 17 3. Jan. 10 4. Dec. 3

B

The nurse should anticipate the labor pattern for a fetal occiput posterior position to be which of the following? 1. Shorter than average during the latent phase 2. Prolonged as regards the overall length of labor 3. Rapid during transition 4. Precipitous

B

The nurse suspects clubfoot in the newborn. It is appropriate to: A. Stimulate the sole of the foot B. Move the foot to midline and determine resistance C. Extend the foot and observe for pain D. Adduct the foot and listen for a click

B

The nurse working in an outpatient obstetric clinic assesses four primigravida clients. Which client findings would the nurse tell the physician about? 1. 17 weeks' gestation and client denies feeling fetal movement 2. 24 weeks' gestation and fundal height is at the umbilicus 3. 4-6 weeks' gestation and softening of the cervix 4. 34 weeks' gestation and complains of hemorrhoidal pain

B

The nursing instructor explains to the class that according to the 1973 Supreme Court decision in Roe v. Wade, abortion is legal if induced: A) Before the 30th week of pregnancy. B) Before the period of viability. C) To provide tissue for therapeutic research. D) Can be done any time if mother, doctor, and hospital all agree.

B

The special care nursery nurse is working with parents of a 3-day-old infant who was born with myelomeningocele and has developed an infection. Which statement from the mother is unexpected? 1. "If I had taken better care of myself, this wouldn't have happened." 2. "I've been sleeping very well since I had the baby." 3. "This is probably the doctor's fault." 4. "If I hadn't seen our baby's birth, I wouldn't believe she is ours."

B

The student nurse has performed a gestational age assessment of an infant, and finds the infant to be at 32 weeks. On which set of characteristics is the nurse basing this assessment? 1. Lanugo mainly gone, little vernix across the body 2. Prominent clitoris, enlarging minora, anus patent 3. Full areola, 5 to 10 mm bud, pinkish-brown in color 4. Skin opaque, cracking at wrists and ankles, no vessels visible

B

What indications would lead the nurse to suspect sepsis in a newborn? 1. Respiratory distress syndrome developing 48 hours after birth 2. Temperature of 97.0°F 2 hours after warming the infant from 97.4°F 3. Irritability and flushing of the skin at 8 hours of age 4. Bradycardia and tachypnea developing when the infant is 36 hours old

B

What type of forceps are designed to be used with a breech presentation? 1. Midforceps 2. Piper 3. Low 4. High

B

Which client requires immediate intervention by the labor and delivery nurse? 1. Client at 8 cm, systolic blood pressure has increased 35 mm Hg 2. Client who delivered 1 hour ago with WBC of 50,000 3. Client at 5 cm with a respiratory rate of 22 between contractions 4. Client in active labor with polyuria

B

Which of the following functions primarily to provide low-income women and children who are at risk for medical or nutritional problems with nutritious foods to supplement their diets, nutrition education and counseling, and screening and referrals to other health, welfare, and social programs? 1. ABM 2. WIC 3. ILCA 4. LLLI

B

14) The nurse is working with a new mother who follows Muslim traditions. Which expectations and actions are appropriate for this client? Select all that apply. 1. To be sure she gets a kosher diet. 2. Expect that most visitors will be women. 3. Uncover only the necessary skin when assessing. 4. The father will take an active role in infant care. 5. She will prefer a male physician.

B, C

27) The postpartum client is suspected of having acute cystitis. Which symptoms would the nurse expect to see in this client? Select all that apply. 1. High fever 2. Frequency 3. Suprapubic pain 4. Chills 5. Nausea and vomiting

B, C

A client has just arrived in the birthing unit. What steps would be most important for the nurse to perform to gain an understanding of the physical status of the client and her fetus? Note: Credit will be given only if all correct choices and no incorrect choices are selected. Select all that apply. 1. Check for ruptured membranes and apply a fetal scalp electrode. 2. Auscultate the fetal heart rate between and during contractions. 3. Palpate contractions and resting uterine tone. 4. Assess the blood pressure, temperature, respiratory rate, and pulse rate. 5. Perform a vaginal exam for cervical dilation, and perform Leopold maneuvers.

B, C

During an interview the nurse learns that a patient's sister was recently diagnosed with endometrial cancer. What should the nurse review to reduce the patient's risk for developing the same disease process? Select all that apply. Smoking cessation Maintain a normal body mass index Consider birth control without estrogen Limit the intake of alcohol to one drink per day Increase exercise to 30 minutes most days of the week

B, C

The nurse is assessing a newborn diagnosed with physiologic jaundice. Which findings would the nurse expect? Note: Credit will be given only if all correct choices and no incorrect choices are selected. Select all that apply. 1. Jaundice present within the first 24 hours of life 2. Appearance of jaundice symptoms after 24 hours of life 3. Yellowish coloration of the sclera of the eyes 4. Cephalohematoma or excessive bruising 5. Cyanosis

B, C

1) Which questions are appropriate for the nurse to ask during a cultural assessment of a client who is new to the clinic? Select all that apply. A) What genetic and other biological differences affect caregiving? B) Which family member must be consulted for decisions about care? C) What type of healthcare provider is the most appropriate? D) Does the client have beliefs or traditions that might impact the care plan? E) Are communications patterns established?

B, C, D

23) When caring for a new mother after cesarean birth, what complications would the nurse anticipate? Select all that apply. 1. Back pain 2. Pulmonary infection 3. Deep vein thrombosis 4. Pulmonary embolism 5. Perineal edema

B, C, D

Nonreassuring fetal status often occurs with a tachysystole contraction pattern. Intrauterine resuscitation measures may become warranted and can include which of the following measures? Note: Credit will be given only if all correct choices and no incorrect choices are selected. Select all that apply. 1. Position the woman on her right side. 2. Apply oxygen via face mask. 3. Call for anesthesia provider for support. 4. Increase intravenous fluids by at least 700 mL bolus. 5. Call the physician/CNM to the bedside.

B, C, D

Prior to conducting the initial assessment of a newborn, the nurse reviews the mother's prenatal record and the delivery record to obtain information concerning possible risk factors for the infant and to anticipate the impact of these factors on the infant's ability to successfully transition to the extrauterine environment. Which information is pertinent to this assessment? Note: Credit will be given only if all correct choices and no incorrect choices are selected. Select all that apply. 1. Drug or alcohol use by the father 2. Infectious disease screening results 3. Maternal history of gestational diabetes 4. Prolonged rupture of the membranes 5. Maternal use of prenatal vitamins

B, C, D

The nurse is seeing a client who asks about the accuracy of Nagele's rule. The nurse explains that accuracy can be compromised under which conditions? Note: Credit will be given only if all correct and no incorrect choices are selected. Select all that apply. 1. There is a history of regular menses every 28 days. 2. Amenorrhea is present and ovulation occurs with breastfeeding. 3. Oral contraception was discontinued, but no regular menstruation was established. 4. There has been 1 or more months of amenorrhea. 5. There is an accurate date for the last menstrual period.

B, C, D

Under which circumstances would the nurse remove prostaglandin from the client's cervix? Note: Credit will be given only if all correct choices and no incorrect choices are selected. Select all that apply. 1. Contractions every 5 minutes 2. Nausea and vomiting 3. Uterine tachysystole 4. Cardiac tachysystole 5. Baseline fetal heart rate of 140-148

B, C, D

Which instructions should the nurse include when teaching parents of a newborn about caring for the umbilical cord? Note: Credit will be given only if all correct choices and no incorrect choices are selected. Select all that apply. 1. Use triple-dye to cleanse the umbilical cord at home. 2. Fold the diaper down to prevent covering the cord stump. 3. Keep the umbilical stump clean and dry to avoid infection. 4. Observe for signs of infection such as foul smell, redness, and drainage. 5. Begin tub baths to help cleanse the cord stump at home.

B, C, D

1) A female patient experiencing menopause is concerned that periodic lapses of memory are symptoms of Alzheimer disease. What should the nurse review with the patient to reduce the risk of developing Alzheimer disease (A D)? Select all that apply. 1. Increase rest 2. Stop smoking 3. Exercise regularly 4. Eat a healthy diet 5. Maintain mental activity

B, C, D, E

1) During a follow-up wellness visit, the nurse determines that a female client is experiencing favorable outcomes after starting combined oral contraceptives. What data did the nurse use to determine this? Select all that apply. A) Reduced appetite B) Reduced menstrual flow C) Fewer menstrual cramps D) No pain with ovulation E) Cycle is regular at 28 days

B, C, D, E

28) The nurse is caring for a client who plans to relinquish her baby for adoption. The nurse would implement which approach to care? Select all that apply. 1. Encourage the client to see and hold her infant. 2. Encourage the client to express her emotions. 3. Respect any special requests for the birth. 4. Acknowledge the grieving process in the client. 5. Allow access to the infant, if the client requests it.

B, C, D, E

The nurse is aware of the different breathing techniques that are used during labor. Why are breathing techniques used during labor? Select all that apply. 1. They are a form of anesthesia. 2. They are a source of relaxation. 3. They increase the ability to cope with contractions. 4. They are a source of distraction. 5. They increase a woman's pain threshold.

B, C, D, E

The nurse knows that the maternal risks associated with postterm pregnancy include which of the following? Note: Credit will be given only if all correct choices and no incorrect choices are selected. Select all that apply. 1. Polyhydramnios 2. Maternal hemorrhage 3. Maternal anxiety 4. Forceps-assisted delivery 5. Perineal damage

B, C, D, E

Which of the following potential problems would the nurse consider when planning care for a client with a persistent occiput posterior position of the fetus? Note: Credit will be given only if all correct choices and no incorrect choices are selected. Select all that apply. 1. Increased fetal mortality 2. Severe perineal lacerations 3. Ceasing of labor progress 4. Fetus born in posterior position 5. Intense back pain during labor

B, C, D, E

19) Nursing interventions that foster the process of becoming a mother include which of the following? 1. Encouraging detachment from the nurse-patient relationship 2. Promoting maternal-infant attachment 3. Building awareness of and responsiveness to infant interactive capabilities 4. Instruct about promoting newborn independence 5. Preparing the woman for the maternal social role

B, C, E

During the initial prenatal visit, the nurse assesses the history of the father of the child for which of the following? Note: Credit will be given only if all correct and no incorrect choices are selected. Select all that apply. 1. Stability of living conditions 2. Blood type and Rh type 3. Significant health problems 4. Nutritional history 5. Current use of tobacco

B, C, E

Maternal risks of occiput posterior (OP) malposition include which of the following? Note: Credit will be given only if all correct choices and no incorrect choices are selected. Select all that apply. 1. Blood loss greater than 1000 mL 2. Postpartum infection 3. Anal sphincter injury 4. Higher rates of vaginal birth 5. Instrument delivery

B, C, E

What signs would indicate that a pregnant client's urinalysis culture was abnormal? Note: Credit will be given only if all correct and no incorrect choices are selected. Select all that apply. 1. pH 4.6-8 2. Alkaline urine 3. Cloudy appearance 4. Negative for protein and red blood cells 5. Hemoglobinuria

B, C, E

Which nursing interventions are appropriate when caring for the newborn undergoing phototherapy? Note: Credit will be given only if all correct choices and no incorrect choices are selected. Select all that apply. 1. Cover the newborn's eyes at all times, even when not under the lights. 2. Close the newborn's eyelids before applying eye patches. 3. Inspect the eyes each shift for conjunctivitis. 4. Keep the baby swaddled in a blanket to prevent heat loss. 5. Reposition the baby every 2 hours.

B, C, E

The nurse is preparing to administer a sitz bath to a postpartum patient. In which order should the nurse perform the steps of this procedure? Open the clamp on the tubing Anchor the infusion bag to the sitz bath basin, with the tube facing upward Fill the drainage bag with warm or cool water up to the top line as indicated on the bag Close the clamp on the tubing, dry perineum with a clean towel, and apply new peri-pad Secure the drainage bag from a hook over the toilet or from the handle used to flush the toilet

B, C, E, A, D

30) Which physical assessment findings would the nurse consider normal for the postpartum client following a vaginal delivery? Select all that apply. 1. Elevated blood pressure 2. Fundus firm and midline 3. Moderate amount of lochia serosa 4. Edema and bruising of perineum 5. Inflamed hemorrhoids

B, D

In which clinical situations would it be appropriate for an obstetrician to order a labor nurse to perform amnioinfusion? Note: Credit will be given only if all correct choices and no incorrect choices are selected. Select all that apply. 1. Placental abruption 2. Meconium-stained fluid 3. Polyhydramnios 4. Variable decelerations 5. Early decelerations

B, D

The nurse interviews a 28-year-old client with a new medical diagnosis of endometriosis. Which question asked by the nurse is appropriate? Select all that apply. "Are you having hot flashes?" "Are you experiencing pain during intercourse?" "Is a vaginal discharge present?" "Are you having pain during your period?" "Have you noticed any skin rashes?"

B, D

The nurse is preparing to instruct the parents of a newborn on the care of the umbilical cord. In which order should the nurse provide these instructions? Check the cord for color Wash hands with soap and water Fold diaper below umbilical cord Clean cord and base of cord with cotton swab Check the cord for odor or oozing of green material

B, D, A, E, C

29) What possible approaches should the nurse use to provide sensitive, holistic nursing care for the mother who is relinquishing her newborn? Select all that apply. 1. Allow the mother minimal control over the infant. 2. Use active listening strategies to determine the clients needs. 3. Provide only physical care in the early postpartum period. 4. Demonstrate empathy, concern, and compassion. 5. Provide nonjudgmental support and personalized care.

B, D, E

32) During the first several postpartum weeks, the new mother must accomplish certain physical and developmental tasks, including which of the following? Select all that apply. 1. Establish a therapeutic relationship with her physician 2. Adapt to altered lifestyles and family structure resulting from the addition of a new member 3. Restore her intellectual abilities 4. Restore physical condition 5. Develop competence in caring for and meeting the needs of her infant

B, D, E

A client at 40 weeks' gestation is to undergo stripping of the membranes. The nurse provides the client with information about the procedure. Which information is accurate? Note: Credit will be given only if all correct choices and no incorrect choices are selected. Select all that apply. 1. Intravenous administration of oxytocin will be used to initiate contractions. 2. The physician/CNM will insert a gloved finger into the cervical os and rotate the finger 360 degrees. 3. Stripping of the membranes will not cause discomfort, and is usually effective. 4. Labor should begin within 24-48 hours after the procedure. 5. Uterine contractions, cramping, and a bloody discharge can occur after the procedure.

B, D, E

Amniotomy as a method of labor induction has which of the following advantages? Note: Credit will be given only if all correct choices and no incorrect choices are selected. Select all that apply. 1. The danger of a prolapsed cord is decreased. 2. There is usually no risk of hypertonus or rupture of the uterus. 3. The intervention can cause a decrease in pain. 4. The color and composition of amniotic fluid can be evaluated. 5. The contractions elicited are similar to those of spontaneous labor.

B, D, E

During a pelvic examination, a patient is diagnosed with a Bartholin gland cyst. For which treatment should the nurse prepare this patient? Select all that apply. Pelvic ultrasound Antibiotic therapy Exploratory laparotomy Incision and drainage of the cyst Culture and sensitivity of the discharge

B, D, E

Risk factors for labor dystocia include which of the following? Note: Credit will be given only if all correct choices and no incorrect choices are selected. Select all that apply. 1. Tall maternal height 2. Labor induction 3. Small-for-gestational-age (SGA) fetus 4. Malpresentation 5. Prolonged latent phase

B, D, E

The nurse determines that a client is carrying her fetus in the vertical (longitudinal) lie. The nurse's judgment should be questioned if the fetal presenting part is which of the following? Note: Credit will be given only for all correct choices and no incorrect choices. Select all that apply. 1. Sacrum 2. Left arm 3. Mentum 4. Left scapula 5. Right scapula

B, D, E

The nurse is monitoring a client who is receiving an amnioinfusion. Which assessments must the nurse perform to prevent a serious complication? Note: Credit will be given only if all correct choices and no incorrect choices are selected. Select all that apply. 1. Color of amniotic fluid 2. Maternal blood pressure 3. Cervical effacement 4. Uterine resting tone 5. Fluid leaking from the vagina

B, D, E

Which of the following are considered risk factors for development of severe hyperbilirubinemia? Note: Credit will be given only if all correct choices and no incorrect choices are selected. Select all that apply. 1. Northern European descent 2. Previous sibling received phototherapy 3. Gestational age 27 to 30 weeks 4. Exclusive breastfeeding 5. Infection

B, D, E

The nurse determines the gestational age of an infant to be 40 weeks. Which characteristics are most likely to be observed? Note: Credit will be given only if all correct choices and no incorrect choices are selected. Select all that apply. Lanugo abundant over shoulders and back Plantar creases over entire sole Pinna of ear springs back slowly when folded. Vernix well distributed over entire body Testes are pendulous, and the scrotum has deep rugae

B, E

The primary care provider is performing a fetal scalp stimulation test. What result would the nurse hope to observe? Note: Credit will be given only if all correct choices and no incorrect choices are selected. Select all that apply. 1. Spontaneous fetal movement 2. Fetal heart acceleration 3. Increase in fetal heart variability 4. Resolution of late decelerations 5. Reactivity associated with the stimulation

B, E

1) A 49-year-old client comes to the clinic with complaints of severe perimenopausal symptoms including hot flashes, night sweats, urinary urgency, and vaginal dryness. The physician has prescribed a combination hormone replacement therapy of estrogen and progestin. When the client asks the nurse why she must take both hormones, what is the nurse's best reply? A) "Hot flashes respond better when replacement includes both hormones." B) "You are having very severe symptoms, so you need more hormones replaced." C) "There is an increased risk of tissue abnormality inside the uterus if only one is given." D) "Your blood pressure can become elevated if only one hormone is used."

C

1) A client asks her nurse, "Is it okay for me to take a tub bath during the heavy part of my menstruation?" What is the nurse's correct response? A) "Tub baths are contraindicated during menstruation." B) "You should shower and douche daily instead." C) "Either a bath or a shower is fine at that time." D) "You should bathe and use a feminine deodorant spray during menstruation."

C

1) A client asks the nurse, "Can you explain to us how to use the basal body temperature method to detect ovulation and prevent pregnancy?" What is the nurse's best response? A) "Take your temperature every evening at the same time and keep a record for a period of several weeks. A noticeable drop in temperature indicates that ovulation has occurred." B) "Take your temperature every day at the same time and keep a record of the findings. A noticeable rise in temperature indicates ovulation." C) "Take your temperature each day, immediately upon awakening, and keep a record of each finding. A noticeable rise in temperature indicates that ovulation is about to occur." D) "This is an unscientific and unproven method of determining ovulation, and is not recognized as a means of birth control."

C

1) A client who wants to use the vaginal sponge method of contraception shows that she understands the appropriate usage when she makes which statement? A) "I need to use a lubricant prior to insertion." B) "I need to add spermicidal cream prior to intercourse." C) "I need to moisten it with water prior to use." D) "I need to leave it in no longer than 6 hours."

C

1) A client who was raped is extremely upset when a pregnancy test confirms that she is pregnant, and requests information regarding pregnancy termination. Which statement is best for the nurse to make? A) "Abortion is morally wrong, and should not be undertaken." B) "Hypertension is a risk with any abortion." C) "Surgical abortion in the first trimester is technically easier and safer than abortion in the second trimester." D) "The most accurate method to determine gestational age are the results of a pregnancy test."

C

1) A female client is disappointed to learn that intrauterine contraception is not an option. For what reason is this form of contraception contraindicated for this client? A) Diabetes B) Breast cancer C) Endometriosis D) Uterine surgery

C

1) A patient in the 2nd trimester of pregnancy is diagnosed with bacterial vaginosis. Which medication regimen should the nurse expect to be prescribed for this patient? A) Metronidazole 500 m g orally one dose B) Metronidazole 250 m g orally once a day for 7 days C) Metronidazole 500 m g orally twice a day for 7 days D) Metronidazole 250 m g orally twice a day for 14 days

C

1) During a health interview focused on sexual history, a female patient makes a statement about douching and intercourse. What should the nurse do in response to this statement? A) Recommend the frequency of douching B) Explain the proper procedure to douche C) Take the time now to educate the patient about the practice D) Document that the patient has misunderstandings about the use of douches

C

1) Persistent early decelerations are noted. What would the nurse's first action be? A) Turn the mother on her left side and give oxygen. B) Check for prolapsed cord. C) Do nothing. This is a benign pattern. D) Prepare for immediate forceps or cesarean delivery.

C

1) The charge nurse is assessing several postpartum clients. Which client has the greatest risk for postpartum hemorrhage? 1. The client who was overdue and delivered vaginally 2. The client who delivered by scheduled cesarean delivery 3. The client who had oxytocin augmentation of labor 4. The client who delivered vaginally at 36 weeks

C

1) The client reports using an alternative therapy that involves the manipulation of soft tissues. This therapy has reduced the client's stress, diminished pain, and increased circulation. Which therapy has this client most likely received? A) Guided imagery B) Homeopathy C) Massage therapy D) Reflexology

C

1) The laboring client with meconium-stained amniotic fluid asks the nurse why the fetal monitor is necessary, as she finds the belt uncomfortable. Which response by the nurse is most important? A) "The monitor is necessary so we can see how your labor is progressing." B) "The monitor will prevent complications from the meconium in your fluid." C) "The monitor helps us to see how the baby is tolerating labor." D) "The monitor can be removed, and oxygen given instead."

C

1) The nurse determines the fundus of a postpartum client to be boggy. Initially, what should the nurse do? 1. Document the findings. 2. Catheterize the client. 3. Massage the uterine fundus until it is firm. 4. Call the physician immediately.

C

1) The nurse has just palpated contractions and compares the consistency to that of the forehead in order to estimate the firmness of the fundus. What would the intensity of these contractions be identified as? A) Mild B) Moderate C) Strong D) Weak

C

1) The nurse in the clinic instructs a client who is using the natural method of contraception to begin counting the first day of her cycle as which day? A) The day her menstrual period ceases B) The first day after her menstrual period ceases C) The first day of her menstrual period D) The day of ovulation

C

1) The nurse in the community should use a family assessment tool to obtain what type of information? A) How long the family has lived at its current address B) What other health insurance the family has had in the past C) How the family meets its nutritional needs and obtains food D) What eye color the family desires in its unborn child

C

1) The nurse is admitting a client to the labor and delivery unit. Which aspect of the client's history requires notifying the physician? A) Blood pressure 120/88 B) Father a carrier of sickle-cell trait C) Dark red vaginal bleeding D) History of domestic abuse

C

1) The nurse is assessing a client who reports seeing an acupuncturist on a weekly basis to treat back pain. The nurse understands that acupuncture is an example of what? A) A risky practice without evidence of efficacy B) A folk remedy C) A complementary therapy D) An alternative therapy

C

1) The nurse is aware that a fetus that is not in any stress would respond to a fetal scalp stimulation test by showing which change on the monitor strip? A) Late decelerations B) Early decelerations C) Accelerations D) Fetal dysrhythmia

C

1) The nurse is developing a teaching plan for a client undergoing a tubal ligation. What information should be included in the plan? A) The surgical procedure is easily reversible. B) Laparotomy is performed following a vaginal birth. C) Minilaparotomy is performed in the postpartum period soon after a vaginal birth. D) Tubal ligation can be done at any time the woman is either pregnant or not pregnant.

C

1) The nurse is preparing educational materials at a family planning clinic. The client who is an appropriate candidate for using emergency contraception would be one who reports which of the following? A) Forgetting to start her pill pack yesterday B) Unprotected intercourse during her menses C) That a condom broke yesterday in the middle of her cycle D) Increased dysmenorrhea since I U C insertion

C

1) The nurse is providing discharge instructions to a client with a diagnosis of vulvovaginal candidiasis (V V C), and knows the client understands when she makes which of the following statements? A) "I need to apply the miconazole for 10 days." B) "I need to douche daily." C) "I need to add yogurt to my diet." D) "I need to wear nylon panties."

C

1) The nurse is speaking to students about changes in maternal-newborn care. One change is that self-care has gained wide acceptance with clients and the healthcare community due to research findings that suggest that it has which effect? A) Shortens newborn length of stay B) Decreases use of home health agencies C) Decreases healthcare costs D) Decreases the number of emergency department visits

C

1) The nurse is teaching a group of women about menopause at a community clinic. The nurse tells them that the best indicator of menopause is which of the following symptoms? A) No menses for 8 consecutive months B) Hot flashes and night sweats C) F S H levels rise and ovarian follicles cease to produce estrogen D) Diagnosed with osteoporosis 4 months ago

C

1) The nurse provides a couple with education about the consequences of not treating chlamydia, and knows they understand when they make which statement? A) "She could become pregnant." B) "She could have severe vaginal itching." C) "He could get an infection in the tube that carries the urine out." D) "It could cause us to develop a rash."

C

1) Which of the following diagnostic tests would the nurse question when ordered for a client diagnosed with pelvic inflammatory disease (P I D)? A) C B C (complete blood count) with differential B) Venereal Disease Research Laboratory (V D R L) C) Throat culture for Streptococcus A D) R P R (Rapid Plasma Reagin)

C

1) Women with pyelonephritis during pregnancy are at significantly increased risk for which condition? A) Foul-smelling discharge B) Ectopic pregnancy C) Preterm labor D) A colicky large intestine

C

1) he labor and delivery nurse is assigned to four clients in early labor. Which electronic fetal monitoring finding would require immediate intervention? A) Early decelerations with each contraction B) Variable decelerations that recover to the baseline C) Late decelerations with minimal variability D) Accelerations

C

10) Which relief measure would be most appropriate for a postpartum client with superficial thrombophlebitis? 1. Urge ambulation 2. Apply ice to the leg 3. Elevate the affected limb 4. Massage her calf

C

11) The breastfeeding client asks the nurse about appropriate contraception. What is the nurses best response? 1. Breastfeeding has many effects on sexual intercourse. 2. IUDs are easy to use and easy to insert prior to sexual intercourse. 3. Its possible to get pregnant before your menstrual period returns. Lets talk about some different options for contraception. 4. Breastfeeding hampers ovulation, so no contraception is needed.

C

11) Which statement by a new mother 1 week postpartum indicates maternal role attainment? 1. I don't think I'll ever know what I'm doing. 2. This baby feels like a real stranger to me. 3. It works better for me to undress the baby and to nurse in the chair rather than the bed. 4. My sister took to mothering in no time. Why cant I?

C

12) On the first postpartum day, the nurse teaches the client about breastfeeding. Two hours later, the mother seems to remember very little of the teaching. The nurse understands this memory lapse to be related to which of the following? 1. The taking-hold phase 2. Postpartum hemorrhage 3. The taking-in period 4. Epidural anesthesia

C

19) What is the advantage of a client using a patient-controlled analgesia (PCA) following a cesarean birth? 1. The client receives a bolus of the analgesia when pressing the button. 2. The client experiences pain relief within 30 minutes. 3. The client feels a greater sense of control, and is less dependent on the nursing staff. 4. The client can deliver as many doses of the medication as needed.

C

2) The nurse is providing education to the new family. Which question by the nurse is best? 1. Do you know how to give the baby a bath? 2. You have diapers and supplies at home, right? 3. How have your breastfeedings been going? 4. How much formal education do you have?

C

20) The postpartum client states that she doesnt understand why she cant enjoy being with her baby. What would the nurse be concerned about? 1. Postpartum psychosis 2. Postpartum infection 3. Postpartum depression 4. Postpartum blues

C

20) The postpartum nurse is caring for a client who gave birth to full-term twins earlier today. The nurse will know to assess for symptoms of which of the following? 1. Increased blood pressure 2. Hypoglycemia 3. Postpartum hemorrhage 4. Postpartum infection

C

21) The nurse is observing a new graduate perform a postpartum assessment. Which action requires intervention by the nurse? 1. Asking the client to void and donning clean gloves 2. Listening to bowel sounds and then asking when her last bowel movement occurred 3. Offering the patient pre-medication 2 hours before the assessment 4. Completing the assessment and explaining the results to the client

C

22) The client delivered her second child 1 day ago. The clients temperature is 101.4 F, her pulse is 100, and her blood pressure is 110/70. Her lochia is moderate, serosanguinous, and malodorous. She is started on IV antibiotics. The nurse provides education for the client and her partner. Which statement indicates that teaching has been effective? 1. "This condition is called parametritis." 2. "Gonorrhea is the most common organism that causes this type of infection." 3. "My positive Beta-strep culture might have contributed to this problem." 4. "If I had walked more yesterday, this probably wouldn't have happened."

C

23) The postpartum client has developed thrombophlebitis in her right leg. Which finding requires immediate intervention? 1. The client reports she had this condition after her last pregnancy. 2. The client develops pain and swelling in her left lower leg. 3. The client appears anxious, and describes pressure in her chest. 4. The client becomes upset that she cannot go home yet.

C

24) The nurse is supervising a student nurse who is working with a 14-year-old client who delivered her first child yesterday. Which statement indicates that the nursing student understands the particular needs of an adolescent client? 1. This client will need less teaching, because she will have gotten the right information in school. 2. Because of her age, this client will require less frequent fundal checks to assess for postpartal hemorrhage. 3. Because of her age, this client will probably need extra teaching about the terminology for her anatomy. 4. This client will need to have her grandmother provide day care and help raise the baby.

C

25) The nurse is caring for a 15-year-old client who gave birth to her first child yesterday. What action is the best indicator that the nurse understands the parenting adolescent? 1. The clients mother is included in all discussions and demonstrations. 2. The father of the baby is encouraged to change a diaper and give a bottle. 3. The nurse explains the characteristics and cues of the baby when assessing him. 4. A discussion on contraceptive methods is the first topic of teaching.

C

31) The nurse is preparing a teaching brochure for Spanish-speaking postpartum clients. Which topics are critical for this population? 1. Baby baths and birth certificates 2. Hygiene practices 3. When and how to contact their healthcare provider 4. Pain-relief options in labor and after birth

C

33) The postpartum client who is being discharged from the hospital experienced severe postpartum depression after her last birth. What should the nurse include in the plan of follow-up care for this client? 1. One visit from a homecare nurse, to take place in 2 days 2. Two visits from a public health nurse over the next month 3. An appointment with a mental health counselor 4. Follow-up with the obstetrician in 6 weeks

C

35) A nurse is caring for several postpartum clients. Which client is demonstrating a problem attaching to her newborn? 1. The client who is discussing how the baby looks like her father 2. The client who is singing softly to her baby 3. The client who continues to touch her baby with only her fingertips 4. The client who picks her baby up when the baby cries

C

37) The community nurse is working with a client whose only child is 8 months old. Which statement does the nurse expect the mother to make? 1. I have a lot more time to myself than I thought I would have. 2. My confidence level in my parenting is higher than I anticipated. 3. I am constantly tired. I feel like I could sleep for a week. 4. My baby likes everyone, and never fusses when shes held by a stranger.

C

7) While a child is being admitting to the hospital, the parent receives information about the pediatric unit's goals, including the statement that the unit practices family-centered care. The parent asks why that is important. The nurse responds that what communication dynamic is characteristic of the family-centered care paradigm? A) The mother is the principal caregiver in each family. B) The child's physician is the key person in ensuring that the health of a child is maintained. C) The family serves as the constant influence and continuing support in the child's life. D) The father is the leader in each home; thus, all communications should include him.

C

8) On the 3rd day postpartum, a client who is not breastfeeding experiences engorgement. To relieve her discomfort, the nurse should encourage the client to do which of the following? 1. Remove her bra 2. Apply heat to the breasts 3. Apply cold packs to the breasts 4. Use a breast pump to release the milk

C

8) The postpartum client is concerned about mastitis because she experienced it with her last baby. Preventive measures the nurse can teach include which of the following? 1. Wearing a tight-fitting bra 2. Limiting breastfeedings 3. Frequent breastfeedings 4. Restricting fluid intake

C

9) A postpartum client reports sharp, shooting pains in her nipple during breastfeeding and flaky, itchy skin on her breasts. Which of the following does the nurse suspect? 1. Nipple soreness 2. Engorgement 3. Mastitis 4. Letdown reflex

C

9) The nurse is providing discharge teaching to a woman who delivered her first child 2 days ago. The nurse understands that additional information is needed if the client makes which statement? 1. I should expect a lighter flow next week. 2. The flow will increase if I am too active. 3. My bleeding will remain red for about a month. 4. I will be able to use a pantiliner in a day or two.

C

A 2-day-old newborn is asleep, and the nurse assesses the apical pulse to be 88 beats/min. What would be the most appropriate nursing action based on this assessment finding? 1. Call the physician. 2. Administer oxygen. 3. Document the finding. 4. Place the newborn under the radiant warmer.

C

A 38-year-old female is scheduled for a laparoscopic-assisted vaginal hysterectomy (L A V H) for severe endometriosis with the removal of both ovaries. What should the nurse expect to be prescribed for this patient postoperatively? Corticosteroid therapy Mineralocorticoid therapy Estrogen replacement therapy Progesterone replacement therapy

C

A client comes into the prenatal clinic accompanied by her boyfriend. When asked by the nurse why she is there, the client looks down, and the boyfriend states, "She says she is pregnant. She constantly complains of feeling tired, and her vomiting is disgusting." What is a priority for the nurse to do at this point? 1. Ask the client what time of the day her fatigue is more common. 2. Recommend that the woman have a pregnancy test done as soon as possible. 3. Continue the interview of the client in private. 4. Give the woman suggestions on ways to decrease the vomiting.

C

A client's labor has progressed so rapidly that a precipitous birth is occurring. What should the nurse do? 1. Go to the nurse's station and immediately call the physician. 2. Run to the delivery room for an emergency birth pack. 3. Stay with the client and ask auxiliary personnel for assistance. 4. Hold back the infant's head forcibly until the physician arrives for the delivery.

C

A group of nurses are meeting as identified in the image below. Which behavior are the nurses demonstrating during this meeting? A) Privacy B) Advocacy C) Collaboration D) Informed Consent

C

A new father asks the nurse to describe what his baby will experience while sleeping and awake. What is the best response? 1. Babies have several sleep and alert states. Keep watching and youll notice them. 2. You might have noticed that your child was in an alert awake state for an hour after birth. 3. Newborns have two stages of sleep: deep or quiet sleep and rapid eye movement sleep. 4. Birth is hard work for babies. It takes them a week or two to recover and become more awake.

C

A new grandfather is marveling over his 12-hour-old newborn grandson. Which statement indicates that the grandfather needs additional education? 1. I cant believe he can already digest fats, carbohydrates, and proteins. 2. It is amazing that his whole digestive tract can move things along at birth. 3. Incredibly, his stomach capacity was already a cupful when he was born. 4. He will lose some weight but then miraculously regain it by about 10 days.

C

A nurse explains to new parents that their newborn has developed respiratory distress syndrome (RDS). Which of the following signs and symptoms would not be characteristic of RDS? 1. Grunting respirations 2. Nasal flaring 3. Respiratory rate of 40 during sleep 4. Chest retractions

C

A patient receiving chemotherapy for breast cancer writes in a journal during the treatments and reads devotional material. Which phase of psychologic adjustment should the nurse identify that this patient is experiencing? Shock Denial Reaction Recovery

C

A woman has been in labor for 16 hours. Her cervix is dilated to 3 cm and is 80% effaced. The fetal presenting part is not engaged. The nurse would suspect which of the following? 1. Breech malpresentation 2. Fetal demise 3. Cephalopelvic disproportion (CPD) 4. Abruptio placentae

C

A woman with polycystic ovarian syndrome (P C O S) is prescribed clomiphene citrate for the treatment of infertility. Which statement does the nurse understand is true? The woman has abnormal ovaries The woman has low prolactin levels The woman's pituitary gland is intact The woman's thyroid gland is normal

C

A young adolescent is transferred to the labor and delivery unit from the emergency department. The client is in active labor, but did not know she was pregnant. What is the most important nursing action? 1. Determine who might be the father of the baby for paternity testing. 2. Ask the client what kind of birthing experience she would like to have. 3. Assess blood pressure and check for proteinuria. 4. Obtain a Social Services referral to discuss adoption.

C

After noting meconium-stained amniotic fluid and fetal heart rate decelerations, the physician diagnoses a depressed fetus. The appropriate nursing action at this time would be to do what? 1. Increase the mother's oxygen rate. 2. Turn the mother to the left lateral position. 3. Prepare the mother for a higher-risk delivery. 4. Increase the intravenous infusion rate.

C

An expectant father has been at the bedside of his laboring partner for more than 12 hours. An appropriate nursing intervention would be to do which of the following? 1. Insist that he leave the room for at least the next hour. 2. Tell him he is not being as effective as he was, and that he needs to let someone else take over. 3. Offer to remain with his partner while he takes a break. 4. Suggest that the client's mother might be of more help.

C

Approximately what percentage of the newborn's body weight is water? 5% to 10% 90% to 95% 70% to 75% 50% to 60%

C

At 1 minute after birth, the infant has a heart rate of 100 beats per minute, and is crying vigorously. The limbs are flexed, the trunk is pink, and the feet and hands are cyanotic. The infant cries easily when the soles of the feet are stimulated. How would the nurse document this infant's Apgar score? 1. 7 2. 8 3. 9 4. 10

C

Before a newborn and mother are discharged from the hospital, the nurse informs the parents about routine screening tests for newborns. What is a good reason for having the screening tests done? 1. The tests prevent infants from developing phenylketonuria. 2. The tests detect such disorders as hypertension and diabetes. 3. The tests detect disorders that cause physical, intellectual, and developmental complications or death if left undiscovered. 4. The tests prevent sickle-cell anemia, galactosemia, and homocystinuria.

C

Client safety goals, which are evaluated and updated regularly, are requirements for what? A) Clinical practice guidelines B) Scope of practice C) Accreditation D) Standards of care

C

During an assessment of a 12-hour-old newborn, the nurse notices pale pink spots on the nape of the neck. The nurse documents this finding as which of the following? 1. Nevus vasculosus 2. Nevus flammeus 3. Telangiectatic nevi 4. A Mongolian spot

C

Persistent early decelerations are noted. What would the nurse's first action be? 1. Turn the mother on her left side and give oxygen. 2. Check for prolapsed cord. 3. Do nothing. This is a benign pattern. 4. Prepare for immediate forceps or cesarean delivery.

C

Screening for gestational diabetes mellitus (GDM) is typically completed between which of the following weeks of gestation? 1. 36 and 40 weeks 2. Before 20 weeks 3. 24 and 28 weeks 4. 30 and 34 weeks

C

Specific cellular immunity is mediated by T lymphocytes, which enhance the efficiency of the phagocytic response. What do cytotoxic activated T cells do? 1. Enable T or B cells to respond to antigens 2. Repress responses to specific B or T lymphocytes to antigens 3. Kill foreign or virus-infected cells 4. Remove pathogens and cell debris

C

The client at 39 weeks' gestation calls the clinic and reports increased bladder pressure but easier breathing and irregular, mild contractions. She also states that she just cleaned the entire house. Which statement should the nurse make? 1. "You shouldn't work so much at this point in pregnancy." 2. "What you are describing is not commonly experienced in the last weeks." 3. "Your body may be telling you it is going into labor soon." 4. "If the bladder pressure continues, come in to the clinic tomorrow."

C

The client demonstrates understanding of the implications for future pregnancies secondary to her classic uterine incision when she states which of the following? 1. "The next time I have a baby, I can try to deliver vaginally." 2. "The risk of rupturing my uterus is too high for me to have any more babies." 3. "Every time I have a baby, I will have to have a cesarean delivery." 4. "I can only have one more baby."

C

The client diagnosed with endometriosis asks the nurse whether there are any long-term health risks associated with this condition. The nurse should include which statement in the client teaching about endometriosis? "There are no other health risks associated with endometriosis." "Pain with intercourse rarely occurs as a long-term problem." "You are at increased risk for ovarian and breast cancer." "Most women with this condition develop severe migraines."

C

The client has been pushing for 2 hours and is exhausted. The physician is performing a vacuum extraction to assist the birth. Which finding is expected and normal? 1. The head is delivered after eight "pop-offs" during contractions. 2. A cephalohematoma is present on the fetal scalp. 3. The location of the vacuum is apparent on the fetal scalp after birth. 4. Positive pressure is applied by the vacuum extraction during contractions.

C

The client has delivered her first child at 37 weeks. The nurse would describe this to the client as what type of delivery? 1. Preterm 2. Postterm 3. Early term 4. Near term

C

The client in early labor asks the nurse what the contractions are like as labor progresses. What would the nurse respond? 1. "In normal labor, as the uterine contractions become stronger, they usually also become less frequent." 2. "In normal labor, as the uterine contractions become stronger, they usually also become less painful." 3. "In normal labor, as the uterine contractions become stronger, they usually also become longer in duration." 4. "In normal labor, as the uterine contractions become stronger, they usually also become shorter in duration."

C

The client is being admitted to the birthing unit. As the nurse begins the assessment, the client's partner asks why the fetus's heart rate will be monitored. After the nurse explains, which statement by the partner indicates a need for further teaching? 1. "The fetus's heart rate will vary between 110 and 160." 2. "The heart rate is monitored to see whether the fetus is tolerating labor." 3. "By listening to the heart, we can tell the gender of the fetus." 4. "After listening to the heart rate, you will contact the midwife."

C

The client is recovering from a delivery that included a midline episiotomy. Her perineum is swollen and sore. Ten minutes after an ice pack is applied, the client asks for another. What is the best response from the nurse? 1. "I'll get you one right away." 2. "You only need to use one ice pack." 3. "You need to leave it off for at least 20 minutes and then reapply." 4. "I'll bring you an extra so that you can change it when you are ready."

C

The labor and delivery nurse is assigned to four clients in early labor. Which electronic fetal monitoring finding would require immediate intervention? 1. Early decelerations with each contraction 2. Variable decelerations that recover to the baseline 3. Late decelerations with minimal variability 4. Accelerations

C

The labor and delivery nurse is preparing a prenatal class about facilitating the progress of labor. Which of the following frequent responses to pain should the nurse indicate is most likely to impede progress in labor? 1. Increased pulse 2. Elevated blood pressure 3. Muscle tension 4. Increased respirations

C

The laboring client with meconium-stained amniotic fluid asks the nurse why the fetal monitor is necessary, as she finds the belt uncomfortable. Which response by the nurse is most important? 1. "The monitor is necessary so we can see how your labor is progressing." 2. "The monitor will prevent complications from the meconium in your fluid." 3. "The monitor helps us to see how the baby is tolerating labor." 4. "The monitor can be removed, and oxygen given instead."

C

The nurse attempts to elicit the Moro reflex on a newborn, and assesses movement of the right arm only. Based on this finding, the nurse immediately assesses for which of the following? Ortolani maneuver Palmar grasping reflex Clavicle Tonic neck reflex

C

The nurse has instructed a new mother on quieting activities for her newborn. The nurse knows that the mother understands when she overhears the mother telling the father to do what? 1. Hold the newborn in an upright position. 2. Massage the hands and feet. 3. Swaddle the newborn in a blanket. 4. Make eye contact while talking to the newborn.

C

The nurse has just palpated contractions and compares the consistency to that of the forehead to estimate the firmness of the fundus. What would the intensity of these contractions be identified as? 1. Mild 2. Moderate 3. Strong 4. Weak

C

The nurse has received the shift change report on infants born within the previous 4 hours. Which newborn should the nurse see first? 1. 37-week male, respiratory rate 45 2. 8 pound 1 ounce female, pulse 150 3. Term male, nasal flaring 4. 4-hour-old female who has not voided

C

The nurse in the prenatal clinic is seeing a pregnant 16-year-old for the first time. What comment by the young client is the most critical for the nurse to address first? 1. "My favorite lunch is burger and fries." 2. "I've been dating my new boyfriend for 2 weeks." 3. "On weekends, we go out and drink a few beers." 4. "I dropped out of school about 3 months ago."

C

The nurse is about to tell a client that her Pap smear result was abnormal. Which statement should the nurse include? "The Pap smear is used to diagnose cervical cancer." "A loop electrosurgical excision procedure (L E E P) is needed." "Colposcopy to further examine your cervix is the next step." "Your cervix needs to be treated with cryotherapy."

C

The nurse is admitting a client to the labor and delivery unit. Which aspect of the client's history requires notifying the physician? 1. Blood pressure 120/88 2. Father a carrier of sickle-cell trait 3. Dark red vaginal bleeding 4. History of domestic abuse

C

The nurse is analyzing several fetal heart rate patterns. The pattern that would be of most concern to the nurse would be which of the following? 1. Moderate variability 2. Early decelerations 3. Late decelerations 4. Accelerations

C

The nurse is assessing an obese pregnant client during a routine prenatal visit. Which is the priority assessment for this client? Complete blood count (C B C) Basic metabolic panel (B M P) Blood pressure Fetal heart rate

C

The nurse is aware that a fetus that is not in any stress would respond to a fetal scalp stimulation test by showing which change on the monitor strip? 1. Late decelerations 2. Early decelerations 3. Accelerations 4. Fetal dysrhythmia

C

The nurse is aware that labor and birth will most likely proceed normally when the fetus is in what position? 1. Right-acromion-dorsal-anterior 2. Right-sacrum-transverse 3. Occiput anterior 4. Posterior position

C

The nurse is caring for a client diagnosed with endometriosis. Which statement by the client would require a need for perhaps another treatment option? "I am having many hot flashes since I had the Lupron injection." "The pain I experience with intercourse is becoming more severe." "I have vaginal dryness, reduced libido, and my clitoris has become larger since taking danazol. Is this normal?" "I've noticed I have not had my period on a regular basis since being on the G n R H analogs

C

The nurse is caring for a client experiencing a uterine rupture. Which outcome demonstrates that the plan of care has been effective for the client? A) The mother remains hemodynamically stable throughout emergency cesarean birth. B) The mother has additional knowledge regarding the problems, implications, and treatment plans. C) The FHR remains in normal range with supportive measures. D) The family is able to cope successfully with fetal or neonatal anomalies, if they exist.

C

The nurse is caring for an infant who was delivered in a car on the way to the hospital and who has developed cold stress. Which finding requires immediate intervention? 1. Increased skin temperature and respirations 2. Blood glucose level of 45 3. Room-temperature IV running 4. Positioned under radiant warmer

C

The nurse is completing a newborn care class. The nurse knows that teaching has been effective if a new parent states which of the following? "My baby might open her arms wide and pull her legs up to her tummy if she is passing gas." "When I hold my baby upright with one of his feet on the floor, his feet will automatically remain still." "When I put my finger in the palm of my daughter's hand, she will curl her fingers and hold on." "I can get my baby to turn his head toward the right if I lift his right arm over his head."

C

The nurse is instructing a new mother on circumcision care with a Plastibell. The nurse knows the mother understands when she states that the Plastibell should fall off within how long? 1. 2 days 2. 10 days 3. 8 days 4. 14 days

C

The nurse is making an initial assessment of the newborn. The findings include a chest circumference of 32.5 c m and a head circumference of 33.5 c Based on these findings, which action should the nurse take first? Notify the physician. Elevate the newborn's head. Document the findings in the chart. Assess for hypothermia immediately.

C

The nurse is measuring the fundal height of a patient who is at 28 weeks' gestation using the following method. What would be considered a normal finding? 22 c m 24 c m 28 c m 30 c m

C

The nurse is performing a postpartum homecare visit. Which teaching has the highest priority? 1. Teaching or reviewing how to bathe the baby 2. Teaching how to thoroughly childproof the house 3. How many wet diapers the baby should have daily 4. Prevention of plagiocephaly

C

The nurse is planning a homecare visit to a mother who just recently delivered. The neighborhood is known to have a significant crime rate. What should the nurse do when planning this visit to facilitate personal safety? 1. Be friendly to all pets encountered on the visit to build client rapport. 2. Wait to find the exact location until arrival in the neighborhood. 3. Put personal possessions in the trunk when leaving the office. 4. Wear flashy jewelry to garner respect.

C

The nurse is teaching a class on infant care to new parents. Which statement by a parent indicates that additional teaching is needed? 1. "The white spots on my baby's nose are called milia, and are harmless." 2. "The whitish cheeselike substance in the creases is vernix, and will be absorbed." 3. "The red spots with a white center on my baby are abnormal acne." 4. "Jaundice is a yellowish discoloration of skin that if noticed on the 1st day of life should be reported to the physician."

C

The nurse is teaching a client who has been diagnosed with vulvitis. Which statement by the client indicates that the nurse's instruction has not been effective? "I should stop having sexual intercourse." "Non-deodorized tampons could make this condition recur." "Wearing pantyhose daily will improve the problem." "A different brand of soap might eliminate the irritation."

C

The nurse is using the New Ballard Score to assess the gestational age of a newborn delivered 4 hours ago. The infant's gestational age is 33 weeks based on early ultrasound and last menstrual period. The nurse expects the infant to exhibit which of the following? 1. Full sole creases, nails extending beyond the fingertips, scarf sign showing the elbow beyond the midline 2. Testes located in the upper scrotum, rugae covering the scrotum, vernix covering the entire body 3. Ear cartilage folded over, lanugo present over much of the body, slow recoil time 4. 1 cm breast bud, peeling skin and veins not visible, rapid recoil of legs and arms to extension

C

The nurse is working with a student nurse during assessment of a 2-hour-old newborn. Which action indicates that the student nurse understands neonatal assessment? The student nurse listens to bowel sounds, then assesses the head for skull consistency and size and tension of fontanelles. The student nurse checks for Ortolani's sign, then palpates the femoral pulse, then assesses respiratory rate. The student nurse determines skin color, then describes the shape of the chest and looks at structures and flexion of the feet. The student nurse counts the number of cord vessels, then assesses genitals, then sclera color and eyelids.

C

The nurse knows that in some cases, breastfeeding is not advisable. Which mother should be counseled against breastfeeding? 1. A mother with a poorly balanced diet 2. A mother who is overweight 3. A mother who is HIV positive 4. A mother who has twins

C

The nurse should explain to new parents that their infant's position should be changed periodically during the early months of life to prevent which of the following? 1. Muscle contractures 2. Respiratory distress 3. Permanently flattened areas of the skull 4. Esophageal reflux

C

The nurse teaches the parents of an infant who recently was circumcised to observe for bleeding. What should the parents be taught to do if bleeding does occur? 1. Wrap the diaper tightly. 2. Clean with warm water with each diaper change. 3. Apply gentle pressure to the site with gauze. 4. Apply a new petroleum ointment gauze dressing.

C

The nurse walks in to find the client crying after the physician informed her of her diagnosis of human papilloma virus (H P V). Which statement by the nurse conveys an attitude of acceptance toward the client with a sexually transmitted infection? "Don't worry about it. In a few weeks, with treatment, the lesions will disappear." "You seem upset. I'll get the doctor." "You seem upset. Can I help answer any questions?" "I think you need to see a therapist."

C

The nurse wants to demonstrate to a new family their infant's individuality. Which assessment tool would be most appropriate for the nurse to use? A. Ortolani's maneuver B. Ballard Maturity Scale C. Brazelton Neonatal Behavioral Assessment Scale D. Dubowitz Gestational Age Scale

C

The parents of a newborn male ask the nurse whether they should circumcise their son. What is the best response by the nurse? 1. "Circumcision should be undertaken to prevent problems in the future." 2. "Circumcision might decrease the child's risk of developing a urinary tract infection." 3. "Circumcision can sometimes cause complications. What questions do you have?" 4. "Circumcision is painful, and should be avoided unless you are Jewish."

C

The parents of a preterm newborn wish to visit their baby in the NICU. A statement by the nurse that would not support the parents as they visit their newborn is which of the following? 1. "Your newborn likes to be touched." 2. "Stroking the newborn will help with stimulation." 3. "Visits must be scheduled between feedings." 4. "Your baby loves her pink blanket."

C

The postpartum nurse is performing a homecare visit to a first-time mother on her third day after delivery. She reports that her nipples are becoming sore. What statement indicates that further teaching is needed? 1. "I can apply lanolin cream to help with the nipple pain." 2. "Watching how much areola is visible will help me see whether my baby has a good mouthful of breast or not." 3. "My nipples will heal if I switch to bottle feeding for about 3 days while I pump my breasts." 4. "Rotating breastfeeding positions will allow the sore areas of my nipples to have less friction."

C

The student nurse is to perform Leopold maneuvers on a laboring client. Which assessment requires intervention by the staff nurse? 1. The client is assisted into supine position, and the position of the fetus is assessed. 2. The upper portion of the uterus is palpated, then the middle section. 3. After determining where the back is located, the cervix is assessed. 4. Following voiding, the client's abdomen is palpated from top to bottom.

C

To prevent sudden infant death syndrome (SIDS), the nurse encourages the parents of a term infant to place the infant in which position when the infant is sleeping? 1. On the parents' waterbed 2. Swaddled in the infant swing 3. On the back 4. On the sides

C

What would the nurse include as part of a routine physical assessment for a second-trimester primiparous patient whose prenatal care began in the first trimester and is ongoing? 1. Pap smear 2. Hepatitis B screening (HBsAg) 3. Fundal height measurement 4. Complete blood count

C

When a breastfeeding mother complains that her breasts are leaking milk, the nurse can offer which effective intervention? 1. Decrease the number of minutes the newborn is at the breast per feeding. 2. Decrease the mother's fluid intake. 3. Place absorbent pads in the bra. 4. Administer oxytocin.

C

When is breastfeeding contraindicated? 1. Infant has hypertension 2. Mother has a history of treated tuberculosis 3. Mother is HIV positive or has AIDS 4. Mother has a history of treated herpes

C

Which of the following is the primary carbohydrate in the breastfeeding newborn? 1. Glucose 2. Fructose 3. Lactose 4. Maltose

C

38) The nurse is preparing a class for mothers and their partners who have just recently delivered. One topic of the class is infant attachment. Which statement by a participant indicates an understanding of this concept? Select all that apply. 1. We should avoid holding the baby too much. 2. Looking directly into the babys eyes might frighten him. 3. Talking to the baby is good because hell recognize our voices. 4. Holding the baby so we have direct face-to-face contact is good. 5. We should only touch the baby with our fingertips for the first month.

C, D

A nurse is providing a client with instructions regarding breast self-examination (B S E). Which of the following statements by the client would indicate that the teaching has been successful? Select all that apply. "I should perform B S E 1 week prior to the start of my period." "When I reach menopause, I will perform B S E every 2 months." "Knowing the density of my breast tissue is important." "I should inspect my breasts while standing with my arms down at my sides." "I should inspect my breasts while in a supine position with my arms at my sides."

C, D

Lacerations of the cervix or vagina may be present when bright red vaginal bleeding persists in the presence of a well-contracted uterus. The incidence of lacerations is higher among which of the following childbearing women? Note: Credit will be given only if all correct choices and no incorrect choices are selected. Select all that apply. 1. Over the age of 35 2. Have not had epidural block 3. Have had an episiotomy 4. Have had a forceps-assisted or vacuum-assisted birth 5. Nulliparous

C, D

The labor and birth nurse is admitting a client. The nurse's assessment includes asking the client whom she would like to have present for the labor and birth, and what the client would prefer to wear. The client's partner asks the nurse the reason for these questions. What would the nurse's best response be? Select all that apply. 1. "These questions are asked of all women. It's no big deal." 2. "I'd prefer that your partner ask me all the questions, not you." 3. "A client's preferences for her birth are important for me to understand." 4. "Many women have beliefs about childbearing that affect these choices." 5. "I'm gathering information that the nurses will use after the birth."

C, D

30) The nurse is planning discharge teaching for a postpartum woman. What information recommendations should the woman receive before being discharged? Select all that apply. 1. To abstain from sexual intercourse for 6 months 2. To avoid showers for 4 weeks 3. To avoid overexertion 4. To practice postpartum exercises 5. To obtain adequate rest

C, D, E

A pregnant client has a hemoglobin of 10 g/dL and a Hct of 30%. The clinic nurse recognizes the fetus is at risk for which of the following? Note: Credit will be given only if all correct and no incorrect choices are selected. Select all that apply. 1. Macrosomia 2. Respiratory distress syndrome 3. Low birth weight 4. Prematurity 5. Fetal death

C, D, E

A woman is scheduled to have an external version for a breech presentation. The nurse carefully reviews the client's chart for contraindications to this procedure, including which of the following? Note: Credit will be given only if all correct choices and no incorrect choices are selected. Select all that apply. 1. Station -2 2. 38 weeks' gestation 3. Abnormal fetal heart rate and tracing 4. Previous cesarean section 5. Rupture of membranes

C, D, E

An abbreviated systematic physical assessment of the newborn is performed by the nurse in the birthing area to detect any abnormalities. Normal findings would include which of the following? Note: Credit will be given only if all correct choices and no incorrect choices are selected. Select all that apply. 1. Skin color: Body blue with pinkish extremities 2. Umbilical cord: two veins and one artery 3. Respiration rate of 30-60 irregular 4. Temperature of above 36.5°C (97.8°F) 5. Sole creases that involve the heel

C, D, E

The nurse is caring for laboring clients. Which women are experiencing problems related to a critical factor of labor? Note: Credit will be given only for all correct choices and no incorrect choices. Select all that apply. 1. Woman at 7 cm, fetus in general flexion 2. Woman at 3 cm, fetus in longitudinal lie 3. Woman at 4 cm, fetus with transverse lie 4. Woman at 6 cm, fetus at -2 station, mild contractions 5. Woman at 5 cm, fetal presenting part is right shoulder

C, D, E

The nurse is meeting with a new mother for the first time during a home visit. The client delivered her first child 3 days ago. She had a normal pregnancy and a vaginal delivery. The infant is breastfeeding. Which statements by the mother indicate that she needs more information about the home visit? Note: Credit will be given only if all correct choices and no incorrect choices are selected. Select all that apply. 1. "You are going to check my baby's weight." 2. "You are going to watch me nurse the baby and give me tips." 3. "You are going to teach my mother about the baby." 4. "You are checking for safety issues when my son starts crawling." 5. "You are going to take blood samples from me and my son."

C, D, E

The nurse takes a telephone call from a women's health clinic patient. What information should cause the nurse to suspect that the patient is experiencing a cystocele? Select all that apply. Bloody urine Low back pain Onset of stress incontinence Feeling of fullness in the perineum Feels like something "fell out" of the vagina

C, D, E

True postterm pregnancies are frequently associated with placental changes that cause a decrease in the uterine-placental-fetal circulation. Complications related to alternations in placenta functioning include which of the following? Note: Credit will be given only if all correct choices and no incorrect choices are selected. Select all that apply. 1. Increased fetal oxygenation 2. Increased placental blood supply 3. Reduced nutritional supply 4. Macrosomia 5. Risk of shoulder dystocia

C, D, E

True postterm pregnancies are frequently associated with placental changes that cause a decrease in uterine-placental-fetal circulation. Complications related to alterations in placenta functioning include which of the following?Note: Credit will be given only if all correct choices and no incorrect choices are selected. Select all that apply. A) Increased fetal oxygenation B) Increased placental blood supply C) Reduced nutritional supply D) Macrosomia E) Risk of shoulder dystocia

C, D, E

What should the healthcare provider consider when prescribing a medication to a woman who is breastfeeding? Note: Credit will be given only if all correct choices and no incorrect choices are selected. Select all that apply. 1. Drug's potential effect on hormone production 2. Amount of drug excreted into the mother's blood 3. Drug's potential adverse effects to the infant 4. Infant's age and health 5. Mother's need for the medication

C, D, E

Five clients are in active labor in the labor unit. Which women should the nurse monitor carefully for the potential of uterine rupture? Note: Credit will be given only if all correct choices and no incorrect choices are selected. Select all that apply. 1. Age 15, in active labor 2. Age 22, with eclampsia 3. Age 25, last delivery by cesarean section 4. Age 32, first baby died during labor 5. Age 27, last delivery 11 months ago

C, E

The nurse is assessing the newborn for symptoms of anemia. If the blood loss is acute, the baby may exhibit which of the following signs of shock? Note: Credit will be given only if all correct choices and no incorrect choices are selected. Select all that apply. 1. Increased pulse 2. High blood pressure 3. Tachycardia 4. Bradycardia 5. Capillary filling time greater than 3 seconds

C, E

The nurse is teaching experienced postpartum nurses about homecare visits. Which statements indicate that teaching was effective? Note: Credit will be given only if all correct choices and no incorrect choices are selected. Select all that apply. 1. "I should tell the family to put any guns or knives away." 2. "It is best to blend in with the community and not bring attention to myself on visits." 3. "If I encounter a crime in progress, I should leave the area." 4. "Wearing jewelry is a good way to demonstrate my professionalism." 5. "Ignoring my 'gut' feelings might lead to an unsafe situation."

C, E

1) A 38-year-old patient is concerned that a month after becoming a widow, her menstrual cycles stopped. What should the nurse suspect as being the cause for this patient's secondary amenorrhea? A) Ovarian failure B) Pituitary dysfunction C) Anatomic abnormality D) Hypothalamic dysfunction

D

1) A client scheduled to have a Mirena levonorgestrel intrauterine system (L N g-I U C) inserted asks how this device stops conception. What should the nurse say in response to this client? A) "It stops ovulation." B) "It slows sperm motility." C) "It shortens the menstrual cycle." D) "It causes the lining of the uterus to waste away."

D

1) A female client who is 36 years old, weighs 200 pounds, is monogamous, and does not smoke desires birth control. The nurse understands that which contraceptive method is inappropriate for this client? A) Intrauterine device B) Vaginal sponge C) Combined oral contraceptives D) Transdermal hormonal contraception

D

1) A nonpregnant client is diagnosed with bacterial vaginosis (B V). What does the nurse expect to administer? A) Penicillin G 2 million units I M one time B) Zithromax 1 m g P O bid for 2 weeks C) Doxycycline 100 m g P O bid for a week D) Metronidazole 500 m g P O bid for a week

D

1) A nurse is examining different nursing roles. Which example best illustrates an advanced practice nursing role? A) A registered nurse who is the manager of a large obstetrical unit B) A registered nurse who is the circulating nurse during surgical deliveries (cesarean sections) C) A clinical nurse specialist working as a staff nurse on a mother-baby unit D) A clinical nurse specialist with whom other nurses consult for her expertise in caring for high-risk infants

D

1) A patient treated for a urinary tract infection a month ago is experiencing symptoms of the same infection. What should the nurse suspect is the reason for the reoccurrence of the infection? A) Using oral contraceptives B) Wearing cotton underwear C) Cleansing from front to back D) Stopped antibiotics after 3 days

D

1) A patient who is postmenopausal is encouraged to take calcium 1500 mg every day. How should the nurse instruct the patient to take this supplement? A) Take calcium 750 m g with breakfast and dinner B) Take the complete dose first thing in the morning C) Take the complete dose prior to bedtime every day D) Take calcium 500 m g three times a day with meals

D

1) A patient with amenorrhea has an elevated serum prolactin level. Which diagnostic test should the nurse expect will be prescribed for this patient? A) Laparoscopy B) Abdominal ultrasound C) C T scan of the abdomen D) Magnetic resonance imaging (M R I)

D

1) A sexually active female asks why an H I V test is needed since she uses condoms with spermicidal agents when having intercourse. How should the nurse respond to this client? A) "Condoms do not protect against contracting H I V." B) "Spermicides only control bacteria and not viruses." C) "All sexually active people are at risk for contracting H I V." D) "The spermicide can make your vaginal cells more susceptible to H I V."

D

1) A woman is in labor. The fetus is in vertex position. When the client's membranes rupture, the nurse sees that the amniotic fluid is meconium-stained. What should the nurse do immediately? A) Change the client's position in bed. B) Notify the physician that birth is imminent. C) Administer oxygen at 2 liters per minute. D) Begin continuous fetal heart rate monitoring.

D

1) After noting meconium-stained amniotic fluid and fetal heart rate decelerations, the physician diagnoses a depressed fetus. The appropriate nursing action at this time would be to do what? A) Increase the mother's oxygen rate. B) Turn the mother to the left lateral position. C) Prepare the mother for a higher-risk delivery. D) Increase the intravenous infusion rate.

D

1) After several hours of labor, the electronic fetal monitor (E F M) shows repetitive variable decelerations in the fetal heart rate. The nurse would interpret the decelerations to be consistent with which of the following? A) Breech presentation B) Uteroplacental insufficiency C) Compression of the fetal head D) Umbilical cord compression

D

1) Duvall's eight stages in the family life cycle of a traditional nuclear family have been used as the foundation for contemporary models that describe the developmental processes and role expectations for different family types. Which of the following is an example of Stage Ⅳ of this family life cycle? A) Families launching young adults (all children leave home) B) Families with preschool-age children (oldest child is between 2.5 and 6 years of age) C) Middle-aged parents (empty nest through retirement) D) Families with schoolchildren (oldest child is between 6 and 13 years of age)

D

1) The nurse is identifying complementary and alternative therapies for a patient with a history of liver disorders who is experiencing symptoms of menopause. Which herbal supplement should this patient be counseled to avoid? A) Ginger B) Ginseng C) Red clover D) Black cohosh

D

1) The nurse is preparing a community presentation on family development. Which statement should the nurse include? A) The youngest child determines the family's current stage. B) A family does not experience overlapping of stages. C) Family development ends when the youngest child leaves home. D) The stages describe the family's progression over time.

D

1) The nurse is preparing to assess the development of a family new to the clinic. The nurse understands that which of the following is the primary use of a family assessment tool? A) Obtain a comprehensive medical history of family members. B) Determine to which clinic the client should be referred. C) Predict how a family will likely change with the addition of children. D) Understand the physical, emotional, and spiritual needs of members.

D

1) The nurse is preparing to assess the sexual history of a 35-year-old female patient. Which approach should the nurse first use to facilitate this data collection? A) Ask if the patient is sexually active B) Review the present method of birth control C) Determine the patient's number of children D) Talk about the patient's medical-surgical history

D

1) The nurse is teaching a client who is having the Skyla L N g-I U C device inserted for contraception. What should the nurse emphasize to the client about this device? A) This device will provide protection for 5 years B) This device will provide protection for 10 years C) This device should not be used with a copper allergy D) This device has a silver ring and could interfere with an M R I

D

1) The nurse suspects that a patient is experiencing bacterial vaginosis. What finding caused the nurse to make this clinical determination? A) Dysuria B) Vaginal itching C) Thick white vaginal discharge D) Fishy odor to vaginal discharge

D

1) The nurse works in a facility that cares for clients from a broad range of racial, ethnic, cultural, and religious backgrounds. Which statement should the nurse include in a presentation to recently hired nurses on the client population of the facility? A) "Our clients come from a broad range of backgrounds, but we have a good interpreter service." B) "Many of our clients come from backgrounds different from your own, but it doesn't cause problems for the nurses." C) "Because most of the doctors are bilingual, we don't have to deal with the differences in cultural backgrounds of our clients." D) "Understanding the common values and health practices of our diverse clients will facilitate better care and health outcomes."

D

1) When teaching a culturally diverse group of childbearing families about hospital birthing options, the culturally competent nurse does which of the following? A) Understands that the families have the same values as the nurse B) Teaches the families how childbearing takes place in the United States C) Insists that the clients answer questions instead of their husbands D) Incorporates the specific beliefs of the cultural groups that are attending the class

D

1) Which client would the nurse document as exhibiting signs and symptoms of primary dysmenorrhea? A) 17-year-old, has never had a menstrual cycle B) 16-year-old, had regular menses for 4 years, but has had no menses in 4 months C) 19-year-old, regular menses for 5 years that have suddenly become painful D) 14-year-old, irregular menses for 1 year, experiences cramping every cycle

D

14) The hospital is developing a new maternity unit. What aspects should be included in the planning of the new unit to best promote family wellness? 1. Normal newborn nursery centrally located to all client rooms 2. A kitchen with a refrigerator stocked with juice and sandwiches 3. Small, cozy rooms with a client bed and rocking chair 4. A nursing care model based on providing couplet care

D

17) How does the nurse assess for Homans sign? 1. Extending the foot and inquiring about calf pain. 2. Extending the leg and inquiring about foot pain. 3. Flexing the knee and inquiring about thigh pain. 4. Dorsiflexing the foot and inquiring about calf pain.

D

22) The client delivered her first child vaginally 7 hours ago. She has not voided since delivery. She has an IV of lactated Ringers solution running at 100 mL/hr. Her fundus is firm and to the right of midline. What is the best nursing action? 1. To massage the fundus vigorously 2. To assess the clients pain level 3. To increase the rate of the IV 4. To assist the client to the bathroom

D

23) During a postpartum examination of a client who delivered an 8-pound newborn 6 hours ago, the following assessment findings are noted: fundus firm and at the umbilicus, and moderate lochia rubra with a steady trickle of blood from the vagina. What is the assessment finding that would necessitate follow-up? 1. Firm fundus 2. Fundus at the umbilical level 3. Moderate lochia rubra 4. Steady trickle of blood

D

25) The nurse is performing a postpartum assessment on a newly delivered client. When checking the fundus, there is a gush of blood. The client asks why that is happening. What is the nurses best response? 1. We see this from time to time. Its not a big deal. 2. The gush is an indication that your fundus isnt contracting. 3. Dont worry. Ill make sure everything is fine. 4. Blood pooled in the vagina while you were in bed.

D

27) The nurse is preparing to receive a newly delivered client. The client is a young single mother who is relinquishing custody of her newborn through an open adoption. What action is most important? 1. Assign the client a room on the GYN surgical floor instead of on the postpartum floor. 2. Prepare to complete teaching in time for discharge at 24 hours post-delivery. 3. Make an effort not to bring up the topic of the baby, and discuss the mothers health instead. 4. Ask the client how much contact she would like with the baby, and whether she wants to feed it.

D

30) A postpartum client with endometritis is being discharged home on antibiotic therapy. The new mother plans to breastfeed her baby. What should the nurses discharge instruction include? 1. The client can douche every other day. 2. Sexual intercourse can be resumed when the client feels up to it. 3. Light housework will provide needed exercise. 4. The babys mouth should be examined for thrush.

D

32) The client delivered her second child yesterday, and is preparing to be discharged. She expresses concern to the nurse because she developed an upper urinary tract infection (UTI) after the birth of her first child. Which statement indicates that the client needs additional teaching about this issue? 1. If I start to have burning with urination, I need to call the doctor. 2. Drinking 8 glasses of water each day will help prevent another UTI. 3. I will remember to wipe from front to back after I move my bowels. 4. Voiding 2 or 3 times per day will help prevent a recurrence.

D

32) To actively involve the postpartal client during discharge teaching, the postpartum nurse applies which learning principle? 1. Reprints of magazine articles 2. Classroom lectures 3. Audiotapes 4. Interactive nurse-patient relationships

D

36) The nurse is caring for a client who recently emigrated from a Southeast Asian country. The mother has been resting since the birth, while her sister has changed the diapers and fed the infant. What is the most likely explanation for this behavior? 1. The client is not attaching to her infant appropriately. 2. The client is not going to be a good mother, and the baby is at risk. 3. The client has no mother present to role-model behaviors. 4. The client is exhibiting normal behavior for her culture.

D

5) The nurse assesses the postpartum client who has not had a bowel movement by the third postpartum day. Which nursing intervention would be appropriate? 1. Encourage the new mother, saying, It will happen soon. 2. Instruct the client to eat a low-fiber diet. 3. Decrease fluid intake. 4. Obtain an order for a stool softener.

D

7) The nurse at an elementary school is performing T B screenings on all of the students. Permission slips were returned for all but the children of one family. When the nurse phones to obtain permission, the parent states in clearly understandable English that permission cannot be given because the grandmother is out of town for 2 more weeks. Which cultural element is contributing to the dilemma that faces the nurse? A) Permissible physical contact with strangers B) Beliefs about the concepts of health and illness C) Religion and social beliefs D) Presence and influence of the extended family

D

7) The registered nurse who has completed a master's degree program and passed a national certification exam has clinic appointments with clients who are pregnant or seeking well-woman care. What is the role of this nurse considered to be? A) Professional nurse B) Certified registered nurse (R N C) C) Clinical nurse specialist D) Nurse practitioner

D

8) To assess the healing of the uterus at the placental site, what does the nurse assess? 1. Lab values 2. Blood pressure 3. Uterine size 4. Type, amount, and consistency of lochia

D

A client at 20 weeks' gestation has not decided on a feeding method for her infant. She asks the nurse for advice. The nurse presents information about the advantages and disadvantages of formula-feeding and breastfeeding. Which statements by the client indicate that the teaching was successful? 1. "Formula-feeding gives the baby protection from infections." 2. "Breast milk cannot be stored; it has to be thrown away after pumping." 3. "Breastfeeding is more expensive than formula-feeding." 4. "My baby will have a lower risk of food allergies if I breastfeed."

D

A mother of a 16-week-old infant calls the clinic, and is concerned because she cannot feel the posterior fontanelle on her infant. Which of the responses by the nurse would be most appropriate? A. "Bring your infant to the clinic immediately." B. "Your baby must be dehydrated." C. "This is due to overriding of the cranial bones during labor." D. "It is normal for the posterior fontanelle to close by 8—12 weeks after birth."

D

A newborn is receiving phototherapy. Which intervention by the nurse would be most important? 1. Measurement of head circumference 2. Encouraging the mother to stop breastfeeding 3. Stool blood testing 4. Assessment of hydration status

D

A woman is in labor. The fetus is in vertex position. When the client's membranes rupture, the nurse sees that the amniotic fluid is meconium-stained. What should the nurse do immediately? 1. Change the client's position in bed. 2. Notify the physician that birth is imminent. 3. Administer oxygen at 2 liters per minute. 4. Begin continuous fetal heart rate monitoring.

D

After inserting prostaglandin gel for cervical ripening, what should the nurse do? 1. Apply an internal fetal monitor. 2. Insert an indwelling catheter. 3. Withhold oral intake and start intravenous fluids. 4. Place the client in a supine position with a right hip wedge.

D

After several hours of labor, the electronic fetal monitor (EFM) shows repetitive variable decelerations in the fetal heart rate. The nurse would interpret the decelerations to be consistent with which of the following? 1. Breech presentation 2. Uteroplacental insufficiency 3. Compression of the fetal head 4. Umbilical cord compression

D

In the operating room, a client is being prepped for a cesarean delivery. The doctor is present. What is the last assessment the nurse should make just before the client is draped for surgery? 1. Maternal temperature 2. Maternal urine output 3. Vaginal exam 4. Fetal heart tones

D

Nurses should educate parents about which of the following AAP recommendations to promote a safe sleep environment and decrease the risk of SIDS and SUID in infants less than 12 months of age? 1. Babies should not be offered a pacifier while falling asleep. 2. Babies should be bottlefed unless contraindicated. 3. Babies should be under many covers when sleeping to keep them warm. 4. Babies should have "tummy time" when they are awake.

D

The charge nurse has received the shift change report. Which client requires immediate intervention? 1. Woman at 6 cm undergoing induction of labor, strong contractions every 3 minutes 2. Woman at 4 cm whose fetus is in a longitudinal lie with a cephalic presentation 3. Woman at 10 cm and fetus at +2 station experiencing a strong expulsion urge 4. Woman at 3 cm screaming in fear because her mother died during childbirth

D

The clinic nurse is compiling data for a yearly report. Which client would be classified as a primigravida? 1. A client at 18 weeks' gestation who had a spontaneous loss at 12 weeks 2. A client at 13 weeks' gestation who had an ectopic pregnancy at 8 weeks 3. A client at 14 weeks' gestation who has a 3-year-old daughter at home 4. A client at 15 weeks' gestation who has never been pregnant before

D

The midwife performs a vaginal exam and determines that the fetal head is at a -2 station. What does this indicate to the nurse about the birth? 1. The birth is imminent. 2. The birth is likely to occur in 1-2 hours. 3. The birth will occur later in the shift. 4. The birth is difficult to predict.

D

The multiparous client at term has arrived to the labor and delivery unit in active labor with intact membranes. Leopold maneuvers indicate the fetus is in a transverse lie with a shoulder presentation. Which physician order is most important? 1. Artificially rupture membranes. 2. Apply internal fetal scalp electrode. 3. Monitor maternal blood pressure every 15 minutes. 4. Alert surgical team of urgent cesarean.

D

The nurse examines the client's placenta and finds that the umbilical cord is inserted at the placental margin. The client comments that the placenta and cord look different than they did for her first two births. The nurse should explain that this variation in placenta and cord is called what? 1. Placenta accreta 2. Circumvallate placenta 3. Succenturiate placenta 4. Battledore placenta

D

The nurse has just palpated a laboring woman's contractions. The uterus cannot be indented during a contraction. What would the intensity of these contractions best be characterized as? 1. Weak 2. Mild 3. Moderate 4. Strong

D

The nurse is analyzing various teaching strategies for teaching new mothers about newborn care. To enhance learning, which teaching method should the nurse implement? 1. Select videos on various topics of newborn care. 2. Organize a class that includes first-time mothers only. 3. Have mothers return in 1 week, when they feel more rested. 4. Schedule time for one-to-one teaching in the mother's room.

D

The nurse is assessing a newborn at 1 hour of age. Which finding requires an immediate intervention? 1. Respiratory rate 60 and irregular in depth and rhythm 2. Pulse rate 145, cardiac murmur heard 3. Mean blood pressure 55 mm Hg 4. Pauses in respiration lasting 30 seconds

D

The nurse is assessing a newly pregnant client. Which finding does the nurse note as a normal psychosocial adjustment in this client's first trimester? 1. An unlisted telephone number 2. Reluctance to tell the partner of the pregnancy 3. Parental disapproval of the woman's partner 4. Ambivalence about the pregnancy

D

The nurse is completing discharge teaching for a client who delivered 2 days ago. Which statement by the client indicates that further information is required? 1. "Because I have a midline episiotomy, I should keep my perineum clean." 2. "I can use an ice pack to relieve some the pain from the episiotomy." 3. "I can take ibuprofen (Motrin) when my perineum starts to hurt." 4. "The tear I have through my rectum is unrelated to my episiotomy."

D

The nurse is discharging a 15-year-old first-time mother. Which statement should the nurse include in the discharge teaching? 1. "Call your pediatrician if the baby's temperature is below 98.6°F axillary." 2. "Your baby's stools will change to a greenish color when your milk comes in." 3. "You can wipe away any eye drainage that might form." 4. "Your infant should wet a diaper at least 6 times per day."

D

The nurse is discussing parent-infant attachment with a prenatal class. Which statement indicates that teaching was successful? 1. "I should avoid looking directly into the baby's eyes to prevent frightening the baby." 2. "My baby will be very sleepy immediately after birth and should go to the nursery." 3. "Newborns cannot focus their eyes, so it doesn't matter how I hold my new baby." 4. "Giving the baby his first bath can really give me a chance to get to know him."

D

The nurse is explaining "quickening" to a client who is pregnant for the first time. Which client indicates the need for further education on this topic? "It will feel like butterflies in my stomach." "It might feel like I have gas." "It should occur during the second trimester of my pregnancy." "It is an indication that I am experiencing preterm labor."

D

The nurse is explaining induction of labor to a client. The client asks what the indications for labor induction are. Which of the following should the nurse include when answering the client? 1. Suspected placenta previa 2. Breech presentation 3. Prolapsed umbilical cord 4. Hypertension

D

The nurse is explaining the nutritional differences between breast milk and formula to an expectant couple. The mother-to-be asks whether breast milk is nutritionally superior to formula. What should the nurse reply? 1. The vitamins and minerals in formula are more bioavailable to the infant. 2. There is no cholesterol in breast milk. 3. The only carbohydrate in breast milk is lactose. 4. The ratio of whey to casein proteins in breast milk changes to meet the nutritional needs of the growing infant.

D

The nurse is instructing the parents of a newborn about car seat safety. Which statement indicates that the parents need additional information? 1. "The baby should be in the back seat." 2. "Newborns must be in rear-facing car seats." 3. "We need instruction on how to use the car seat before installing it." 4. "We can bring the baby home from the hospital without a car seat as it is only a short drive home."

D

The nurse is orienting a new graduate nurse to the labor and birth unit. Which statement indicates that teaching has been effective? 1. "When a client arrives in labor, a urine specimen is obtained by catheter to check for protein and ketones." 2. "When a client arrives in labor, she will be positioned supine to facilitate a normal blood pressure." 3. "When a client arrives in labor, her prenatal record is reviewed for indications of domestic abuse." 4. "When a client arrives in labor, a vaginal exam is performed unless birth appears to be imminent."

D

The nurse is planning home visits to the homes of new parents and their newborns. Which client should the nurse see first? 1. 3-day-old male who received hepatitis B vaccine prior to discharge 2. 4-day-old female whose parents are both hearing-impaired 3. 5-day-old male with light, sticky, yellow drainage on the circumcision site 4. 6-day-old female with greenish discharge from the umbilical cord site

D

The nurse is preparing to assess a newborn's neurological status. Which finding would require an immediate intervention? At rest, the infant has partially flexed arms and the legs drawn up to the abdomen. When the corner of the mouth is touched, the infant turns the head that direction. The infant blinks when the exam light is turned on over the face and body. The right arm is flaccid while the infant brings the left arm and fist upward to the head.

D

The nurse is working with an adolescent parent. The adolescent tells the nurse, "I'm really scared that I won't take care of my baby correctly. My mother says I'll probably hurt the baby because I'm too young to be a mother." What is the best response by the nurse? 1. "You are very young, and parenting will be a challenge for you." 2. "Your mother was probably right. Be very careful with your baby." 3. "Mothers have instincts that kick in when they get their babies home." 4. "We can give the baby's bath together. I'll help you learn how to do it."

D

The nurse manager is planning a presentation on ethical issues in caring for childbearing families. Which example should the nurse manager include to illustrate maternal-fetal conflict? A) A client chooses an abortion after her fetus is diagnosed with a genetic anomaly. B) A 39-year-old nulliparous client undergoes therapeutic insemination. C) A family of a child with leukemia requests cord-blood banking at a sibling's birth. D) A cesarean delivery of a breech fetus is court ordered after the client refuses.

D

The nurse reviewing charts for quality improvement notes that a client experienced a complication during labor. The nurse is uncertain whether the labor nurse took the appropriate action during the situation. What is the best way for the nurse to determine what the appropriate action should have been? A) Call the nurse manager of the labor and delivery unit and ask what the nurse should have done. B) Ask the departmental chair of the obstetrical physicians what the best nursing action would have been. C) Examine other charts to find cases of the same complication, and determine how it was handled in those situations. D) Look in the policy and procedure book, and examine the practice guidelines published by a professional nursing organization.

D

The nurse will be bringing the parents of a neonate with sepsis to the neonatal intensive care nursery for the first time. Which statement is best? 1. "I'll bring you to your baby and then leave so you can have some privacy." 2. "Your baby is on a ventilator with 50% oxygen, and has an umbilical line." 3. "I am so sorry this has all happened. I know how stressful this can be." 4. "Your baby is working hard to breathe and lying quite still, and has an IV."

D

The postpartum homecare nurse is assessing a new mother, and finds her temperature to be 101.6°F. What is the most important nursing action? 1. Ask the mother how often and how well the baby is nursing. 2. Determine the frequency of the mother's voiding and stooling. 3. Verify how many hours of sleep she is getting per day. 4. Assess the odor and color of the lochia and perineum.

D

The primigravida at 22 weeks' gestation has a fundal height palpated slightly below the umbilicus. Which of the following statements would best describe to the client why she needs to be seen by a physician today? 1. "Your baby is growing too much and getting too big." 2. "Your uterus might have an abnormal shape." 3. "The position of your baby can't be felt." 4. "Your baby might not be growing enough."

D

When comparing the anterior and posterior fontanelles of a newborn, the nurse knows that both are what? 1. Both are approximately the same size 2. Both close within 12 months of birth 3. Both are used in labor to identify station 4. Both allow for assessing the status of the newborn after birth

D

Which of the following is a sign of dehydration in the newborn? 1. Slow, weak pulse 2. Soft, loose stools 3. Light colored, concentrated urine 4. Depressed fontanelles

D

While caring for a client in labor, the nurse notices during a vaginal exam that the fetus's head has rotated internally. What would the nurse expect the next set of cardinal movements for a fetus in a vertex presentation to be? 1. Flexion, extension, restitution, external rotation, and expulsion 2. Expulsion, external rotation, and restitution 3. Restitution, flexion, external rotation, and expulsion 4. Extension, restitution, external rotation, and expulsion

D

Why is it important for the nurse to assess the bladder regularly and encourage the laboring client to void frequently? 1. A full bladder impedes oxygen flow to the fetus. 2. Frequent voiding prevents bruising of the bladder. 3. Frequent voiding encourages sphincter control. 4. A full bladder can impede fetal descent.

D

A woman who is 40 weeks pregnant calls the labor suite to ask whether she should be evaluated. Which statements by the client indicate she is likely in labor? Note: Credit will be given only for all correct choices and no incorrect choices. Select all that apply. 1. "The contractions are 5-20 minutes apart." 2. "I had pink discharge on the toilet paper." 3. "I have had cramping for the past 3-4 hours." 4. "The contractions start in my back and then go to my abdomen and are very intense." 5. "The contractions hurt more when I walk."

D, E

Which nursing intervention is appropriate in the management of the preterm infant with hypothermia? Note: Credit will be given only if all correct choices and no incorrect choices are selected. Select all that apply. 1. Warm the baby rapidly to reverse the hypothermia. 2. Monitor skin temperature every 2 hours to determine whether the infant's temperature is increasing. 3. Keep IV fluids at room temperature. 4. Initiate efforts to maintain the newborn in a neutral thermal environment. 5. Warm the baby slowly to reverse hypothermia and reach a neutral thermal environment

D, E

The mother of a newly circumcised infant is concerned about caring for the infant at home. What should the nurse instruct the mother about the infant's care? Place the following actions in the order that should be instructed to the mother. Pat dry Rinse area with warm water Fasten diaper snuggly over the penis Apply small amount of petroleum jelly Squeeze water over the circumcision site

E, B, A ,D, C

A female patient is anxious about having a pelvic examination. To help reduce the patient's fears in which order should the nurse explain that the examination will be performed? The speculum is inserted The speculum is removed The perineum is inspected The rectal examination is performed The healthcare provider applies gloves The bimanual examination is performed

E, C, A, B, F, D


Kaugnay na mga set ng pag-aaral

AS Physics Formulas and Definitions

View Set

Business Law Ch. 19 (Corporations)

View Set

macro: the Phillips curve: foundational concepts

View Set

Chapter 9. Section 1. Jefferson Takes Office

View Set

College American History 2 Chapter 10

View Set

Special Education and Processing Midterm

View Set

Chapter 9: Annuities (Life Only)

View Set

Level 1 - Chap 5: Leasehold Estates

View Set